Exam 1

Lakukan tugas rumah & ujian kamu dengan baik sekarang menggunakan Quizwiz!

The nurse is assessing a patient admitted with renal stones. During the admission assessment, what parameters would be priorities for the nurse to address? Select all that apply. A) Dietary history B) Family history of renal stones C) Medication history D) Surgical history E) Vaccination history

A, B, C Dietary and medication histories and family history of renal stones are obtained to identify factors predisposing the patient to stone formation. When caring for a patient with renal stones it would not normally be a priority to assess the vaccination history or surgical history, since these factors are not usually related to the etiology of kidney stones.

A patient is brought to the renal unit from the PACU status post resection of a renal tumor. Which of the following nursing actions should the nurse prioritize in the care of this patient? A) Increasing oral intake B) Managing postoperative pain C) Managing dialysis D) Increasing mobility

B The patient requires frequent analgesia during the postoperative period and assistance with turning, coughing, use of incentive spirometry, and deep breathing to prevent atelectasis and other pulmonary complications. Increasing oral intake and mobility are not priority nursing actions in the immediate postoperative care of this patient. Dialysis is not necessary following kidney surgery.

A nurse is caring for a patient who has just had a rigid fixation of a mandibular fracture. When planning the discharge teaching for this patient, what would the nurse be sure to include? A) Increasing calcium intake to promote bone healing B) Avoiding chewing food for the specified number of weeks after surgery C) Techniques for managing parenteral nutrition in the home setting D) Techniques for managing a gastrostomy

B The patient who has had rigid fixation should be instructed not to chew food in the first 1 to 4 weeks after surgery. A liquid diet is recommended, and dietary counseling should be obtained to ensure optimal caloric and protein intake. Increased calcium intake will not have an appreciable effect on healing. Enteral and parenteral nutrition are rarely necessary.

Results of a patient barium swallow suggest that the patient has GERD. The nurse is planning health education to address the patients knowledge of this new diagnosis. Which of the following should the nurse encourage? A) Eating several small meals daily rather than 3 larger meals B) Keeping the head of the bed slightly elevated C) Drinking carbonated mineral water rather than soft drinks D) Avoiding food or fluid intake after 6:00 p.m.

B The patient with GERD is encouraged to elevate the head of the bed on 6- to 8-inch (15- to 20-cm) blocks. Frequent meals are not specifically encouraged and the patient should avoid food and fluid within 2 hours of bedtime. All carbonated beverages should be avoided.

A 16-year-old presents at the emergency department complaining of right lower quadrant pain and is subsequently diagnosed with appendicitis. When planning this patients nursing care, the nurse should prioritize what nursing diagnosis? A) Imbalanced Nutrition: Less Than Body Requirements Related to Decreased Oral Intake B) Risk for Infection Related to Possible Rupture of Appendix C) Constipation Related to Decreased Bowel Motility and Decreased Fluid Intake D) Chronic Pain Related to Appendicitis

B The patient with a diagnosis of appendicitis has an acute risk of infection related to the possibility of rupture. This immediate physiologic risk is a priority over nutrition and constipation, though each of these concerns should be addressed by the nurse. The pain associated with appendicitis is acute, not chronic.

A patient with cirrhosis has experienced a progressive decline in his health; and liver transplantation is being considered by the interdisciplinary team. How will the patients prioritization for receiving a donor liver be determined? A) By considering the patients age and prognosis B) By objectively determining the patients medical need C) By objectively assessing the patients willingness to adhere to post-transplantation care D) By systematically ruling out alternative treatment options

B The patient would undergo a classification of the degree of medical need through an objective determination known as the Model of End-Stage Liver Disease (MELD) classification, which stratifies the level of illness of those awaiting a liver transplant. This algorithm considers multiple variables, not solely age, prognosis, potential for adherence, and the rejection of alternative options.

A patient admitted with acute diverticulitis has experienced a sudden increase in temperature and complains of a sudden onset of exquisite abdominal tenderness. The nurses rapid assessment reveals that the patients abdomen is uncharacteristically rigid on palpation. What is the nurses best response? A) Administer a Fleet enema as ordered and remain with the patient. B) Contact the primary care provider promptly and report these signs of perforation. C) Position the patient supine and insert an NG tube. D) Page the primary care provider and report that the patient may be obstructed.

B The patients change in status is suggestive of perforation, which is a surgical emergency. Obstruction does not have this presentation involving fever and abdominal rigidity. An enema would be strongly contraindicated. An order is needed for NG insertion and repositioning is not a priority.

A nurse is caring for a patient with gallstones who has been prescribed ursodeoxycholic acid (UDCA). The patient askshow this medicine is going to help his symptoms. The nurse should be aware of what aspect of this drugs pharmacodynamics? A) It inhibits the synthesis of bile. B) It inhibits the synthesis and secretion of cholesterol. C) It inhibits the secretion of bile. D) It inhibits the synthesis and secretion of amylase.

B UDCA acts by inhibiting the synthesis and secretion of cholesterol, thereby desaturating bile. UDCA does not directly inhibit either the synthesis or secretion of bile or amylase.

A patient who had a hemiglossectomy earlier in the day is assessed postoperatively, revealing a patent airway, stable vital signs, and no bleeding or drainage from the operative site. The nurse notes the patient is alert. What is the patients priority need at this time? A) Emotional support from visitors and staff B) An effective means of communicating with the nurse C) Referral to a speech therapist D) Dietary teaching focused on consistency of food and frequency of feedings

B Verbal communication may be impaired by radical surgery for oral cancer. It is therefore vital to assess the patients ability to communicate in writing before surgery. Emotional support and dietary teaching are critical aspects of the plan of care; however, the patients ability to communicate would be essential for both. Referral to a speech therapist will be required as part of the patients rehabilitation; however, it is not a priority at this particular time. Communication with the nurse is crucial for the delivery of safe and effective care.

A nurse who provides care in a long-term care facility is aware of the high incidence and prevalence of urinary tract infections among older adults. What action has the greatest potential to prevent UTIs in this population? A) Administer prophylactic antibiotics as ordered. B) Limit the use of indwelling urinary catheters. C) Encourage frequent mobility and repositioning. D) Toilet residents who are immobile on a scheduled basis.

B When indwelling catheters are used, the risk of UTI increases dramatically. Limiting their use significantly reduces an older adults risk of developing a UTI. Regular toileting promotes continence, but has only an indirect effect on the risk of UTIs. Prophylactic antibiotics are not normally administered. Mobility does not have a direct effect on UTI risk.

A nurse is preparing a plan of care for a patient with pancreatic cysts that have necessitated drainage through the abdominal wall. What nursing diagnosis should the nurse prioritize? A) Disturbed Body Image B) Impaired Skin Integrity C) Nausea D) Risk for Deficient Fluid Volume

B While each of the diagnoses may be applicable to a patient with pancreatic drainage, the priority nursing diagnosis is Impaired Skin Integrity. The drainage is often perfuse and destructive to tissue because of the enzyme contents. Nursing measures must focus on steps to protect the skin near the drainage site from excoriation. The application of ointments or the use of a suction apparatus protects the skin from excoriation.

The nurse is caring for a patient with acute glomerular inflammation. When assessing for the characteristic signs and symptoms of this health problem, the nurse should include which assessments? Select all that apply. A) Percuss for pain in the right lower abdominal quadrant. B) Assess for the presence of peripheral edema. C) Auscultate the patients apical heart rate for dysrhythmias. D) Assess the patients BP. E) Assess the patients orientation and judgment.

B, D Most patients with acute glomerular inflammation have some degree of edema and hypertension. Dysrhythmias, RLQ pain, and changes in mental status are not among the most common manifestations of acute glomerular inflammation.

An adult patient has been diagnosed with diverticular disease after ongoing challenges with constipation. The patient will be treated on an outpatient basis. What components of treatment should the nurse anticipate? Select all that apply. A) Anticholinergic medications B) Increased fiber intake C) Enemas on alternating days D) Reduced fat intake E) Fluid reduction

B, D Patients whose diverticular disease does not warrant hospital treatment often benefit from a high-fiber, low-fat diet. Neither enemas nor anticholinergics are indicated, and fluid intake is encouraged.

A 23-year-old woman is brought to the ED complaining of stomach cramps, nausea, vomiting, and diarrhea. The care team suspects food poisoning. What is the key to treatment in food poisoning? A) Administering IV antibiotics B) Assessing immunization status C) Determining the source and type of food poisoning D) Determining if anyone else in the family is ill

C Determining the source and type of food poisoning is essential to treatment, and is more important than determining other sick family members. Antibiotics are not normally indicated and immunizations are not relevant to diagnosis or treatment of food poisoning.

A patient with multiple trauma is brought to the ED by ambulance after a fall while rock climbing. What is a responsibility of the ED nurse in this patients care? A) Intubating the patient B) Notifying family members C) Ensuring IV access D) Delivering specimens to the laboratory

C ED nursing responsibilities include ensuring airway and IV access. Nurses are not normally responsible for notifying family members. Nurses collect specimens, but are not responsible for their delivery. Physicians or other team members with specialized training intubate the patient.

The nurse is providing a health education workshop to a group of adults focusing on cancer prevention. The nurse should emphasize what action in order to reduce participants risks of renal carcinoma? A) Avoiding heavy alcohol use B) Control of sodium intake C) Smoking cessation D) Adherence to recommended immunization schedules

C Tobacco use is a significant risk factor for renal cancer, surpassing the significance of high alcohol and sodium intake. Immunizations do not address an individuals risk of renal cancer.

A nurse is providing oral care to a patient who is comatose. What action best addresses the patients risk of tooth decay and plaque accumulation? A) Irrigating the mouth using a syringe filled with a bacteriocidal mouthwash B) Applying a water-soluble gel to the teeth and gums C) Wiping the teeth and gums clean with a gauze pad D) Brushing the patients teeth with a toothbrush and small amount of toothpaste

D Application of mechanical friction is the most effective way to cleanse the patients mouth. If the patient is unable to brush teeth, the nurse may brush them, taking precautions to prevent aspiration; or as a substitute, the nurse can achieve mechanical friction by wiping the teeth with a gauze pad. Bacteriocidal mouthwash does reduce plaque-causing bacteria; however, it is not as effective as application of mechanical friction. Water-soluble gel may be applied to lubricate dry lips, but it is not part of oral care.

A patient has been brought to the ED after suffering genitourinary trauma in an assault. Initial assessment reveals that the patients bladder is distended. What is the nurses most appropriate action? A) Withhold fluids from the patient. B) Perform intermittent urinary catheterization. C) Insert a narrow-gauge indwelling urinary catheter. D) Await orders following the urologists assessment.

D Urethral catheter insertion when a possible urethral injury is present is contraindicated; a urology consultation and further evaluation of the urethra are required. The nurse would withhold fluids, but urologic assessment is the priority.

A patient with cancer of the bladder has just returned to the unit from the PACU after surgery to create an ileal conduit. The nurse is monitoring the patients urine output hourly and notifies the physician when the hourly output is less than what? A) 30 mL B) 50 mL C) 100 mL D) 125 mL

A A urine output below 30 mL/hr may indicate dehydration or an obstruction in the ileal conduit, with possible backflow or leakage from the ureteroileal anastomosis.

The nurse is caring for a patient who has just returned to the post-surgical unit following renal surgery. When assessing the patients output from surgical drains, the nurse should assess what parameters? Select all that apply. A) Quantity of output B) Color of the output C) Visible characteristics of the output D) Odor of the output E) pH of the output

A, B, C Urine output and drainage from tubes inserted during surgery are monitored for amount, color, and type or characteristics. Odor and pH are not normally assessed.

When teaching a patient about testing to diagnose metabolic syndrome, which topic would the nurse include? A) Blood glucose test B) Cardiac enzyme tests C) Postural blood pressures D) Resting electrocardiogram

A A fasting blood glucose test >100 mg/dL is one of the diagnostic criteria for metabolic syndrome. The other tests are not used to diagnose metabolic syndrome although they may be used to check for cardiovascular complications of the disorder.

Renal failure can have prerenal, renal, or postrenal causes. A patient with acute kidney injury is being assessed to determine where, physiologically, the cause is. If the cause is found to be prerenal, which condition most likely caused it? A) Heart failure B) Glomerulonephritis C) Ureterolithiasis D) Aminoglycoside toxicity

A By causing inadequate renal perfusion, heart failure can lead to prerenal failure. Glomerulonephritis and aminoglycoside toxicity are renal causes, and ureterolithiasis is a postrenal cause.

The nurse is caring for a patient recently diagnosed with renal calculi. The nurse should instruct the patient to increase fluid intake to a level where the patient produces at least how much urine each day? A) 1,250 mL B) 2,000 mL C) 2,750 mL D) 3,500 mL

B Unless contraindicated by renal failure or hydronephrosis, patients with renal stones should drink at least eight 8-ounce glasses of water daily or have IV fluids prescribed to keep the urine dilute. A urine output exceeding 2 L a day is advisable.

The nurse is caring for a patient who has undergone creation of a urinary diversion. Forty-eight hours postoperatively, the nurses assessment reveals that the stoma is a dark purplish color. What is the nurses most appropriate response? A) Document the presence of a healthy stoma. B) Assess the patient for further signs and symptoms of infection. C) Inform the primary care provider that the vascular supply may be compromised. D) Liaise with the wound-ostomy-continence (WOC) nurse because the ostomy appliance around the stoma may be too loose.

C A healthy stoma is pink or red. A change from this normal color to a dark purplish color suggests that the vascular supply may be compromised. A loose ostomy appliance and infections do not cause a dark purplish stoma.

After successfully losing 1 lb weekly for several months, a patient at the clinic has not lost any weight for the last month. The nurse should first A) review the diet and exercise guidelines with the patient. B) instruct the patient to weigh and record weights weekly. C) ask the patient whether there have been any changes in exercise or diet patterns. D) discuss the possibility that the patient has reached a temporary weight loss plateau.

c The initial nursing action should be assessment of any reason for the change in weight loss. The other actions may be needed, but further assessment is required before any interventions are planned or implemented.

A patient has a recent diagnosis of chronic pancreatitis and is undergoing diagnostic testing to determine pancreatic islet cell function. The nurse should anticipate what diagnostic test? A) Glucose tolerance test B) ERCP C) Pancreatic biopsy D) Abdominal ultrasonography

A A glucose tolerance test evaluates pancreatic islet cell function and provides necessary information for making decisions about surgical resection of the pancreas. This specific clinical information is not provided by ERCP, biopsy, or ultrasound.

A nurse is providing care for a patient who has a diagnosis of irritable bowel syndrome (IBS). When planning this patients care, the nurse should collaborate with the patient and prioritize what goal? A) Patient will accurately identify foods that trigger symptoms. B) Patient will demonstrate appropriate care of his ileostomy. C) Patient will demonstrate appropriate use of standard infection control precautions. D) Patient will adhere to recommended guidelines for mobility and activity.

A A major focus of nursing care for the patient with IBS is to identify factors that exacerbate symptoms. Surgery is not used to treat this health problem and infection control is not a concern that is specific to this diagnosis. Establishing causation likely is more important to the patient than managing physical activity.

A patient has been diagnosed with achalasia based on his history and diagnostic imaging results. The nurse should identify what risk diagnosis when planning the patients care? A) Risk for Aspiration Related to Inhalation of Gastric Contents B) Risk for Imbalanced Nutrition: Less than Body Requirements Related to Impaired Absorption C) Risk for Decreased Cardiac Output Related to Vasovagal Response D) Risk for Impaired Verbal Communication Related to Oral Trauma

A Achalasia can result in the aspiration of gastric contents. It is not normally an acute risk to the patients nutritional status and does not affect cardiac output or communication.

A nurse is caring for a patient admitted with symptoms of an anorectal infection; cultures indicate that the patient has a viral infection. The nurse should anticipate the administration of what drug? A) Acyclovir (Zovirax) B) Doxycycline (Vibramycin) C) Penicillin (penicillin D) Metronidazole (Flagyl)

A Acyclovir (Zovirax) is often given to patients with viral anorectal infections. Doxycycline (Vibramycin) and penicillin (penicillin G) are drugs of choice for bacterial infections. Metronidazole (Flagyl) is used for other infections with a bacterial etiology; it is ineffective against viruses.

A patient is admitted to the ED after being involved in a motor vehicle accident. The patient has multiple injuries. After establishing an airway and adequate ventilation, the ED team should prioritize what aspect of care? A) Control the patients hemorrhage. B) Assess for cognitive effects of the injury. C) Splint the patients fractures. D) Assess the patients neurologic status.

A After establishing airway and ventilation, the team should evaluate and restore cardiac output by controlling hemorrhage. This must precede neurologic assessments and treatment of skeletal injuries.

The nurse is caring for a patient postoperative day 4 following a kidney transplant. When assessing for potential signs and symptoms of rejection, what assessment should the nurse prioritize? A) Assessment of the quantity of the patients urine output B) Assessment of the patients incision C) Assessment of the patients abdominal girth D) Assessment for flank or abdominal pain

A After kidney transplantation, the nurse should perform all of the listed assessments. However, oliguria is considered to be more suggestive of rejection than changes to the patients abdomen or incision.

The nurse is caring for a patient who is to begin receiving external radiation for a malignant tumor of the neck. While providing patient education, what potential adverse effects should the nurse discuss with the patient? A) Impaired nutritional status B) Cognitive changes C) Diarrhea D) Alopecia

A Alterations in oral mucosa, change and loss of taste, pain, and dysphasia often occur as a result of radiotherapy to the head and neck. The patient is at an increased risk of impaired nutritional status. Radiotherapy does not cause cognitive changes. Diarrhea is not a likely concern for this patient. Radiation only results in alopecia when targeted at the whole brain; radiation of other parts of the body does not lead to hair loss.

A patient has been scheduled for an ultrasound of the gallbladder the following morning. What should the nurse do in preparation for this diagnostic study? A) Have the patient refrain from food and fluids after midnight. B) Administer the contrast agent orally 10 to 12 hours before the study. C) Administer the radioactive agent intravenously the evening before the study. D) Encourage the intake of 64 ounces of water 8 hours before the study.

A An ultrasound of the gallbladder is most accurate if the patient fasts overnight, so that the gallbladder is distended. Contrast and radioactive agents are not used when performing ultrasonography of the gallbladder, as an ultrasound is based on reflected sound waves.

A 50-year-old man diagnosed with leukemia will begin chemotherapy. What would the nurse do to combat the most common adverse effects of chemotherapy? A) Administer an antiemetic. B) Administer an antimetabolite. C) Administer a tumor antibiotic. D) Administer an anticoagulant.

A Antiemetics are used to treat nausea and vomiting, the most common adverse effects of chemotherapy. Antihistamines and certain steroids are also used to treat nausea and vomiting. Antimetabolites and tumor antibiotics are classes of chemotherapeutic medications. Anticoagulants slow blood clotting time, thereby helping to prevent thrombi and emboli.

The nurse has identified the nursing diagnosis of risk for infection in a patient who undergoes peritoneal dialysis. What nursing action best addresses this risk? A) Maintain aseptic technique when administering dialysate. B) Wash the skin surrounding the catheter site with soap and water prior to each exchange. C) Add antibiotics to the dialysate as ordered. D) Administer prophylactic antibiotics by mouth or IV as ordered.

A Aseptic technique is used to prevent peritonitis and other infectious complications of peritoneal dialysis. It is not necessary to cleanse the skin with soap and water prior to each exchange. Antibiotics may be added to dialysate to treat infection, but they are not used to prevent infection.

During a health education session, a participant has asked about the hepatitis E virus. What prevention measure should the nurse recommend for preventing infection with this virus? A) Following proper hand-washing techniques B) Avoiding chemicals that are toxic to the liver C) Wearing a condom during sexual contact D) Limiting alcohol intake

A Avoiding contact with the hepatitis E virus through good hygiene, including hand-washing, is the major method of prevention. Hepatitis E is transmitted by the fecaloral route, principally through contaminated water in areas with poor sanitation. Consequently, none of the other listed preventative measures is indicated.

The nurse has tested the pH of urine from a patients newly created ileal conduit and obtained a result of 6.8. What is the nurses best response to this assessment finding? A) Obtain an order to increase the patients dose of ascorbic acid. B) Administer IV sodium bicarbonate as ordered. C) Encourage the patient to drink at least 500 mL of water and retest in 3 hours. D) Irrigate the ileal conduit with a dilute citric acid solution as ordered.

A Because severe alkaline encrustation can accumulate rapidly around the stoma, the urine pH is kept below 6.5 by administration of ascorbic acid by mouth. An increased pH may suggest a need to increase ascorbic acid dosing. This is not treated by administering bicarbonate or citric acid, nor by increasing fluid intake.

A nurse is caring for a patient with cancer of the liver whose condition has required the insertion of a percutaneous biliary drainage system. The nurses most recent assessment reveals the presence of dark green fluid in the collection container. What is the nurses best response to this assessment finding? A) Document the presence of normal bile output. B) Irrigate the drainage system with normal saline as ordered. C) Aspirate a sample of the drainage for culture. D) Promptly report this assessment finding to the primary care provider.

A Bile is usually a dark green or brownish-yellow color, so this would constitute an expected assessment finding, with no other action necessary.

The nurse is caring for a patient who has returned to the postsurgical suite after post-anesthetic recovery from a nephrectomy. The nurses most recent hourly assessment reveals a significant drop in level of consciousness and BP as well as scant urine output over the past hour. What is the nurses best response? A) Assess the patient for signs of bleeding and inform the physician. B) Monitor the patients vital signs every 15 minutes for the next hour. C) Reposition the patient and reassess vital signs. D) Palpate the patients flanks for pain and inform the physician.

A Bleeding may be suspected when the patient experiences fatigue and when urine output is less than 30 mL/h. The physician must be made aware of this finding promptly. Palpating the patients flanks would cause intense pain that is of no benefit to assessment.

A 35-year-old male patient presents at the emergency department with symptoms of a small bowel obstruction. In collaboration with the primary care provider, what intervention should the nurse prioritize? A) Insertion of a nasogastric tube B) Insertion of a central venous catheter C) Administration of a mineral oil enema D) Administration of a glycerin suppository and an oral laxative

A Decompression of the bowel through a nasogastric tube is necessary for all patients with small bowel obstruction. Peripheral IV access is normally sufficient. Enemas, suppositories, and laxatives are not indicated if an obstruction is present.

The school nurse is planning a health fair for a group of fifth graders and dental health is one topic that the nurse plans to address. What would be most likely to increase the risk of tooth decay? A) Organic fruit juice B) Roasted nuts C) Red meat that is high in fat D) Cheddar cheese

A Dental caries may be prevented by decreasing the amount of sugar and starch in the diet. Patients who snack should be encouraged to choose less cariogenic alternatives, such as fruits, vegetables, nuts, cheeses, or plain yogurt. Fruit juice is high in sugar, regardless of whether it is organic.

While a patient is receiving IV doxorubicin hydrochloride for the treatment of cancer, the nurse observes swelling and pain at the IV site. The nurse should prioritize what action? A) Stopping the administration of the drug immediately B) Notifying the patients physician C) Continuing the infusion but decreasing the rate D) Applying a warm compress to the infusion site

A Doxorubicin hydrochloride is a chemotherapeutic vesicant that can cause severe tissue damage. The nurse should stop the administration of the drug immediately and then notify the patients physician. Ice can be applied to the site once the drug therapy has stopped.

An adult patient has been admitted to the medical unit for the treatment of acute pancreatitis. What nursing action should be included in this patients plan of care? A) Measure the patients abdominal girth daily. B) Limit the use of opioid analgesics. C) Monitor the patient for signs of dysphagia. D) Encourage activity as tolerated.

A Due to the risk of ascites, the nurse should monitor the patients abdominal girth. There is no specific need to avoid the use of opioids or to monitor for dysphagia, and activity is usually limited.

The nurse is working with a patient who has been experiencing episodes of urinary retention. What assessment finding would suggest that the patient is experiencing retention? A) The patients suprapubic region is dull on percussion. B) The patient is uncharacteristically drowsy. C) The patient claims to void large amounts of urine 2 to 3 times daily. D) The patient takes a beta adrenergic blocker for the treatment of hypertension.

A Dullness on percussion of the suprapubic region is suggestive of urinary retention. Patients retaining urine are typically restless, not drowsy. A patient experiencing retention usually voids frequent, small amounts of urine and the use of beta-blockers is unrelated to urinary retention.

An 83-year-old patient is brought in by ambulance from a long-term care facility. The patients symptoms are weakness, lethargy, incontinence, and a change in mental status. The nurse knows that emergencies in older adults may be more difficult to manage. Why would this be true? A) Older adults may have an altered response to treatment. B) Older adults are often reluctant to adhere to prescribed treatment. C) Older adults have difficulty giving a health history. D) Older adults often stigmatize their peers who use the ED.

A Emergencies in this age group may be more difficult to manage because elderly patients may have an atypical presentation, an altered response to treatment, a greater risk of developing complications, or a combination of these factors. The elderly patient may perceive the emergency as a crisis signaling the end of an independent lifestyle or even resulting in death. Stigmatization and nonadherence to treatment are not commonly noted. Older adults do not necessarily have difficulty giving a health history.

The nurse is coaching a community group for individuals who are overweight. Which participant behavior is an example of the best exercise plan for weight loss? A) Walking for 40 minutes 6 or 7 days/week B) Lifting weights with friends 3 times/week C) Playing soccer for an hour on the weekend D) Running for 10 to 15 minutes 3 times/week

A Exercise should be done daily for 30 minutes to an hour. Exercising in highly aerobic activities for short bursts or only once a week is not helpful and may be dangerous in an individual who has not been exercising. Running may be appropriate, but a patient should start with an exercise that is less stressful and can be done for a longer period. Weight lifting is not as helpful as aerobic exercise in weight loss.

A nurse is caring for a patient who is postoperative from a neck dissection. What would be the most appropriate nursing action to enhance the patients appetite? A) Encourage the family to bring in the patients favored foods. B) Limit visitors at mealtimes so that the patient is not distracted. C) Avoid offering food unless the patient initiates. D) Provide thorough oral care immediately after the patient eats.

A Family involvement and home-cooked favorite foods may help the patient to eat. Having visitors at mealtimes may make eating more pleasant and increase the patients appetite. The nurse should not place the complete onus for initiating meals on the patient. Oral care after meals is necessary, but does not influence appetite.

The clinic nurse is caring for a 42-year-old male oncology patient. He complains of extreme fatigue and weakness after his first week of radiation therapy. Which response by the nurse would best reassure this patient? A) These symptoms usually result from radiation therapy; however, we will continue to monitor your laboratory and x-ray studies. B) These symptoms are part of your disease and are an unfortunately inevitable part of living with cancer. C) Try not to be concerned about these symptoms. Every patient feels this way after having radiation therapy. D) Even though it is uncomfortable, this is a good sign. It means that only the cancer cells are dying.

A Fatigue and weakness result from radiation treatment and usually do not represent deterioration or disease progression. The symptoms associated with radiation therapy usually decrease after therapy ends. The symptoms may concern the patient and should not be belittled. Radiation destroys both cancerous and normal cells.

A patient with kidney stones is scheduled for extracorporeal shock wave lithotripsy (ESWL). What should the nurse include in the patients post-procedure care? A) Strain the patients urine following the procedure. B) Administer a bolus of 500 mL normal saline following the procedure. C) Monitor the patient for fluid overload following the procedure. D) Insert a urinary catheter for 24 to 48 hours after the procedure.

A Following ESWL, the nurse should strain the patients urine for gravel or sand. There is no need to administer an IV bolus after the procedure and there is not a heightened risk of fluid overload. Catheter insertion is not normally indicated following ESWL.

A backcountry skier has been airlifted to the ED after becoming lost and developing hypothermia and frostbite. How should the nurse best manage the patients frostbite? A) Immerse affected extremities in water slightly above normal body temperature. B) Immerse the patients frostbitten extremities in the warmest water the patient can tolerate. C) Gently massage the patients frozen extremities in between water baths. D) Perform passive range-of-motion exercises of the affected extremities to promote circulation.

A Frozen extremities are usually placed in a 37C to 40C (98.6F to 104F) circulating bath for 30- to 40- minute spans. To avoid further mechanical injury, the body part is not handled. Massage is contraindicated.

A group of nurses have attended an inservice on the prevention of occupationally acquired diseases that affect healthcare providers. What action has the greatest potential to reduce a nurses risk of acquiring hepatitis C in the workplace? A) Disposing of sharps appropriately and not recapping needles B) Performing meticulous hand hygiene at the appropriate moments in care C) Adhering to the recommended schedule of immunizations D) Wearing an N95 mask when providing care for patients on airborne precautions

A HCV is bloodborne. Consequently, prevention of needlestick injuries is paramount. Hand hygiene, immunizations and appropriate use of masks are important aspects of overall infection control, but these actions do not directly mitigate the risk of HCV.

A football player is thought to have sustained an injury to his kidneys from being tackled from behind. The ER nurse caring for the patient reviews the initial orders written by the physician and notes that an order to collect all voided urine and send it to the laboratory for analysis. The nurse understands that this nursing intervention is important for what reason? A) Hematuria is the most common manifestation of renal trauma and blood losses may be microscopic, so laboratory analysis is essential. B) Intake and output calculations are essential and the laboratory will calculate the precise urine output produced by this patient. C) A creatinine clearance study may be ordered at a later time and the laboratory will hold all urine until it is determined if the test will be necessary. D) There is great concern about electrolyte imbalances and the laboratory will monitor the urine for changes in potassium and sodium concentrations.

A Hematuria is the most common manifestation of renal trauma; its presence after trauma suggests renal injury. Hematuria may not occur, or it may be detectable only on microscopic examination. All urine should be saved and sent to the laboratory for analysis to detect RBCs and to evaluate the course of bleeding. Measuring intake and output is not a function of the laboratory. The laboratory does not save urine to test creatinine clearance at a later time. The laboratory does not monitor the urine for sodium or potassium concentrations.

A 45-year-old man with diabetic nephropathy has ESKD and is starting dialysis. What should the nurse teach the patient about hemodialysis? A) Hemodialysis is a treatment option that is usually required three times a week. B) Hemodialysis is a program that will require you to commit to daily treatment. C) This will require you to have surgery and a catheter will need to be inserted into your abdomen. D) Hemodialysis is a treatment that is used for a few months until your kidney heals and starts to produce urine again.

A Hemodialysis is the most commonly used method of dialysis. Patients receiving hemodialysis must undergo treatment for the rest of their lives or until they undergo successful kidney transplantation. Treatments usually occur three times a week for at least 3 to 4 hours per treatment.

A patient is admitted to the ED with suspected alcohol intoxication. The ED nurse is aware of the need to assess for conditions that can mimic acute alcohol intoxication. In light of this need, the nurse should perform what action? A) Check the patients blood glucose level. B) Assess for a documented history of major depression. C) Determine whether the patient has ingested a corrosive substance. D) Arrange for assessment of serum potassium levels.

A Hypoglycemia can mimic alcohol intoxication and should be assessed in a patient suspected of alcohol intoxication. Potassium imbalances, depression, and poison ingestion are not noted to mimic the characteristic signs and symptoms of alcohol intoxication.

A patients abdominal ultrasound indicates cholelithiasis. When the nurse is reviewing the patients laboratory studies, what finding is most closely associated with this diagnosis? A) Increased bilirubin B) Decreased serum cholesterol C) Increased blood urea nitrogen (BUN) D) Decreased serum alkaline phosphatase level

A If the flow of blood is impeded, bilirubin, a pigment derived from the breakdown of red blood cells, does not enter the intestines. As a result, bilirubin levels in the blood increase. Cholesterol, BUN, and alkaline phosphatase levels are not typically affected.

A patients neck dissection surgery resulted in damage to the patients superior laryngeal nerve. What area of assessment should the nurse consequently prioritize? A) The patients swallowing ability B) The patients ability to speak C) The patients management of secretions D) The patients airway patency

A If the superior laryngeal nerve is damaged, the patient may have difficulty swallowing liquids and food because of the partial lack of sensation of the glottis. Damage to this particular nerve does not inhibit speech and only affects management of secretions and airway patency indirectly.

A patient is undergoing diagnostic testing for a suspected urinary obstruction. The nurse should know that incomplete emptying of the bladder due to bladder outlet obstruction can cause what? A) Hydronephrosis B) Nephritic syndrome C) Pylonephritis D) Nephrotoxicity

A If voiding dysfunction goes undetected and untreated, the upper urinary system may become compromised. Chronic incomplete bladder emptying from poor detrusor pressure results in recurrent bladder infection. Incomplete bladder emptying due to bladder outlet obstruction, causing high-pressure detrusor contractions, can result in hydronephrosis from the high detrusor pressure that radiates up the ureters to the renal pelvis. This problem does not normally cause nephritic syndrome or pyelonephritis. Nephrotoxicity results from chemical causes.

A patient with a liver mass is undergoing a percutaneous liver biopsy. What action should the nurse perform when assisting with this procedure? A) Position the patient on the right side with a pillow under the costal margin after the procedure. B) Administer 1 unit of albumin 90 minutes before the procedure as ordered. C) Administer at least 1 unit of packed red blood cells as ordered the day before the scheduled procedure. D) Confirm that the patients electrolyte levels have been assessed prior to the procedure.

A Immediately after a percutaneous liver biopsy, assist the patient to turn onto the right side and place a pillow under the costal margin. Prior administration of albumin or PRBCs is unnecessary. Coagulation tests should be performed, but electrolyte analysis is not necessary.

A patient has received treatment for oral cancer. The combination of medications and radiotherapy has resulted in leukopenia. Which of the following is an appropriate response to this change in health status? A) Ensure that none of the patients visitors has an infection. B) Arrange for a diet that is high in protein and low in fat. C) Administer colony stimulating factors (CSFs) as ordered. D) Prepare to administer chemotherapeutics as ordered.

A Leukopenia reduces defense mechanisms, increasing the risk of infections. Visitors who might transmit microorganisms are prohibited if the patients immunologic system is depressed. Changes in diet, CSFs, and the use of chemotherapy do not resolve leukopenia.

A patient with gastroesophageal reflux disease (GERD) has a diagnosis of Barretts esophagus with minor cell changes. Which of the following principles should be integrated into the patients subsequent care? A) The patient will require an upper endoscopy every 6 months to detect malignant changes. B) Liver enzymes must be checked regularly, as H2 receptor antagonists may cause hepatic damage. C) Small amounts of blood are likely to be present in the stools and are not cause for concern. D) Antacids may be discontinued when symptoms of heartburn subside.

A In the patient with Barretts esophagus, the cells lining the lower esophagus have undergone change and are no longer squamous cells. The altered cells are considered precancerous and are a precursor to esophageal cancer. In order to facilitate early detection of malignant cells, an upper endoscopy is recommended every 6 months. H2 receptor antagonists are commonly prescribed for patients with GERD; however, monitoring of liver enzymes is not routine. Stools that contain evidence of frank bleeding or that are tarry are not expected and should be reported immediately. When antacids are prescribed for patients with GERD, they should be taken as ordered whether or not the patient is symptomatic.

A nurse on a solid organ transplant unit is planning the care of a patient who will soon be admitted upon immediate recovery following liver transplantation. What aspect of nursing care is the nurses priority? A) Implementation of infection-control measures B) Close monitoring of skin integrity and color C) Frequent assessment of the patients psychosocial status D) Administration of antiretroviral medications

A Infection control is paramount following liver transplantation. This is a priority over skin integrity and psychosocial status, even though these are valid areas of assessment and intervention. Antiretrovirals are not indicated.

A patient is being discharged after a liver transplant and the nurse is performing discharge education. When planning this patients continuing care, the nurse should prioritize which of the following risk diagnoses? A) Risk for Infection Related to Immunosuppressant Use B) Risk for Injury Related to Decreased Hemostasis C) Risk for Unstable Blood Glucose Related to Impaired Gluconeogenesis D) Risk for Contamination Related to Accumulation of Ammonia

A Infection is the leading cause of death after liver transplantation. Pulmonary and fungal infections are common; susceptibility to infection is increased by the immunosuppressive therapy that is needed to prevent rejection. This risk exceeds the threats of injury and unstable blood glucose. The diagnosis of Risk for Contamination relates to environmental toxin exposure.

The nurse will be teaching self-management to patients after gastric bypass surgery. Which information will the nurse plan to include? A) Drink fluids between meals but not with meals. B) Choose high-fat foods for at least 30% of intake. C) Developing flabby skin can be prevented by exercise. D) Choose foods high in fiber to promote bowel function.

A Intake of fluids with meals tends to cause dumping syndrome and diarrhea. Food choices should be low in fat and fiber. Exercise does not prevent the development of flabby skin.

A patient is brought to the ED by two police officers. The patient was found unconscious on the sidewalk, with his face and hands covered in blood. At present, the patient is verbally abusive and is fighting the staff in the ED, but appears medically stable. The decision is made to place the patient in restraints. What action should the nurse perform when the patient is restrained? A) Frequently assess the patients skin integrity. B) Inform the patient that he is likely to be charged with assault. C) Avoid interacting with the patient until the restraints are removed. D) Take the opportunity to perform a full physical assessment.

A It is important to assess skin integrity when physical restraints are used. Criminal charges are not the responsibility of the nurse and the nurse should still interact with the patient. A full physical assessment, however, would likely be delayed until the patient is not combative.

A patient has developed hepatic encephalopathy secondary to cirrhosis and is receiving care on the medical unit. The patients current medication regimen includes lactulose (Cephulac) four times daily. What desired outcome should the nurse relate to this pharmacologic intervention? A) Two to 3 soft bowel movements daily B) Significant increase in appetite and food intake C) Absence of nausea and vomiting D) Absence of blood or mucus in stool

A Lactulose (Cephulac) is administered to reduce serum ammonia levels. Two or three soft stools per day are desirable; this indicates that lactulose is performing as intended. Lactulose does not address the patients appetite, symptoms of nausea and vomiting, or the development of blood and mucus in the stool.

A patient with GERD has undergone diagnostic testing and it has been determined that increasing the pace of gastric emptying may help alleviate symptoms. The nurse should anticipate that the patient may be prescribed what drug? A) Metoclopramide (Reglan) B) Omeprazole (Prilosec) C) Lansoprazole (Prevacid) D) Famotidine (Pepcid)

A Metoclopramide (Reglan) is useful in promoting gastric motility. Omeprazole and lansoprozole are proton pump inhibitors that reduce gastric acid secretion. Famotidine (Pepcid) is an H2receptor antagonist, which has a similar effect.

A patient with ongoing back pain, nausea, and abdominal bloating has been diagnosed with cholecystitis secondary to gallstones. The nurse should anticipate that the patient will undergo what intervention? A) Laparoscopic cholecystectomy B) Methyl tertiary butyl ether (MTBE) infusion C) Intracorporeal lithotripsy D) Extracorporeal shock wave therapy (ESWL)

A Most of the nonsurgical approaches, including lithotripsy and dissolution of gallstones, provide only temporary solutions to gallstone problems and are infrequently used in the United States. Cholecystectomy is the preferred treatment.

An elderly patient comes into the emergency department complaining of an earache. The patient and has an oral temperature of 100.2F and otoscopic assessment of the ear reveals a pearly gray tympanic membrane with no evidence of discharge or inflammation. Which action should the triage nurse take next? A) Palpate the patients parotid glands to detect swelling and tenderness. B) Assess the temporomandibular joint for evidence of a malocclusion. C) Test the integrity of cranial nerve XII by asking the patient to protrude the tongue. D) Inspect the patients gums for bleeding and hyperpigmentation.

A Older adults and debilitated patients of any age who are dehydrated or taking medications that reduce saliva production are at risk for parotitis. Symptoms include fever and tenderness, as well as swelling of the parotid glands. Pain radiates to the ear. Pain associated with malocclusion of the temporomandibular joint may also radiate to the ears; however, a temperature elevation would not be associated with malocclusion. The 12th cranial nerve is not associated with the auditory system. Bleeding and hyperpigmented gums may be caused by pyorrhea or gingivitis. These conditions do not cause earache; fever would not be present unless the teeth were abscessed.

A patient has been brought to the ED with multiple trauma after a motor vehicle accident. After immediate threats to life have been addressed, the nurse and trauma team should take what action? A) Perform a rapid physical assessment. B) Initiate health education. C) Perform diagnostic imaging. D) Establish the circumstances of the accident.

A Once immediate threats to life have been corrected, a rapid physical examination is done to identify injuries and priorities of treatment. Health education is initiated later in the care process and diagnostic imaging would take place after a rapid physical assessment. It is not the care teams responsibility to determine the circumstances of the accident.

A patient with a cholelithiasis has been scheduled for a laparoscopic cholecystectomy. Why is laparoscopic cholecystectomy preferred by surgeons over an open procedure? A) Laparoscopic cholecystectomy poses fewer surgical risks than an open procedure. B) Laparoscopic cholecystectomy can be performed in a clinic setting, while an open procedure requires an OR. C) A laparoscopic approach allows for the removal of the entire gallbladder. D) A laparoscopic approach can be performed under conscious sedation.

A Open surgery has largely been replaced by laparoscopic cholecystectomy (removal of the gallbladder through a small incision through the umbilicus). As a result, surgical risks have decreased, along with the length of hospital stay and the long recovery period required after standard surgical cholecystectomy. Both approaches allow for removal of the entire gallbladder and must be performed under general anesthetic in an operating theater.

A nurse who provides care in an ambulatory clinic integrates basic cancer screening into admission assessments. What patient most likely faces the highest immediate risk of oral cancer? A) A 65-year-old man with alcoholism who smokes B) A 45-year-old woman who has type 1 diabetes and who wears dentures C) A 32-year-old man who is obese and uses smokeless tobacco D) A 57-year-old man with GERD and dental caries

A Oral cancers are often associated with the use of alcohol and tobacco, which when used together have a synergistic carcinogenic effect. Most cases of oral cancers occur in people over the age of 60 and a disproportionate number of cases occur in men. Diabetes, dentures, dental caries, and GERD are not risk factors for oral cancer.

A patient has come to the clinic complaining of pain just above her umbilicus. When assessing the patient, the nurse notes Sister Mary Josephs nodules. The nurse should refer the patient to the primary care provider to be assessed for what health problem? A) A GI malignancy B) Dumping syndrome C) Peptic ulcer disease D) Esophageal/gastric obstruction

A Palpable nodules around the umbilicus, called Sister Mary Josephs nodules, are a sign of a GI malignancy, usually a gastric cancer. This would not be a sign of dumping syndrome, peptic ulcer disease, or esophageal/gastric obstruction.

The nurse who is leading a wellness workshop has been asked about actions to reduce the risk of bladder cancer. What health promotion action most directly addresses a major risk factor for bladder cancer? A) Smoking cessation B) Reduction of alcohol intake C) Maintenance of a diet high in vitamins and nutrients D) Vitamin D supplementation

A People who smoke develop bladder cancer twice as often as those who do not smoke. High alcohol intake and low vitamin intake are not noted to contribute to bladder cancer.

A patient with ESKD receives continuous ambulatory peritoneal dialysis. The nurse observes that the dialysate drainage fluid is cloudy. What is the nurses most appropriate action? A) Inform the physician and assess the patient for signs of infection. B) Flush the peritoneal catheter with normal saline. C) Remove the catheter promptly and have the catheter tip cultured. D) Administer a bolus of IV normal saline as ordered.

A Peritonitis is the most common and serious complication of peritoneal dialysis. The first sign of peritonitis is cloudy dialysate drainage fluid, so prompt reporting to the primary care provider and rapid assessment for other signs of infection are warranted. Administration of an IV bolus is not necessary or appropriate and the physician would determine whether removal of the catheter is required. Flushing the catheter does not address the risk for infection.

A teenage patient with a pilonidal cyst has been brought for care by her mother. The nurse who is contributing to the patients care knows that treatment will be chosen based on what risk? A) Risk for infection B) Risk for bowel incontinence C) Risk for constipation D) Risk for impaired tissue perfusion

A Pilonidal cysts frequently develop into an abscess, necessitating surgical repair. These cysts do not contribute to bowel incontinence, constipation, or impaired tissue perfusion.

A nurse in the postanesthesia care unit admits a patient following resection of a gastric tumor. Following immediate recovery, the patient should be placed in which position to facilitate patient comfort and gastric emptying? A) Fowlers B) Supine C) Left lateral D) Left Sims

A Positioning the patient in a Fowlers position postoperatively promotes comfort and facilitates emptying of the stomach following gastric surgery. Any position that involves lying down delays stomach emptying and is not recommended for this type of patient. Supine positioning and the left lateral (left Sims) position do not achieve this goal.

The nurse on a urology unit is working with a patient who has been diagnosed with oxalate renal calculi. When planning this patients health education, what nutritional guidelines should the nurse provide? A) Restrict protein intake as ordered. B) Increase intake of potassium-rich foods. C) Follow a low-calcium diet. D) Encourage intake of food containing oxalates.

A Protein is restricted to 60 g/d, while sodium is restricted to 3 to 4 g/d. Low-calcium diets are generally not recommended except for true absorptive hypercalciuria. The patient should avoid intake of oxalate- containing foods and there is no need to increase potassium intake.

A 16-year-old female patient experiences alopecia resulting from chemotherapy, prompting the nursing diagnoses of disturbed body image and situational low self-esteem. What action by the patient would best indicate that she is meeting the goal of improved body image and self-esteem? A) The patient requests that her family bring her makeup and wig. B) The patient begins to discuss the future with her family. C) The patient reports less disruption from pain and discomfort. D) The patient cries openly when discussing her disease.

A Requesting her wig and makeup indicates that the patient with alopecia is becoming interested in looking her best and that her body image and self-esteem may be improving. The other options may indicate that other nursing goals are being met, but they do not necessarily indicate improved body image and self-esteem.

A nurse is presenting an educational event to a local community group. When speaking about colorectal cancer, what risk factor should the nurse cite? A) High levels of alcohol consumption B) History of bowel obstruction C) History of diverticulitis D) Longstanding psychosocial stress

A Risk factors include high alcohol intake; cigarette smoking; and high-fact, high-protein, low-fiber diet. Diverticulitis, obstruction, and stress are not noted as risk factors for colorectal cancer.

A 37-year-old male patient presents at the emergency department (ED) complaining of nausea and vomiting and severe abdominal pain. The patients abdomen is rigid, and there is bruising to the patients flank. The patients wife states that he was on a drinking binge for the past 2 days. The ED nurse should assist in assessing the patient for what health problem? A) Severe pancreatitis with possible peritonitis B) Acute cholecystitis C) Chronic pancreatitis D) Acute appendicitis with possible perforation

A Severe abdominal pain is the major symptom of pancreatitis that causes the patient to seek medical care. Pain in pancreatitis is accompanied by nausea and vomiting that does not relieve the pain or nausea. Abdominal guarding is present and a rigid or board-like abdomen may be a sign of peritonitis. Ecchymosis (bruising) to the flank or around the umbilicus may indicate severe peritonitis. Pain generally occurs 24 to 48 hours after a heavy meal or alcohol ingestion. The link with alcohol intake makes pancreatitis a more likely possibility than appendicitis or cholecystitis.

Which patient should the nurse prioritize as needing emergent treatment, assuming no other injuries are present except the ones outlined below? A) A patient with a blunt chest trauma with some difficulty breathing B) A patient with a sore neck who was immobilized in the field on a backboard with a cervical collar C) A patient with a possible fractured tibia with adequate pedal pulses D) A patient with an acute onset of confusion

A The patient with blunt chest trauma possibly has a compromised airway. Establishment and maintenance of a patent airway and adequate ventilation is prioritized over other health problems, including skeletal injuries and changes in cognition.

A triage nurse in the emergency department is assessing a patient who presented with complaints of general malaise. Assessment reveals the presence of jaundice and increased abdominal girth. What assessment question best addresses the possible etiology of this patients presentation? A) How many alcoholic drinks do you typically consume in a week? B) To the best of your knowledge, are your immunizations up to date? C) Have you ever worked in an occupation where you might have been exposed to toxins? D) Has anyone in your family ever experienced symptoms similar to yours?

A Signs or symptoms of hepatic dysfunction indicate a need to assess for alcohol use. Immunization status, occupational risks, and family history are also relevant considerations, but alcohol use is a more common etiologic factor in liver disease.

A medical nurse who is caring for a patient being discharged home after a radical neck dissection has collaborated with the home health nurse to develop a plan of care for this patient. What is a priority psychosocial outcome for a patient who has had a radical neck dissection? A) Indicates acceptance of altered appearance and demonstrates positive self-image B) Freely expresses needs and concerns related to postoperative pain management C) Compensates effectively for alteration in ability to communicate related to dysarthria D) Demonstrates effective stress management techniques to promote muscle relaxation

A Since radical neck dissection involves removal of the sternocleidomastoid muscle, spinal accessory muscles, and cervical lymph nodes on one side of the neck, the patients appearance is visibly altered. The face generally appears asymmetric, with a visible neck depression; shoulder drop also occurs frequently. These changes have the potential to negatively affect self-concept and body image. Facilitating adaptation to these changes is a crucial component of nursing intervention. Patients who have had head and neck surgery generally report less pain as compared with other postoperative patients; however, the nurse must assess each individual patients level of pain and response to analgesics. Patients may experience transient hoarseness following a radical neck dissection; however, their ability to communicate is not permanently altered. Stress management is beneficial but would not be considered the priority in this clinical situation.

A 42-year-old woman comes to the clinic complaining of occasional urinary incontinence when she sneezes. The clinic nurse should recognize what type of incontinence? A) Stress incontinence B) Reflex incontinence C) Overflow incontinence D) Functional incontinence

A Stress incontinence is the involuntary loss of urine through an intact urethra as a result of sudden increase in intra-abdominal pressure. Reflex incontinence is loss of urine due to hyperreflexia or involuntary urethral relaxation in the absence of normal sensations usually associated with voiding. Overflow incontinence is an involuntary urine loss associated with overdistension of the bladder. Functional incontinence refers to those instances in which the function of the lower urinary tract is intact, but other factors (outside the urinary system) make it difficult or impossible for the patient to reach the toilet in time for voiding.

The home health nurse is performing a home visit for an oncology patient discharged 3 days ago after completing treatment for non-Hodgkin lymphoma. The nurses assessment should include examination for the signs and symptoms of what complication? A) Tumor lysis syndrome (TLS) B) Syndrome of inappropriate antiduretic hormone (SIADH) C) Disseminated intravascular coagulation (DIC) D) Hypercalcemia

A TLS is a potentially fatal complication that occurs spontaneously or more commonly following radiation, biotherapy, or chemotherapy-induced cell destruction of large or rapidly growing cancers such as leukemia, lymphoma, and small cell lung cancer. DIC, SIADH and hypercalcemia are less likely complications following this treatment and diagnosis.

A nurse is assisting with serving dinner trays on the unit. Upon receiving the dinner tray for a patient admitted with acute gallbladder inflammation, the nurse will question which of the following foods on the tray? A) Fried chicken B) Mashed potatoes C) Dinner roll D) Tapioca pudding

A The diet immediately after an episode of acute cholecystitis is initially limited to low-fat liquids. Cooked fruits, rice or tapioca, lean meats, mashed potatoes, bread, and coffee or tea may be added as tolerated. The patient should avoid fried foods such as fried chicken, as fatty foods may bring on an episode of cholecystitis.

The nurse is planning patient teaching for a patient with ESKD who is scheduled for the creation of a fistula. The nurse would include which of the following in teaching the patient about the fistula? A) A vein and an artery in your arm will be attached surgically. B) The arm should be immobilized for 4 to 6 days. C) One needle will be inserted into the fistula for each dialysis treatment. D) The fistula can be used 2 days after the surgery for dialysis treatment.

A The fistula joins an artery and a vein, either side-to-side or end-to-end. This access will need time, usually 2 to 3 months, to mature before it can be used. The patient is encouraged to perform exercises to increase the size of the affected vessels (e.g., squeezing a rubber ball for forearm fistulas). Two needles will be inserted into the fistula for each dialysis treatment.

The hospice nurse has just admitted a new patient to the program. What principle guides hospice care? A) Care addresses the needs of the patient as well as the needs of the family. B) Care is focused on the patient centrally and the family peripherally. C) The focus of all aspects of care is solely on the patient. D) The care team prioritizes the patients physical needs and the family is responsible for the patients emotional needs.

A The focus of hospice care is on the family as well as the patient. The family is not solely responsible for the patients emotional well-being

The ED nurse is planning the care of a patient who has been admitted following a sexual assault. The nurse knows that all of the nursing interventions are aimed at what goal? A) Encouraging the patient to gain a sense of control over his or her life B) Collecting sufficient evidence to secure a criminal conviction C) Helping the patient understand that this will not happen again D) Encouraging the patient to verbalize what happened during the assault

A The goals of management are to provide support, to reduce the patients emotional trauma, and to gather available evidence for possible legal proceedings. All of the interventions are aimed at encouraging the patient to gain a sense of control over his or her life. The patients well-being should be considered a priority over criminal proceedings. No health professional can guarantee the patients future safety and having the patient verbalize the event is not a priority.

The nurse is providing care for a patient whose inflammatory bowel disease has necessitated hospital treatment. Which of the following would most likely be included in the patients medication regimen? A) Anticholinergic medications 30 minutes before a meal B) Antiemetics on a PRN basis C) Vitamin B12 injections to prevent pernicious anemia D) Beta adrenergic blockers to reduce bowel motility

A The nurse administers anticholinergic medications 30 minutes before a meal as prescribed to decrease intestinal motility and administers analgesics as prescribed for pain. Antiemetics, vitamin B12 injections and beta blockers do not address the signs, symptoms, or etiology of inflammatory bowel disease.

A patient has received a diagnosis of gastric cancer and is awaiting a surgical date. During the preoperative period, the patient should adopt what dietary guidelines? A) Eat small, frequent meals with high calorie and vitamin content. B) Eat frequent meals with an equal balance of fat, carbohydrates, and protein. C) Eat frequent, low-fat meals with high protein content. D) Try to maintain the pre-diagnosis pattern of eating.

A The nurse encourages the patient to eat small, frequent portions of nonirritating foods to decrease gastric irritation. Food supplements should be high in calories, as well as vitamins A and C and iron, to enhance tissue repair.

The nurse is working on the renal transplant unit. To reduce the risk of infection in a patient with a transplanted kidney, it is imperative for the nurse to do what? A) Wash hands carefully and frequently. B) Ensure immediate function of the donated kidney. C) Instruct the patient to wear a face mask. D) Bar visitors from the patients room

A The nurse ensures that the patient is protected from exposure to infection by hospital staff, visitors, and other patients with active infections. Careful handwashing is imperative; face masks may be worn by hospital staff and visitors to reduce the risk for transmitting infectious agents while the patient is receiving high doses of immunosuppressants. Visitors may be limited, but are not normally barred outright. Ensuring kidney function is vital, but does not prevent infection.

A client is in the bariatric clinic 1 month after having gastric bypass surgery. The client is crying and says I didnt know it would be this hard to live like this. What response by the nurse is best? A) Assess the clients coping and support systems. B) Inform the client that things will get easier. C) Re-educate the client on needed dietary changes. D) Tell the client lifestyle changes are always hard.

A The nurse should assess this clients coping styles and support systems in order to provide holistic care. The other options do not address the clients distress.

The nurse notes that a patient who has undergone skin, tissue, and muscle grafting following a modified radical neck dissection requires suctioning. What is the most important consideration for the nurse when suctioning this patient? A) Avoid applying suction on or near the suture line. B) Position patient on the non operative side with the head of the bed down. C) Assess the patients ability to perform self-suctioning. D) Evaluate the patients ability to swallow saliva and clear fluids.

A The nurse should avoid positioning the suction catheter on or near the graft suture lines. Application of suction in these areas could damage the graft. Self-sectioning may be unsafe because the patient may damage the suture line. Following a modified radical neck dissection with graft, the patient is usually positioned with the head of the bed elevated to promote drainage and reduce edema. Assessing viability of the graft is important but is not part of the suctioning procedure and may delay initiating suctioning. Maintenance of a patent airway is a nursing priority. Similarly, the patients ability to swallow is an important assessment for the nurse to make; however, it is not directly linked to the patients need for suctioning.

A 40-year-old obese woman reports that she wants to lose weight. Which question should the nurse ask first? A) What factors led to your obesity? B) Which types of food do you like best? C) How long have you been overweight? D) What kind of activities do you enjoy?

A The nurse should obtain information about the patients perceptions of the reasons for the obesity to develop a plan individualized to the patient. The other information also will be obtained from the patient, but the patient is more likely to make changes when the patients beliefs are considered in planning.

A nurse is caring for a patient with hepatic encephalopathy. While making the initial shift assessment, the nurse notes that the patient has a flapping tremor of the hands. The nurse should document the presence of what sign of liver disease? A) Asterixis B) Constructional apraxia C) Fetor hepaticus D) Palmar erythema

A The nurse will document that a patient exhibiting a flapping tremor of the hands is demonstrating asterixis. While constructional apraxia is a motor disturbance, it is the inability to reproduce a simple figure. Fetor hepaticus is a sweet, slightly fecal odor to the breath and not associated with a motor disturbance. Skin changes associated with liver dysfunction may include palmar erythema, which is a reddening of the palms, but is not a flapping tremor.

The triage nurse is working in the ED. A homeless person is admitted during a blizzard with complaints of being unable to feel his feet and lower legs. Core temperature is noted at 33.2C (91.8F). The patient is intoxicated with alcohol at the time of admission and is visibly malnourished. What is the triage nurses priority in the care of this patient? A) Addressing the patients hypothermia B) Addressing the patients frostbite in his lower extremities C) Addressing the patients alcohol intoxication D) Addressing the patients malnutrition

A The patient may also have frostbite, but hypothermia takes precedence in treatment because it is systemic rather than localized. The alcohol abuse and the alteration in nutrition do not take precedence over the treatment of hypothermia because both problems are a less acute threat to the patients survival.

The paramedics bring a patient who has suffered a sexual assault to the ED. What is important for the sexual assault nurse examiner to do when assessing a sexual assault victim? A) Respect the patients privacy during assessment. B) Shave all pubic hair for laboratory analysis. C) Place items for evidence in plastic bags. D) Bathe the patient before the examination.

A The patients privacy and sensitivity must be respected, because the patient will be experiencing a stress response to the assault. Pubic hair is combed or trimmed for sampling. Paper bags are used for evidence collection because plastic bags retain moisture, which promotes mold and mildew that can destroy evidence. Bathing the patient before the examination would destroy or remove key evidence.

A nurse is preparing to provide care for a patient whose exacerbation of ulcerative colitis has required hospital admission. During an exacerbation of this health problem, the nurse would anticipate that the patients stools will have what characteristics? A) Watery with blood and mucus B) Hard and black or tarry C) Dry and streaked with blood D) Loose with visible fatty streaks

A The predominant symptoms of ulcerative colitis are diarrhea and abdominal pain. Stools may be bloody and contain mucus. Stools are not hard, dry, tarry, black or fatty in patients who have ulcerative colitis.

The nurse is assessing a patient suspected of having developed acute glomerulonephritis. The nurse should expect to address what clinical manifestation that is characteristic of this health problem? A) Hematuria B) Precipitous decrease in serum creatinine levels C) Hypotension unresolved by fluid administration D) Glucosuria

A The primary presenting feature of acute glomerulonephritis is hematuria (blood in the urine), which may be microscopic (identifiable through microscopic examination) or macroscopic or gross (visible to the eye). Proteinuria, primarily albumin, which is present, is due to increased permeability of the glomerular membrane. Blood urea nitrogen (BUN) and serum creatinine levels may rise as urine output drops. Some degree of edema and hypertension is noted in most patients.

A client has been prescribed lorcaserin (Belviq). What teaching is most appropriate? A) Increase the fiber and water in your diet. B) Reduce fat to less than 30% each day. C) Report dry mouth and decreased sweating. D) Lorcaserin may cause loose stools for a few days.

A This drug can cause constipation, so the client should increase fiber and water in the diet to prevent this from occurring. Reducing fat in the diet is important with orlistat. Lorcaserin can cause dry mouth but not decreased sweating. Loose stools are common with orlistat.

A 60-year-old patient with a diagnosis of prostate cancer is scheduled to have an interstitial implant for high-dose radiation (HDR). What safety measure should the nurse include in this patients subsequent plan of care? A) Limit the time that visitors spend at the patients bedside. B) Teach the patient to perform all aspects of basic care independently. C) Assign male nurses to the patients care whenever possible. D) Situate the patient in a shared room with other patients receiving brachytherapy.

A To limit radiation exposure, visitors should generally not spend more than 30 minutes with the patient. Pregnant nurses or visitors should not be near the patient, but there is no reason to limit care to nurses who are male. All necessary care should be provided to the patient and a single room should be used.

The nurse is caring for a patient admitted with a drug overdose. What is the nurses priority responsibility in caring for this patient? A) Support the patients respiratory and cardiovascular function. B) Provide for the safety of the patient. C) Enhance clearance of the offending agent. D) Ensure the safety of the staff.

A Treatment goals for a patient with a drug overdose are to support the respiratory and cardiovascular functions, to enhance clearance of the agent, and to provide for safety of the patient and staff. Of these responsibilities, however, support of vital physiologic function is a priority.

A 55-year-old female patient with hepatocellular carcinoma (HCC) is undergoing radiofrequency ablation. The nurse should recognize what goal of this treatment? A) Destruction of the patients liver tumor B) Restoration of portal vein patency C) Destruction of a liver abscess D) Reversal of metastasis

A Using radiofrequency ablation, a tumor up to 5 cm in size can be destroyed in one treatment session. This technique does not address circulatory function or abscess formation. It does not allow for the reversal of metastasis.

A nurse is caring for a patient who has had surgery for oral cancer. When addressing the patients long- term needs, the nurse should prioritize interventions and referrals with what goal? A) Enhancement of verbal communication B) Enhancement of immune function C) Maintenance of adequate social support D) Maintenance of fluid balance

A Verbal communication may be impaired by radical surgery for oral cancer. Addressing this impairment often requires a long-term commitment. Immune function, social support, and fluid balance are all necessary, but communication is a priority issue for patients recovering from this type of surgery.

You are caring for a patient who has just been told that her stage IV colon cancer has recurred and metastasized to the liver. The oncologist offers the patient the option of surgery to treat the progression of this disease. What type of surgery does the oncologist offer? A) Palliative B) Reconstructive C) Salvage D) Prophylactic

A When cure is not possible, the goals of treatment are to make the patient as comfortable as possible and to promote quality of life as defined by the patient and his or her family. Palliative surgery is performed in an attempt to relieve complications of cancer, such as ulceration, obstruction, hemorrhage, pain, and malignant effusion. Reconstructive surgery may follow curative or radical surgery in an attempt to improve function or obtain a more desirable cosmetic effect. Salvage surgery is an additional treatment option that uses an extensive surgical approach to treat the local recurrence of a cancer after the use of a less extensive primary approach. Prophylactic surgery involves removing nonvital tissues or organs that are at increased risk to develop cancer.

A patient is admitted to the medical unit with a diagnosis of intestinal obstruction. When planning this patients care, which of the following nursing diagnoses should the nurse prioritize? A) Ineffective Tissue Perfusion Related to Bowel Ischemia B) Imbalanced Nutrition: Less Than Body Requirements Related to Impaired Absorption C) Anxiety Related to Bowel Obstruction and Subsequent Hospitalization D) Impaired Skin Integrity Related to Bowel Obstruction

A When the bowel is completely obstructed, the possibility of strangulation and tissue necrosis (i.e., tissue death) warrants surgical intervention. As such, this immediate physiologic need is a nursing priority. Nutritional support and management of anxiety are necessary, but bowel ischemia is a more immediate threat. Skin integrity is not threatened.

The nurse and urologist have both been unsuccessful in catheterizing a patient with a prostatic obstruction and a full bladder. What approach does the nurse anticipate the physician using to drain the patients bladder? A) Insertion of a suprapubic catheter B) Scheduling the patient immediately for a prostatectomy C) Application of warm compresses to the perineum to assist with relaxation D) Medication administration to relax the bladder muscles and reattempting catheterization in 6 hours

A When the patient cannot void, catheterization is used to prevent overdistention of the bladder. In the case of prostatic obstruction, attempts at catheterization by the urologist may not be successful, requiring insertion of a suprapubic catheter. A prostatectomy may be necessary, but would not be undertaken for the sole purpose of relieving a urethral obstruction. Delaying by applying compresses or administering medications could result in harm.

A female patient has been prescribed a course of antibiotics for the treatment of a UTI. When providing health education for the patient, the nurse should address what topic? A) The risk of developing a vaginal yeast infection as a consequent of antibiotic therapy B) The need to expect a heavy menstrual period following the course of antibiotics C) The risk of developing antibiotic resistance after the course of antibiotics D) The need to undergo a series of three urine cultures after the antibiotics have been completed

A Yeast vaginitis occurs in as many as 25% of patients treated with antimicrobial agents that affect vaginal flora. Yeast vaginitis can cause more symptoms and be more difficult and costly to treat than the original UTI. Antibiotics do not affect menstrual periods and serial urine cultures are not normally necessary. Resistance is normally a result of failing to complete a prescribed course of antibiotics.

A nurse who works in an oncology clinic is assessing a patient who has arrived for a 2-month follow-up appointment following chemotherapy. The nurse notes that the patients skin appears yellow. Which blood tests should be done to further explore this clinical sign? A) Liver function tests (LFTs) B) Complete blood count (CBC) C) Platelet count D) Blood urea nitrogen and creatinine

A Yellow skin is a sign of jaundice and the liver is a common organ affected by metastatic disease. An LFT should be done to determine if the liver is functioning. A CBC, platelet count and tests of renal function would not directly assess for liver disease.

A nurse educator is teaching a group of recent nursing graduates about their occupational risks for contracting hepatitis B. What preventative measures should the educator promote? Select all that apply. A) Immunization B) Use of standard precautions C) Consumption of a vitamin-rich diet D) Annual vitamin K injections E) Annual vitamin B12 injections

A, B People who are at high risk, including nurses and other health care personnel exposed to blood or blood products, should receive active immunization. The consistent use of standard precautions is also highly beneficial. Vitamin supplementation is unrelated to an individuals risk of HBV.

A patient with pancreatic cancer has been scheduled for a pancreaticoduodenectomy (Whipple procedure). During health education, the patient should be informed that this procedure will involve the removal of which of the following? Select all that apply. A) Gallbladder B) Part of the stomach C) Duodenum D) Part of the common bile duct E) Part of the rectum

A, B, C, D A pancreaticoduodenectomy (Whipple procedure or resection) is used for potentially resectable cancer of the head of the pancreas (Fig. 50-7). This procedure involves removal of the gallbladder, a portion of the stomach, duodenum, proximal jejunum, head of the pancreas, and distal common bile duct. The rectum is not affected.

A nurse on the renal unit is caring for a patient who will soon begin peritoneal dialysis. The family of the patient asks for education about the peritoneal dialysis catheter that has been placed in the patients peritoneum. The nurse explains the three sections of the catheter and talks about the two cuffs on the dialysis catheter. What would the nurse explain about the cuffs? Select all that apply. A) The cuffs are made of Dacron polyester. B) The cuffs stabilize the catheter. C) The cuffs prevent the dialysate from leaking. D) The cuffs provide a barrier against microorganisms. E) The cuffs absorb dialysate

A, B, C, D Most of these catheters have two cuffs, which are made of Dacron polyester. The cuffs stabilize the catheter, limit movement, prevent leaks, and provide a barrier against microorganisms. They do not absorb dialysate.

The nurse is caring for a patient status after a motor vehicle accident. The patient has developed AKI. What is the nurses role in caring for this patient? Select all that apply. A) Providing emotional support for the family B) Monitoring for complications C) Participating in emergency treatment of fluid and electrolyte imbalances D) Providing nursing care for primary disorder (trauma) E) Directing nutritional interventions

A, B, C, D The nurse has an important role in caring for the patient with AKI. The nurse monitors for complications, participates in emergency treatment of fluid and electrolyte imbalances, assesses the patients progress and response to treatment, and provides physical and emotional support. Additionally, the nurse keeps family members informed about the patients condition, helps them understand the treatments, and provides psychological support. Although the development of AKI may be the most serious problem, the nurse continues to provide nursing care indicated for the primary disorder (e.g., burns, shock, trauma, obstruction of the urinary tract). The nurse does not direct the patients nutritional status; the dietician and the physician normally collaborate on directing the patients nutritional status.

A nurse is caring for a patient who has been admitted to the hospital with diverticulitis. Which of the following would be appropriate nursing diagnoses for this patient? Select all that apply. A) Acute Pain Related to Increased Peristalsis and GI Inflammation B) Activity Intolerance Related to Generalized Weakness C) Bowel Incontinence Related to Increased Intestinal Peristalsis D) Deficient Fluid Volume Related to Anorexia, Nausea, and Diarrhea E) Impaired Urinary Elimination Related to GI Pressure on the Bladder

A, B, D Patients with diverticulitis are likely to experience pain and decreased activity levels, and are at risk of fluid volume deficit. The patient is unlikely to experience fecal incontinence and urinary function is not directly influenced.

A patient with ESKD is scheduled to begin hemodialysis. The nurse is working with the patient to adapt the patients diet to maximize the therapeutic effect and minimize the risks of complications. The patients diet should include which of the following modifications? Select all that apply. A) Decreased protein intake B) Decreased sodium intake C) Increased potassium intake D) Fluid restriction E) Vitamin D supplementation

A, B, D Restricting dietary protein decreases the accumulation of nitrogenous wastes, reduces uremic symptoms, and may even postpone the initiation of dialysis for a few months. Restriction of fluid is also part of the dietary prescription because fluid accumulation may occur. As well, sodium is usually restricted to 2 to 3 g/day. Potassium intake is usually limited, not increased, and there is no particular need for vitamin D supplementation.

The nurse is describing some of the major characteristics of cancer to a patient who has recently received a diagnosis of malignant melanoma. When differentiating between benign and malignant cancer cells, the nurse should explain differences in which of the following aspects? Select all that apply. A) Rate of growth B) Ability to cause death C) Size of cells D) Cell contents E) Ability to spread

A, B, E Benign and malignant cells differ in many cellular growth characteristics, including the method and rate of growth, ability to metastasize or spread, general effects, destruction of tissue, and ability to cause death. Cells come in many sizes, both benign and malignant. Cell contents are basically the same, but they behave differently.

A patients assessment and diagnostic testing are suggestive of acute pancreatitis. When the nurse is performing the health interview, what assessment questions address likely etiologic factors? Select all that apply. A) How many alcoholic drinks do you typically consume in a week? B) Have you ever been tested for diabetes? C) Have you ever been diagnosed with gallstones? D) Would you say that you eat a particularly high-fat diet? E) Does anyone in your family have cystic fibrosis?

A, C Eighty percent of patients with acute pancreatitis have biliary tract disease such as gallstones or a history of long-term alcohol abuse. Diabetes, high-fat consumption, and cystic fibrosis are not noted etiologic factors.

A patients physician has ordered a liver panel in response to the patients development of jaundice. When reviewing the results of this laboratory testing, the nurse should expect to review what blood tests? Select all that apply. A) Alanine aminotransferase (ALT) B) C-reactive protein (CRP) C) Gamma-glutamyl transferase (GGT) D) Aspartate aminotransferase (AST) E) B-type natriuretic peptide (BNP)

A, C, D Liver function testing includes GGT, ALT, and AST. CRP addresses the presence of generalized inflammation and BNP is relevant to heart failure; neither is included in a liver panel.

You are caring for an adult patient who has developed a mild oral yeast infection following chemotherapy. What actions should you encourage the patient to perform? Select all that apply. A) Use a lip lubricant. B) Scrub the tongue with a firm-bristled toothbrush. C) Use dental floss every 24 hours. D) Rinse the mouth with normal saline. E) Eat spicy food to aid in eradicating the yeast.

A, C, D Stomatitis is an inflammation of the oral cavity. The patient should be encouraged to brush the teeth with a soft toothbrush after meals, use dental floss every 24 hours, rinse with normal saline, and use a lip lubricant. Mouthwashes and hot foods should be avoided.

A patient with end-stage liver disease has developed hypervolemia. What nursing interventions would be most appropriate when addressing the patients fluid volume excess? Select all that apply. A) Administering diuretics B) Administering calcium channel blockers C) Implementing fluid restrictions D) Implementing a 1500 kcal/day restriction E) Enhancing patient positioning

A, C, E Administering diuretics, implementing fluid restrictions, and enhancing patient positioning can optimize the management of fluid volume excess. Calcium channel blockers and calorie restriction do not address this problem.

A nurse is caring for a patient in the late stages of esophageal cancer. The nurse should plan to prevent or address what characteristics of this stage of the disease? Select all that apply. A) Perforation into the mediastinum B) Development of an esophageal lesion C) Erosion into the great vessels D) Painful swallowing E) Obstruction of the esophagus

A, C, E In the later stages of esophageal cancer, obstruction of the esophagus is noted, with possible perforation into the mediastinum and erosion into the great vessels. Painful swallowing and the emergence of a lesion are early signs of esophageal cancer.

A patient admitted to the ED with severe diarrhea and vomiting is subsequently diagnosed with food poisoning. The nurse caring for this patient assesses for signs and symptoms of fluid and electrolyte imbalances. For what signs and symptoms would this nurse assess? Select all that apply. A) Dysrhythmias B) Hypothermia C) Hypotension D) Hyperglycemia E) Delirium

A, C, E The patient is assessed for signs and symptoms of fluid and electrolyte imbalances, including lethargy, rapid pulse rate, fever, oliguria, anuria, hypotension, and delirium. Hyperglycemia and hypothermia are not typically associated with fluid and electrolyte imbalances.

A nurse is caring for a patient who has undergone neck resection with a radial forearm free flap. The nurses most recent assessment of the graft reveals that it has a bluish color and that mottling is visible. What is the nurses most appropriate action? A) Document the findings as being consistent with a viable graft. B) Promptly report these indications of venous congestion. C) Closely monitor the patient and reassess in 30 minutes. D) Reposition the patient to promote peripheral circulation.

B A graft that is blue with mottling may indicate venous congestion. This finding constitutes a risk for tissue ischemia and necrosis; prompt referral is necessary.

A 6-year-old is admitted to the ED after being rescued from a pond after falling through the ice while ice skating. What action should the nurse perform while rewarming the patient? A) Assessing the patients oral temperature frequently B) Ensuring continuous ECG monitoring C) Massaging the patients skin surfaces to promote circulation D) Administering bronchodilators by nebulizer

B A hypothermic patient requires continuous ECG monitoring and assessment of core temperatures with an esophageal probe, bladder, or rectal thermometer. Massage is not performed and bronchodilators would normally be insufficient to meet the patients respiratory needs.

A patient on the critical care unit is postoperative day 1 following kidney transplantation from a living donor. The nurses most recent assessments indicate that the patient is producing copious quantities of dilute urine. What is the nurses most appropriate response? A) Assess the patient for further signs or symptoms of rejection. B) Recognize this as an expected finding. C) Inform the primary care provider of this finding. D) Administer exogenous antidiuretic hormone as ordered.

B A kidney from a living donor related to the patient usually begins to function immediately after surgery and may produce large quantities of dilute urine. This is not suggestive of rejection and treatment is not warranted. There is no obvious need to report this finding.

A patient with a diagnosis of colon cancer is 2 days postoperative following bowel resection and anastomosis. The nurse has planned the patients care in the knowledge of potential complications. What assessment should the nurse prioritize? A) Close monitoring of temperature B) Frequent abdominal auscultation C) Assessment of hemoglobin, hematocrit, and red blood cell levels D) Palpation of peripheral pulses and leg girth

B After bowel surgery, it is important to frequently assess the abdomen, including bowel sounds and abdominal girth, to detect bowel obstruction. The resumption of bowel motility is a priority over each of the other listed assessments, even though each should be performed by the nurse.

A patient is being treated for bites that she suffered during an assault. After the bites have been examined and documented by a forensic examiner, the nurse should perform what action? A) Apply a dressing saturated with chlorhexidine. B) Wash the bites with soap and water. C) Arrange for the patient to receive a hepatitis B vaccination. D) Assess the patients immunization history.

B After forensic evidence has been gathered, cleansing with soap and water is necessary, followed by the administration of antibiotics and tetanus toxoid as prescribed. The patients immunization history does not directly influence the course of treatment and hepatitis B vaccination is not indicated. Chlorhexidine bandages are not recommended.

A patient with a diagnosis of esophageal varices has undergone endoscopy to gauge the progression of this complication of liver disease. Following the completion of this diagnostic test, what nursing intervention should the nurse perform? A) Keep patient NPO until the results of test are known. B) Keep patient NPO until the patients gag reflex returns. C) Administer analgesia until post-procedure tenderness is relieved. D) Give the patient a cold beverage to promote swallowing ability.

B After the examination, fluids are not given until the patients gag reflex returns. Lozenges and gargles may be used to relieve throat discomfort if the patients physical condition and mental status permit. The result of the test is known immediately. Food and fluids are contraindicated until the gag reflex returns.

A patient has undergone the creation of an Indiana pouch for the treatment of bladder cancer. The nurse identified the nursing diagnosis of disturbed body image. How can the nurse best address the effects of this urinary diversion on the patients body image? A) Emphasize that the diversion is an integral part of successful cancer treatment. B) Encourage the patient to speak openly and frankly about the diversion. C) Allow the patient to initiate the process of providing care for the diversion. D) Provide the patient with detailed written materials about the diversion at the time of discharge.

B Allowing the patient to express concerns and anxious feelings can help with body image, especially in adjusting to the changes in toileting habits. The nurse may have to initiate dialogue about the management of the diversion, especially if the patient is hesitant. Provision of educational materials is rarely sufficient to address a sudden change and profound change in body image. Emphasizing the role of the diversion in cancer treatment does not directly address the patients body image.

A participant in a health fair has asked the nurse about the role of drugs in liver disease. What health promotion teaching has the most potential to prevent drug-induced hepatitis? A) Finish all prescribed courses of antibiotics, regardless of symptom resolution. B) Adhere to dosing recommendations of OTC analgesics. C) Ensure that expired medications are disposed of safely. D) Ensure that pharmacists regularly review drug regimens for potential interactions.

B Although any medication can affect liver function, use of acetaminophen (found in many over-the- counter medications used to treat fever and pain) has been identified as the leading cause of acute liver failure. Finishing prescribed antibiotics and avoiding expired medications are unrelated to this disease. Drug interactions are rarely the cause of drug-induced hepatitis.

A 71-year-old patient with ESKD has been told by the physician that it is time to consider hemodialysis until a transplant can be found. The patient tells the nurse she is not sure she wants to undergo a kidney transplant. What would be an appropriate response for the nurse to make? A) The decision is certainly yours to make, but be sure not to make a mistake. B) Kidney transplants in patients your age are as successful as they are in younger patients. C) I understand your hesitancy to commit to a transplant surgery. Success is comparatively rare. D) Have you talked this over with your family?

B Although there is no specific age limitation for renal transplantation, concomitant disorders (e.g., coronary artery disease, peripheral vascular disease) have made it a less common treatment for the elderly. However, the outcome is comparable to that of younger patients. The other listed options either belittle the patient or give the patient misinformation

A patient has been diagnosed with an esophageal diverticulum after undergoing diagnostic imaging. When taking the health history, the nurse should expect the patient to describe what sign or symptom? A) Burning pain on swallowing B) Regurgitation of undigested food C) Symptoms mimicking a heart attack D) Chronic parotid abscesses

B An esophageal diverticulum is an outpouching of mucosa and submucosa that protrudes through the esophageal musculature. Food becomes trapped in the pouch and is frequently regurgitated when the patient assumes a recumbent position. The patient may experience difficulty swallowing; however, burning pain is not a typical finding. Symptoms mimicking a heart attack are characteristic of GERD. Chronic parotid abscesses are not associated with a diagnosis of esophageal diverticulum.

A nurse caring for a patient with colorectal cancer is preparing the patient for upcoming surgery. The nurse administers cephalexin (Keflex) to the patient and explains what rationale? A) To treat any undiagnosed infections B) To reduce intestinal bacteria levels C) To reduce bowel motility D) To reduce abdominal distention postoperatively

B Antibiotics such a kanamycin (Kantrex), neomycin (Mycifradin), and cephalexin (Keflex) are administered orally the day before surgery to reduce intestinal bacterial. Preoperative antibiotics are not given to treat undiagnosed infections, reduce motility, or prevent abdominal distention.

Which finding for a patient who has been taking orlistat (Xenical) is most important to report to the health care provider? A) The patient frequently has liquid stools. B) The patient is pale and has many bruises. C) The patient complains of bloating after meals. D) The patient is experiencing a weight loss plateau.

B Because orlistat blocks the absorption of fat-soluble vitamins, the patient may not be receiving an adequate amount of vitamin K, resulting in a decrease in clotting factors. Abdominal bloating and liquid stools are common side effects of orlistat and indicate that the nurse should remind the patient that fat in the diet may increase these side effects. Weight loss plateaus are normal during weight reduction.

Which nursing action is appropriate when coaching obese adults enrolled in a behavior modification program? A) Having the adults write down the caloric intake of each meal B) Asking the adults about situations that tend to increase appetite C) Suggesting that the adults plan rewards, such as sugarless candy, for achieving their goals D) Encouraging the adults to eat small amounts frequently rather than having scheduled meals

B Behavior modification programs focus on how and when the person eats and de-emphasize aspects such as calorie counting. Nonfood rewards are recommended for achievement of weight-loss goals. Patients are often taught to restrict eating to designated meals when using behavior modification.

A patient returns to the unit after a neck dissection. The surgeon placed a Jackson Pratt drain in the wound. When assessing the wound drainage over the first 24 postoperative hours the nurse would notify the physician immediately for what? A) Presence of small blood clots in the drainage B) 60 mL of milky or cloudy drainage C) Spots of drainage on the dressings surrounding the drain D) 120 mL of serosanguinous drainage

B Between 80 and 120 mL of serosanguineous secretions may drain over the first 24 hours. Milky drainage is indicative of a chyle fistula, which requires prompt treatment.

A 13-year-old is being admitted to the ED after falling from a roof and sustaining blunt abdominal injuries. To assess for internal injury in the patients peritoneum, the nurse should anticipate what diagnostic test? A) Radiograph B) Computed tomography (CT) scan C) Complete blood count (CBC) D) Barium swallow

B CT scan of the abdomen, diagnostic peritoneal lavage, and abdominal ultrasound are appropriate diagnostic tools to assess intra-abdominal injuries. X-rays do not yield sufficient data and a CBC would not reveal the presence of intraperitoneal injury.

The public health nurse is presenting a health-promotion class to a group at a local community center. Which intervention most directly addresses the leading cause of cancer deaths in North America? A) Monthly self-breast exams B) Smoking cessation C) Annual colonoscopies D) Monthly testicular exams

B Cancer is second only to cardiovascular disease as a leading cause of death in the United States. Although the numbers of cancer deaths have decreased slightly, more than 570,000 Americans were expected to die from a malignant process in 2011. The leading causes of cancer death in the United States, in order of frequency, are lung, prostate, and colorectal cancer in men and lung, breast, and colorectal cancer in women, so smoking cessation is the health promotion initiative directly related to lung cancer.

A patient with a sacral pressure ulcer has had a urinary catheter inserted. As a result of this new intervention, the nurse should prioritize what nursing diagnosis in the patients plan of care? A) Impaired physical mobility related to presence of an indwelling urinary catheter B) Risk for infection related to presence of an indwelling urinary catheter C) Toileting self-care deficit related to urinary catheterization D) Disturbed body image related to urinary catheterization

B Catheters create a high risk for UTIs. Because of this acute physiologic threat, the patients risk for infection is usually prioritized over functional and psychosocial diagnoses.

The nurse is admitting an oncology patient to the unit prior to surgery. The nurse reads in the electronic health record that the patient has just finished radiation therapy. With knowledge of the consequent health risks, the nurse should prioritize assessments related to what health problem? A) Cognitive deficits B) Impaired wound healing C) Cardiac tamponade D) Tumor lysis syndrome

B Combining other treatment methods, such as radiation and chemotherapy, with surgery contributes to postoperative complications, such as infection, impaired wound healing, altered pulmonary or renal function, and the development of deep vein thrombosis.

Which information will the nurse prioritize in planning preoperative teaching for a patient undergoing a Roux-en-Y gastric bypass? A) Educating the patient about the nasogastric (NG) tube B) Instructing the patient on coughing and breathing techniques C) Discussing necessary postoperative modifications in lifestyle D) Demonstrating passive range-of-motion exercises for the legs

B Coughing and deep breathing can prevent major postoperative complications such as carbon monoxide retention and hypoxemia. Information about passive range of motion, the NG tube, and postoperative modifications in lifestyle will also be discussed, but avoidance of respiratory complications is the priority goal after surgery.

Which assessment action will help the nurse determine if an obese patient has metabolic syndrome? A) Take the patients apical pulse. B) Check the patients blood pressure. C) Ask the patient about dietary intake. D) Dipstick the patients urine for protein.

B Elevated blood pressure is one of the characteristics of metabolic syndrome. The other information also may be obtained by the nurse, but it will not assist with the diagnosis of metabolic syndrome.

A patient has recently received a diagnosis of gastric cancer; the nurse is aware of the importance of assessing the patients level of anxiety. Which of the following actions is most likely to accomplish this? A) The nurse gauges the patients response to hypothetical outcomes. B) The patient is encouraged to express fears openly. C) The nurse provides detailed and accurate information about the disease. D) The nurse closely observes the patients body language.

B Encouraging the patient to discuss his or her fears and anxieties is usually the best way to assess a patients anxiety. Presenting hypothetical situations is a surreptitious and possibly inaccurate way of assessing anxiety. Observing body language is part of assessment, but it is not the complete assessment. Presenting information may alleviate anxiety for some patients, but it is not an assessment.

A nurse is caring for a patient who has been admitted for the treatment of advanced cirrhosis. What assessment should the nurse prioritize in this patients plan of care? A) Measurement of abdominal girth and body weight B) Assessment for variceal bleeding C) Assessment for signs and symptoms of jaundice D) Monitoring of results of liver function testing

B Esophageal varices are a major cause of mortality in patients with uncompensated cirrhosis. Consequently, this should be a focus of the nurses assessments and should be prioritized over the other listed assessments, even though each should be performed.

A patient with a history of major depression is brought to the ED by her parents. Which of the following nursing actions is most appropriate? A) Noting that symptoms of physical illness are not relevant to the current diagnosis B) Asking the patient if she has ever thought about taking her own life C) Conducting interviews in a brief and direct manner D) Arranging for the patient to spend time alone to consider her feelings

B Establishing if the patient has suicidal thoughts or intents helps identify the level of depression and intervention. Physical symptoms are relevant and should be explored. Allow the patient to express feelings, and conduct the interview at a comfortable pace for the patient. Never leave the patient alone, because suicide is usually committed in solitude.

The nurse is caring for a patient who has just returned from the ERCP removal of gallstones. The nurse should monitor the patient for signs of what complications? A) Pain and peritonitis B) Bleeding and perforation C) Acidosis and hypoglycemia D) Gangrene of the gallbladder and hyperglycemia

B Following ERCP removal of gallstones, the patient is observed closely for bleeding, perforation, and the development of pancreatitis or sepsis. Blood sugar alterations, gangrene, peritonitis, and acidosis are less likely complications.

A nurse is providing care for a patient whose recent colostomy has contributed to a nursing diagnosis of Disturbed Body Image Related to Colostomy. What intervention best addresses this diagnosis? A) Encourage the patient to conduct online research into colostomies. B) Engage the patient in the care of the ostomy to the extent that the patient is willing. C) Emphasize the fact that the colostomy was needed to alleviate a much more serious health problem. D) Emphasize the fact that the colostomy is temporary measure and is not permanent.

B For some patients, becoming involved in the care of the ostomy helps to normalize it and enhance familiarity. Emphasizing the benefits of the intervention is unlikely to improve the patients body image, since the benefits are likely already known. Online research is not likely to enhance the patients body image and some ostomies are permanent.

A patient has had an ileostomy created for the treatment of irritable bowel disease and the patient is now preparing for discharge. What should the patient be taught about changing this device in the home setting? A) Apply antibiotic ointment as ordered after cleaning the stoma. B) Apply a skin barrier to the peristomal skin prior to applying the pouch. C) Dispose of the clamp with each bag change. D) Cleanse the area surrounding the stoma with alcohol or chlorhexidine.

B Guidelines for changing an ileostomy appliance are as follows. Skin should be washed with soap and water, and dried. A skin barrier should be applied to the peristomal skin prior to applying the pouch. Clamps are supplied one per box and should be reused with each bag change. Topical antibiotics are not utilized, but an antifungal spray or powder may be used.

The nursing educator is reviewing the signs and symptoms of heat stroke with a group of nurses who provide care in a desert region. The educator should describe what sign or symptom? A) Hypertension with a wide pulse pressure B) Anhidrosis C) Copious diuresis D) Cheyne-Stokes respirations

B Heat stroke is manifested by anhidrosis confusion, bizarre behavior, coma, elevated body temperature, hot dry skin, tachypnea, hypotension, and tachycardia. This health problem is not associated with anhidrosis or Cheyne-Stokes respirations.

During a patients scheduled home visit, an older adult patient has stated to the community health nurse that she has been experiencing hemorrhoids of increasing severity in recent months. The nurse should recommend which of the following? A) Regular application of an OTC antibiotic ointment B) Increased fluid and fiber intake C) Daily use of OTC glycerin suppositories D) Use of an NSAID to reduce inflammation

B Hemorrhoid symptoms and discomfort can be relieved by good personal hygiene and by avoiding excessive straining during defecation. A high-residue diet that contains fruit and bran along with an increased fluid intake may be all the treatment that is necessary to promote the passage of soft, bulky stools to prevent straining. Antibiotics, regular use of suppositories, and NSAIDs are not recommended, as they do not address the etiology of the health problem.

A nursing instructor is discussing hemorrhoids with the nursing class. Which patients would the nursing instructor identify as most likely to develop hemorrhoids? A) A 45-year-old teacher who stands for 6 hours per day B) A pregnant woman at 28 weeks gestation C) A 37-year-old construction worker who does heavy lifting D) A 60-year-old professional who is under stress

B Hemorrhoids commonly affect 50% of patients after the age of 50. Pregnancy may initiate hemorrhoids or aggravate existing ones. This is due to increased constipation during pregnancy. The significance of pregnancy is greater than that of standing, lifting, or stress in the development of hemorrhoids.

A patient has been diagnosed with acute pancreatitis. The nurse is addressing the diagnosis of Acute Pain Related to Pancreatitis. What pharmacologic intervention is most likely to be ordered for this patient? A) Oral oxycodone B) IV hydromorphone (Dilaudid) C) IM meperidine (Demerol) D) Oral naproxen (Aleve)

B The pain of acute pancreatitis is often very severe and pain relief may require parenteral opioids such as morphine, fentanyl (Sublimaze), or hydromorphone (Dilaudid). There is no clinical evidence to support the use of meperidine for pain relief in pancreatitis. Opioids are preferred over NSAIDs.

A patient with a diagnosis of gastric cancer has been unable to tolerate oral food and fluid intake and her tumor location precludes the use of enteral feeding. What intervention should the nurse identify as best meeting this patients nutritional needs? A) Administration of parenteral feeds via a peripheral IV B) TPN administered via a peripherally inserted central catheter C) Insertion of an NG tube for administration of feeds D) Maintaining NPO status and IV hydration until treatment completion

B If malabsorption is severe, or the cancer involves the upper GI tract, parenteral nutrition may be necessary. TPN is administered by way of a central line, not a peripheral IV. An NG would be contraindicated for this patient. Long-term NPO status would result in malnutrition.

A patient has had a laparoscopic cholecystectomy. The patient is now complaining of right shoulder pain. What should the nurse suggest to relieve the pain? A) Aspirin every 4 to 6 hours as ordered B) Application of heat 15 to 20 minutes each hour C) Application of an ice pack for no more than 15 minutes D) Application of liniment rub to affected area

B If pain occurs in the right shoulder or scapular area (from migration of the CO2 used to insufflate the abdominal cavity during the procedure), the nurse may recommend use of a heating pad for 15 to 20 minutes hourly, walking, and sitting up when in bed. Aspirin would constitute a risk for bleeding.

A nurse is caring for a patient with a blocked bile duct from a tumor. What manifestation of obstructive jaundice should the nurse anticipate? A) Watery, blood-streaked diarrhea B) Orange and foamy urine C) Increased abdominal girth D) Decreased cognition

B If the bile duct is obstructed, the bile will be reabsorbed into the blood and carried throughout the entire body. It is excreted in the urine, which becomes deep orange and foamy. Bloody diarrhea, ascites, and cognitive changes are not associated with obstructive jaundice.

A patient is being treated for AKI and the patient daily weights have been ordered. The nurse notes a weight gain of 3 pounds over the past 48 hours. What nursing diagnosis is suggested by this assessment finding? A) Imbalanced nutrition: More than body requirements B) Excess fluid volume C) Sedentary lifestyle D) Adult failure to thrive

B If the patient with AKI gains or does not lose weight, fluid retention should be suspected. Short-term weight gain is not associated with excessive caloric intake or a sedentary lifestyle. Failure to thrive is not associated with weight gain.

A patient with chronic kidney disease is completing an exchange during peritoneal dialysis. The nurse observes that the peritoneal fluid is draining slowly and that the patients abdomen is increasing in girth. What is the nurses most appropriate action? A) Advance the catheter 2 to 4 cm further into the peritoneal cavity. B) Reposition the patient to facilitate drainage. C) Aspirate from the catheter using a 60-mL syringe. D) Infuse 50 mL of additional dialysate.

B If the peritoneal fluid does not drain properly, the nurse can facilitate drainage by turning the patient from side to side or raising the head of the bed. The catheter should never be pushed further into the peritoneal cavity. It would be unsafe to aspirate or to infuse more dialysate.

The nurse has implemented a bladder retraining program for an older adult patient. The nurse places the patient on a timed voiding schedule and performs an ultrasonic bladder scan after each void. The nurse notes that the patient typically has approximately 50 mL of urine remaining in her bladder after voiding. What would be the nurses best response to this finding? A) Perform a straight catheterization on this patient. B) Avoid further interventions at this time, as this is an acceptable finding. C) Place an indwelling urinary catheter. D) Press on the patients bladder in an attempt to encourage complete emptying.

B In adults older than 60 years of age, 50 to 100 mL of residual urine may remain after each voiding because of the decreased contractility of the detrusor muscle. Consequently, further interventions are not likely warranted.

A patient has been successfully treated for kidney stones and is preparing for discharge. The nurse recognizes the risk of recurrence and has planned the patients discharge education accordingly. What preventative measure should the nurse encourage the patient to adopt? A) Increasing intake of protein from plant sources B) Increasing fluid intake C) Adopting a high-calcium diet D) Eating several small meals each day

B Increased fluid intake is encouraged to prevent the recurrence of kidney stones. Protein intake from all sources should be limited. Most patients do not require a low-calcium diet, but increased calcium intake would be contraindicated for all patients. Eating small, frequent meals does not influence the risk for recurrence.

A nurse caring for a patient with a newly created ileostomy assesses the patient and notes that the patient has had not ostomy output for the past 12 hours. The patient also complains of worsening nausea. What is the nurses priority action? A) Facilitate a referral to the wound-ostomy-continence (WOC) nurse. B) Report signs and symptoms of obstruction to the physician. C) Encourage the patient to mobilize in order to enhance motility. D) Contact the physician and obtain a swab of the stoma for culture.

B It is important to report nausea and abdominal distention, which may indicate intestinal obstruction. This requires prompt medical intervention. Referral to the WOC nurse is not an appropriate short-term response, since medical treatment is necessary. Physical mobility will not normally resolve an obstruction. There is no need to collect a culture from the stoma, because infection is unrelated to this problem.

The clinic nurse is caring for a patient whose grandmother and sister have both had breast cancer. She requested a screening test to determine her risk of developing breast cancer and it has come back positive. The patient asks you what she can do to help prevent breast cancer from occurring. What would be your best response? A) Research has shown that eating a healthy diet can provide all the protection you need against breast cancer. B) Research has shown that taking the drug tamoxifen can reduce your chance of breast cancer. C) Research has shown that exercising at least 30 minutes every day can reduce your chance of breast cancer. D) Research has shown that there is little you can do to reduce your risk of breast cancer if you have a genetic predisposition.

B Large-scale breast cancer prevention studies supported by the National Cancer Institute (NCI) indicated that chemoprevention with the medication tamoxifen can reduce the incidence of breast cancer by 50% in women at high risk for breast cancer. A healthy diet and regular exercise are important, but not wholly sufficient preventive measures.

A nurse is caring for a patient with constipation whose primary care provider has recommended senna (Senokot) for the management of this condition. The nurse should provide which of the following education points? A) Limit your fluid intake temporarily so you dont get diarrhea. B) Avoid taking the drug on a long-term basis. C) Make sure to take a multivitamin with each dose. D) Take this on an empty stomach to ensure maximum effect.

B Laxatives should not be taken on an ongoing basis in order to reduce the risk of dependence. Fluid should be increased, not limited, and there is no need to take each dose with a multivitamin. Senna does not need to be taken on an empty stomach.

A patients most recent diagnostic imaging has revealed that his lung cancer has metastasized to his bones and liver. What is the most likely mechanism by which the patients cancer cells spread? A) Hematologic spread B) Lymphatic circulation C) Invasion D) Angiogenesis

B Lymph and blood are key mechanisms by which cancer cells spread. Lymphatic spread (the transport of tumor cells through the lymphatic circulation) is the most common mechanism of metastasis.

The nurses comprehensive assessment of a patient includes inspection for signs of oral cancer. What assessment finding is most characteristic of oral cancer in its early stages? A) Dull pain radiating to the ears and teeth B) Presence of a painless sore with raised edges C) Areas of tenderness that make chewing difficult D) Diffuse inflammation of the buccal mucosa

B Malignant lesions of the oral cavity are most often painless lumps or sores with raised borders. Because they do not bother the patient, delay in seeking treatment occurs frequently, and negatively affects prognosis. Dull pain radiating to the ears and teeth is characteristic of malocclusion. Inflammation of the buccal mucosa causes discomfort and often occurs as a side effect of chemotherapy. Tenderness resulting in pain on chewing may be associated with gingivitis, abscess, irritation from dentures, and other causes. Pain related to oral cancer is a late symptom.

A nurse is caring for a patient who has been scheduled for endoscopic retrograde cholangiopancreatography (ERCP) the following day. When providing anticipatory guidance for this patient, the nurse should describe what aspect of this diagnostic procedure? A) The need to protect the incision postprocedure B) The use of moderate sedation C) The need to infuse 50% dextrose during the procedure D) The use of general anesthesia

B Moderate sedation, not general anesthesia, is used during ERCP. D50 is not administered and the procedure does not involve the creation of an incision.

A nurse is caring for a patient with cirrhosis secondary to heavy alcohol use. The nurses most recent assessment reveals subtle changes in the patients cognition and behavior. What is the nurses most appropriate response? A) Ensure that the patients sodium intake does not exceed recommended levels. B) Report this finding to the primary care provider due to the possibility of hepatic encephalopathy. C) Inform the primary care provider that the patient should be assessed for alcoholic hepatitis. D) Implement interventions aimed at ensuring a calm and therapeutic care environment.

B Monitoring is an essential nursing function to identify early deterioration in mental status. The nurse monitors the patients mental status closely and reports changes so that treatment of encephalopathy can be initiated promptly. This change in status is likely unrelated to sodium intake and would not signal the onset of hepatitis. A supportive care environment is beneficial, but does not address the patients physiologic deterioration.

A client is awaiting bariatric surgery in the morning. What action by the nurse is most important? A) Answering questions the client has about surgery B) Beginning venous thromboembolism prophylaxis C) Informing the client that he or she will be out of bed tomorrow D) Teaching the client about needed dietary changes

B Morbidly obese clients are at high risk of venous thromboembolism and should be started on a regimen to prevent this from occurring as a priority. Answering questions about the surgery is done by the surgeon. Teaching is important, but safety comes first.

A patient is admitted to the ED with an apparent overdose of IV heroin. After stabilizing the patients cardiopulmonary status, the nurse should prepare to perform what intervention? A) Administer a bolus of lactated Ringers. B) Administer naloxone hydrochloride (Narcan). C) Insert an indwelling urinary catheter. D) Perform a focused neurologic assessment.

B Narcan is an opioid antagonist that is administered for the treatment of narcotic overdoses. There is no definitive need for a urinary catheter or for a bolus of lactated Ringers. The patients basic neurologic status should be ascertained during the rapid assessment, but a detailed examination would be take precedence over administration of an antidote.

A patient newly diagnosed with cancer is scheduled to begin chemotherapy treatment and the nurse is providing anticipatory guidance about potential adverse effects. When addressing the most common adverse effect, what should the nurse describe? A) Pruritis (itching) B) Nausea and vomiting C) Altered glucose metabolism D) Confusion

B Nausea and vomiting, the most common side effects of chemotherapy, may persist for as long as 24 to 48 hours after its administration. Antiemetic drugs are frequently prescribed for these patients. Confusion, alterations in glucose metabolism, and pruritis are not common adverse effects.

A nurse is creating a plan of care for an oncology patient and one of the identified nursing diagnoses is risk for infection related to myelosuppression. What intervention addresses the leading cause of infection-related death in oncology patients? A) Encourage several small meals daily. B) Provide skin care to maintain skin integrity. C) Assist the patient with hygiene, as needed. D) Assess the integrity of the patients oral mucosa regularly.

B Nursing care for patients with skin reactions includes maintaining skin integrity, cleansing the skin, promoting comfort, reducing pain, preventing additional trauma, and preventing and managing infection. Malnutrition in oncology patients may be present, but it is not the leading cause of infection-related death. Poor hygiene does not normally cause events that result in death. Broken oral mucosa may be an avenue for infection, but it is not the leading cause of death in an oncology patient.

A patient admitted with inflammatory bowel disease asks the nurse for help with menu selections. What menu selection is most likely the best choice for this patient? A) Spinach B) Tofu C) Multigrain bagel D) Blueberries

B Nutritional management of inflammatory bowel disease requires ingestion of a diet that is bland, low- residue, high-protein, and high-vitamin. Tofu meets each of the criteria. Spinach, multigrain bagels, and blueberries are not low-residue.

A patient on the medical unit has a documented history of polycystic kidney disease (PKD). What principle should guide the nurses care of this patient? A) The disease is self-limiting and cysts usually resolve spontaneously in the fifth or sixth decade of life. B) The patients disease is incurable and the nurses interventions will be supportive. C) The patient will eventually require surgical removal of his or her renal cysts. D) The patient is likely to respond favorably to lithotripsy treatment of the cysts.

B PKD is incurable and care focuses on support and symptom control. It is not self-limiting and is not treated surgically or with lithotripsy.

A patient with chronic pancreatitis had a pancreaticojejunostomy created 3 months ago for relief of pain and to restore drainage of pancreatic secretions. The patient has come to the office for a routine postsurgical appointment. The patient is frustrated that the pain has not decreased. What is the most appropriate initial response by the nurse? A) The majority of patients who have a pancreaticojejunostomy have their normal digestion restored but do not achieve pain relief. B) Pain relief occurs by 6 months in most patients who undergo this procedure, but some people experience a recurrence of their pain. C) Your physician will likely want to discuss the removal of your gallbladder to achieve pain relief. D) You are probably not appropriately taking the medications for your pancreatitis and pain, so we will need to discuss your medication regimen in detail.

B Pain relief from a pancreaticojejunostomy often occurs by 6 months in more than 85% of the patients who undergo this procedure, but pain returns in a substantial number of patients as the disease progresses. This patient had surgery 3 months ago; the patient has 3 months before optimal benefits of the procedure may be experienced. There is no obvious indication for gallbladder removal and nonadherence is not the most likely factor underlying the pain.

A nurse on a busy medical unit provides care for many patients who require indwelling urinary catheters at some point during their hospital care. The nurse should recognize a heightened risk of injury associated with indwelling catheter use in which patient? A) A patient whose diagnosis of chronic kidney disease requires a fluid restriction B) A patient who has Alzheimers disease and who is acutely agitated C) A patient who is on bed rest following a recent episode of venous thromboembolism D) A patient who has decreased mobility following a transmetatarsal amputation

B Patients who are confused and agitated risk trauma through the removal of an indwelling catheter which has the balloon still inflated. Recent VTE, amputation, and fluid restriction do not directly create a risk for injury or trauma associated with indwelling catheter use.

The clinic nurse is preparing a plan of care for a patient with a history of stress incontinence. What role will the nurse have in implementing a behavioral therapy approach? A) Provide medication teaching related to pseudoephedrine sulfate. B) Teach the patient to perform pelvic floor muscle exercises. C) Prepare the patient for an anterior vaginal repair procedure. D) Provide information on periurethral bulking.

B Pelvic floor muscle exercises (sometimes called Kegel exercises) represent the cornerstone of behavioral intervention for addressing symptoms of stress, urge, and mixed incontinence. None of the other listed interventions has a behavioral approach.

A patient is brought to the ED by ambulance with a gunshot wound to the abdomen. The nurse knows that the most common hollow organ injured in this type of injury is what? A) Liver B) Small bowel C) Stomach D) Large bowel

B Penetrating abdominal wounds have a high incidence of injury to hollow organs, especially the small bowel. The liver is also injured frequently, but it is a solid organ.

A patient is scheduled for a CT scan of the abdomen with contrast. The patient has a baseline creatinine level of 2.3 mg/dL. In preparing this patient for the procedure, the nurse anticipates what orders? A) Monitor the patients electrolyte values every hour before the procedure. B) Preprocedure hydration and administration of acetylcysteine C) Hemodialysis immediately prior to the CT scan D) Obtain a creatinine clearance by collecting a 24-hour urine specimen.

B Radiocontrast-induced nephropathy is a major cause of hospital-acquired acute kidney injury. Baseline levels of creatinine greater than 2 mg/dL identify the patient as being high risk. Preprocedure hydration and prescription of acetylcysteine (Mucomyst) the day prior to the test is effective in prevention. The nurse would not monitor the patients electrolytes every hour preprocedure. Nothing in the scenario indicates the need for hemodialysis. A creatinine clearance is not necessary prior to a CT scan with contrast.

A patient has been admitted to the critical care unit with a diagnosis of toxic hepatitis. When planning the patients care, the nurse should be aware of what potential clinical course of this health problem? Place the following events in the correct sequence. 1. Fever rises. 2. Hematemesis. 3. Clotting abnormalities. 4. Vascular collapse. 5. Coma. A) 1, 2, 5, 4, 3 B) 1, 2, 3, 4, 5 C) 2, 3, 1, 4, 5 D) 3, 1, 2, 5, 4

B Recovery from acute toxic hepatitis is rapid if the hepatotoxin is identified early and removed or if exposure to the agent has been limited. Recovery is unlikely if there is a prolonged period between exposure and onset of symptoms. There are no effective antidotes. The fever rises; the patient becomes toxic and prostrated. Vomiting may be persistent, with the emesis containing blood. Clotting abnormalities may be severe, and hemorrhages may appear under the skin. The severe GI symptoms may lead to vascular collapse. Delirium, coma, and seizures develop, and within a few days the patient may die of fulminant hepatic failure unless he or she receives a liver transplant.

The nurse observes that the family members of a patient who was injured in an accident are blaming each other for the circumstances leading up to the accident. The nurse appropriately lets the family members express their feelings of responsibility, while explaining that there was probably little they could do to prevent the injury. In what stage of crisis is this family? A) Anxiety and denial B) Remorse and guilt C) Anger D) Grief

B Remorse and guilt are natural processes of the stages of a crisis and should be facilitated for the family members to process the crisis. The familys sense of blame and responsibility are more suggestive of guilt than anger, grief, or anxiety.

The nurse is caring for a patient with an advanced stage of breast cancer and the patient has recently learned that her cancer has metastasized. The nurse enters the room and finds the patient struggling to breath and the nurses rapid assessment reveals that the patients jugular veins are distended. The nurse should suspect the development of what oncologic emergency? A) Increased intracranial pressure B) Superior vena cava syndrome (SVCS) C) Spinal cord compression D) Metastatic tumor of the neck

B SVCS occurs when there is gradual or sudden impaired venous drainage giving rise to progressive shortness of breath (dyspnea), cough, hoarseness, chest pain, and facial swelling; edema of the neck, arms, hands, and thorax and reported sensation of skin tightness and difficulty swallowing; as well as possibly engorged and distended jugular, temporal, and arm veins. Increased intracranial pressure may be a part of SVCS, but it is not what is causing the patients symptoms. The scenario does not mention a problem with the patients spinal cord. The scenario says that the cancer has metastasized, but not that it has metastasized to the neck.

A nurse at an outpatient surgery center is caring for a patient who had a hemorrhoidectomy. What discharge education topics should the nurse address with this patient? A) The appropriate use of antibiotics to prevent postoperative infection B) The correct procedure for taking a sitz bath C) The need to eat a low-residue, low-fat diet for the next 2 weeks D) The correct technique for keeping the perianal region clean without the use of water

B Sitz baths are usually indicated after perianal surgery. A low-residue, low-fat diet is not necessary and water is used to keep the region clean. Postoperative antibiotics are not normally prescribed.

The nurse coming on shift on the medical unit is taking a report on four patients. What patient does the nurse know is at the greatest risk of developing ESKD? A) A patient with a history of polycystic kidney disease B) A patient with diabetes mellitus and poorly controlled hypertension C) A patient who is morbidly obese with a history of vascular disorders D) A patient with severe chronic obstructive pulmonary disease

B Systemic diseases, such as diabetes mellitus (leading cause); hypertension; chronic glomerulonephritis; pyelonephritis; obstruction of the urinary tract; hereditary lesions, such as in polycystic kidney disease; vascular disorders; infections; medications; or toxic agents may cause ESKD. A patient with more than one of these risk factors is at the greatest risk for developing ESKD. Therefore, the patient with diabetes and hypertension is likely at highest risk for ESKD.

The nurse is teaching a health class about UTIs to a group of older adults. What characteristic of UTIs should the nurse cite? A) Men over age 65 are equally prone to UTIs as women, but are more often asymptomatic. B) The prevalence of UTIs in men older than 50 years of age approaches that of women in the same age group. C) Men of all ages are less prone to UTIs, but typically experience more severe symptoms. D) The prevalence of UTIs in men cannot be reliably measured, as men generally do not report UTIs.

B The antibacterial activity of the prostatic secretions that protect men from bacterial colonization of the urethra and bladder decreases with aging. The prevalence of infection in men older than 50 years of age approaches that of women in the same age group. Men are not more likely to be asymptomatic and are not known to be reluctant to report UTIs.

A 54-year-old has a diagnosis of breast cancer and is tearfully discussing her diagnosis with the nurse. The patient states, They tell me my cancer is malignant, while my coworker's breast tumor was benign. I just don't understand at all. When preparing a response to this patient, the nurse should be cognizant of what characteristic that distinguishes malignant cells from benign cells of the same tissue type? A) Slow rate of mitosis of cancer cells B) Different proteins in the cell membrane C) Differing size of the cells D) Different molecular structure in the cells

B The cell membrane of malignant cells also contains proteins called tumor-specific antigens (e.g., carcinoembryonic antigen [CEA] and prostate-specific antigen [PSA]), which develop over time as the cells become less differentiated (mature). These proteins distinguish malignant cells from benign cells of the same tissue type.

An oncology nurse educator is providing health education to a patient who has been diagnosed with skin cancer. The patients wife has asked about the differences between normal cells and cancer cells. What characteristic of a cancer cell should the educator cite? A) Malignant cells contain more fibronectin than normal body cells. B) Malignant cells contain proteins called tumor-specific antigens. C) Chromosomes contained in cancer cells are more durable and stable than those of normal cells. D) The nuclei of cancer cells are unusually large, but regularly shaped.

B The cell membranes are altered in cancer cells, which affect fluid movement in and out of the cell. The cell membrane of malignant cells also contains proteins called tumor-specific antigens. Malignant cellular membranes also contain less fibronectin, a cellular cement. Typically, nuclei of cancer cells are large and irregularly shaped (pleomorphism). Fragility of chromosomes is commonly found when cancer cells are analyzed.

A patient has been diagnosed with advanced stage breast cancer and will soon begin aggressive treatment. What assessment findings would most strongly suggest that the patient may have developed liver metastases? A) Persistent fever and cognitive changes B) Abdominal pain and hepatomegaly C) Peripheral edema unresponsive to diuresis D) Spontaneous bleeding and jaundice

B The early manifestations of malignancy of the liver include paina continuous dull ache in the right upper quadrant, epigastrium, or back. Weight loss, loss of strength, anorexia, and anemia may also occur. The liver may be enlarged and irregular on palpation. Jaundice is present only if the larger bile ducts are occluded by the pressure of malignant nodules in the hilum of the liver. Fever, cognitive changes, peripheral edema, and bleeding are atypical signs.

The nurse is caring for a patient who is undergoing diagnostic testing for suspected malabsorption. When taking this patients health history and performing the physical assessment, the nurse should recognize what finding as most consistent with this diagnosis? A) Recurrent constipation coupled with weight loss B) Foul-smelling diarrhea that contains fat C) Fever accompanied by a rigid, tender abdomen D) Bloody bowel movements accompanied by fecal incontinence

B The hallmarks of malabsorption syndrome from any cause are diarrhea or frequent, loose, bulky, foul- smelling stools that have increased fat content and are often grayish (steatorrhea). Constipation and bloody bowel movements are not suggestive of malabsorption syndromes. Fever and a tender, rigid abdomen are associated with peritonitis.

A staff educator is reviewing the causes of gastroesophageal reflux disease (GERD) with new staff nurses. What area of the GI tract should the educator identify as the cause of reduced pressure associated with GERD? A) Pyloric sphincter B) Lower esophageal sphincter C) Hypopharyngeal sphincter D) Upper esophageal sphincter

B The lower esophageal sphincter, also called the gastroesophageal sphincter or cardiac sphincter, is located at the junction of the esophagus and the stomach. An incompetent lower esophageal sphincter allows reflux (backward flow) of gastric contents. The upper esophageal sphincter and the hypopharyngeal sphincter are synonymous and are not responsible for the manifestations of GERD. The pyloric sphincter exists between the stomach and the duodenum.

The ED nurse admitting a patient with a history of depression is screening the patient for suicide risk. What assessment question should the nurse ask when screening the patient? A) How would you describe your mood over the past few days? B) Have you ever thought about taking your own life? C) How do you think that your life is most likely to end? D) How would you rate the severity of your depression right now on a 10-point scale?

B The nurse should address the patients possible plans for suicide in a direct yet empathic manner. The nurse should avoid oblique or indirect references to suicide and should not limit questions to the patients depression.

An older adult who resides in an assisted living facility has sought care from the nurse because of recurrent episodes of constipation. Which of the following actions should the nurse first perform? A) Encourage the patient to take stool softener daily. B) Assess the patients food and fluid intake. C) Assess the patients surgical history. D) Encourage the patient to take fiber supplements.

B The nurse should follow the nursing process and perform an assessment prior to interventions. The patients food and fluid intake is more likely to affect bowel function than surgery.

A male patient with multiple injuries is brought to the ED by ambulance. He has had his airway stabilized and is breathing on his own. The ED nurse does not see any active bleeding, but should suspect internal hemorrhage based on what finding? A) Absence of bruising at contusion sites B) Rapid pulse and decreased capillary refill C) Increased BP with narrowed pulse pressure D) Sudden diaphoresis

B The nurse would anticipate that the pulse would increase and BP would decrease. Urine output would also decrease. An absence of bruising and the presence of diaphoresis would not suggest internal hemorrhage.

Resection of a patients bladder tumor has been incomplete and the patient is preparing for the administration of the first ordered instillation of topical chemotherapy. When preparing the patient, the nurse should emphasize the need to do which of the following? A) Remain NPO for 12 hours prior to the treatment. B) Hold the solution in the bladder for 2 hours before voiding. C) Drink the intravesical solution quickly and on an empty stomach. D) Avoid acidic foods and beverages until the full cycle of treatment is complete.

B The patient is allowed to eat and drink before the instillation procedure. Once the bladder is full, the patient must retain the intravesical solution for 2 hours before voiding. The solution is instilled through the meatus; it is not consumed orally. There is no need to avoid acidic foods and beverages during treatment.

A nurse is caring for a female patient whose urinary retention has not responded to conservative treatment. When educating this patient about self-catheterization, the nurse should encourage what practice? A) Assuming a supine position for self-catheterization B) Using clean technique at home to catheterize C) Inserting the catheter 1 to 2 inches into the urethra D) Self-catheterizing every 2 hours at home

B The patient may use a clean (nonsterile) technique at home, where the risk of cross-contamination is reduced. The average daytime clean intermittent catheterization schedule is every 4 to 6 hours and just before bedtime. The female patient assumes a Fowlers position and uses a mirror to help locate the urinary meatus. The nurse teaches her to catheterize herself by inserting a catheter 7.5 cm (3 inches) into the urethra, in a downward and backward direction.

A nurse in an oral surgery practice is working with a patient scheduled for removal of an abscessed tooth. When providing discharge education, the nurse should recommend which of the following actions? A) Rinse the mouth with alcohol before bedtime for the next 7 days. B) Use warm saline to rinse the mouth as needed. C) Brush around the area with a firm toothbrush to prevent infection. D) Use a toothpick to dislodge any debris that gets lodged in the socket.

B The patient should be assessed for bleeding after the tooth is extracted. The mouth can be rinsed with warm saline to keep the area clean. A firm toothbrush or toothpick could injure the tissues around the extracted area. Alcohol would injure tissues that are healing.

A 52-year-old patient is scheduled to undergo ileal conduit surgery. When planning this patients discharge education, what is the most plausible nursing diagnosis that the nurse should address? A) Impaired mobility related to limitations posed by the ileal conduit B) Deficient knowledge related to care of the ileal conduit C) Risk for deficient fluid volume related to urinary diversion D) Risk for autonomic dysreflexia related to disruption of the sacral plexus

B The patient will most likely require extensive teaching about the care and maintenance of a new urinary diversion. A diversion does not create a serious risk of fluid volume deficit. Mobility is unlikely to be impaired after the immediate postsurgical recovery. The sacral plexus is not threatened by the creation of a urinary diversion.

Which adult will the nurse plan to teach about risks associated with obesity? A) Man who has a BMI of 18 kg/m2 B) Man with a 42 in waist and 44 in hips C) Woman who has a body mass index (BMI) of 24 kg/m2 D) Woman with a waist circumference of 34 inches (86 cm)

B The waist-to-hip ratio for this patient is 0.95, which exceeds the recommended level of <0.80. A patient with a BMI of 18 kg/m2 is considered underweight. A BMI of 24 kg/m2 is normal. Health risks associated with obesity increase in women with a waist circumference larger than 35 in (89 cm) and men with a waist circumference larger than 40 in (102 cm).

A patient with a history of injection drug use has been diagnosed with hepatitis C. When collaborating with the care team to plan this patients treatment, the nurse should anticipate what intervention? A) Administration of immune globulins B) A regimen of antiviral medications C) Rest and watchful waiting D) Administration of fresh-frozen plasma (FFP)

B There is no benefit from rest, diet, or vitamin supplements in HCV treatment. Studies have demonstrated that a combination of two antiviral agents, Peg-interferon and ribavirin (Rebetol), is effective in producing improvement in patients with hepatitis C and in treating relapses. Immune globulins and FFP are not indicated.

After the nurse teaches a patient about the recommended amounts of foods from animal and plant sources, which menu selections indicate that the initial instructions about diet have been understood? A) 3 oz of lean beef, 2 oz of low-fat cheese, and a tomato slice B) 3 oz of roasted pork, a cup of corn, and a cup of carrot sticks C) Cup of tossed salad and nonfat dressing topped with a chicken breast D) Half cup of tuna mixed with nonfat mayonnaise and a half cup of celery

B This selection is most consistent with the recommendation of the American Institute for Cancer Research that one third of the diet should be from animal sources and two thirds from plant source foods. The other choices all have higher ratios of animal origin foods to plant source foods than would be recommended.

After bariatric surgery, a patient who is being discharged tells the nurse, I prefer to be independent. I am not interested in any support groups. Which response by the nurse is best? A) I hope you change your mind so that I can suggest a group for you. B) Tell me what types of resources you think you might use after this surgery. C) Support groups have been found to lead to more successful weight loss after surgery. D) Because there are many lifestyle changes after surgery, we recommend support groups.

B This statement allows the nurse to assess the individual patients potential needs and preferences. The other statements offer the patient more information about the benefits of support groups, but fail to acknowledge the patients preferences.

A patient is admitted to the unit with acute cholecystitis. The physician has noted that surgery will be scheduled in 4 days. The patient asks why the surgery is being put off for a week when he has a sick gallbladder. What rationale would underlie the nurses response? A) Surgery is delayed until the patient can eat a regular diet without vomiting. B) Surgery is delayed until the acute symptoms subside. C) The patient requires aggressive nutritional support prior to surgery. D) Time is needed to determine whether a laparoscopic procedure can be used.

B Unless the patients condition deteriorates, surgical intervention is delayed just until the acute symptoms subside (usually within a few days). There is no need to delay surgery pending an improvement in nutritional status, and deciding on a laparoscopic approach is not a lengthy process.

The critical care nurse is monitoring the patients urine output and drains following renal surgery. What should the nurse promptly report to the physician? A) Increased pain on movement B) Absence of drain output C) Increased urine output D) Blood-tinged serosanguineous drain output

B Urine output and drainage from tubes inserted during surgery are monitored for amount, color, and type or characteristics. Decreased or absent drainage is promptly reported to the physician because it may indicate obstruction that could cause pain, infection, and disruption of the suture lines. Reporting increased pain on movement has nothing to do with the scenario described. Increased urine output and serosanguineous drainage are expected.

A patient with gallstones has been prescribed ursodeoxycholic acid (UDCA). The nurse understands that additional teaching is needed regarding this medication when the patient states: A) It is important that I see my physician for scheduled follow-up appointments while taking this medication. B) I will take this medication for 2 weeks and then gradually stop taking it. C) If I lose weight, the dose of the medication may need to be changed. D) This medication will help dissolve small gallstones made of cholesterol.

B Ursodeoxycholic acid (UDCA) has been used to dissolve small, radiolucent gallstones composed primarily of cholesterol. This drug can reduce the size of existing stones, dissolve small stones, and prevent new stones from forming. Six to 12 months of therapy is required in many patients to dissolve stones, and monitoring of the patient is required during this time. The effective dose of medication depends on body weight.

An ED nurse is triaging patients according to the Emergency Severity Index (ESI). When assigning patients to a triage level, the nurse will consider the patients acuity as well as what other variable? A) The likelihood of a repeat visit to the ED in the next 7 days B) The resources that the patient is likely to require C) The patients or insurers ability to pay for care D) Whether the patient is known to ED staff from previous visits

B With the ESI, patients are assigned to triage levels based on both their acuity and their anticipated resource needs. Ability to pay, the likelihood of repeat visits, and the history of prior visits are not explicitly considered.

A patient is admitted to the ED complaining of abdominal pain. Further assessment of the abdomen reveals signs of peritoneal irritation. What assessment findings would corroborate this diagnosis? Select all that apply. A) Ascites B) Rebound tenderness C) Changes in bowel sounds D) Muscular rigidity E) Copious diarrhea

B, C, D Signs of peritoneal irritation include abdominal distention, involuntary guarding, tenderness, pain, muscular rigidity, or rebound tenderness along with changes in bowel sounds. Diarrhea and ascites are not signs of peritoneal irritation.

A nurse has entered the room of a patient with cirrhosis and found the patient on the floor. The patient states that she fell when transferring to the commode. The patients vital signs are within reference ranges and the nurse observes no apparent injuries. What is the nurses most appropriate action? A) Remove the patients commode and supply a bedpan. B) Complete an incident report and submit it to the unit supervisor. C) Have the patient assessed by the physician due to the risk of internal bleeding. D) Perform a focused abdominal assessment in order to rule out injury.

C A fall would necessitate thorough medical assessment due to the patients risk of bleeding. The nurses abdominal assessment is an appropriate action, but is not wholly sufficient to rule out internal injury. Medical assessment is a priority over removing the commode or filling out an incident report, even though these actions are appropriate.

A nurse is assessing a patients stoma on postoperative day 3. The nurse notes that the stoma has a shiny appearance and a bright red color. How should the nurse best respond to this assessment finding? A) Irrigate the ostomy to clear a possible obstruction. B) Contact the primary care provider to report this finding. C) Document that the stoma appears healthy and well perfused. D) Document a nursing diagnosis of Impaired Skin Integrity.

C A healthy, viable stoma should be shiny and pink to bright red. This finding does not indicate that the stoma is blocked or that skin integrity is compromised.

A patient is brought to the ED by ambulance after swallowing highly acidic toilet bowl cleaner 2 hours earlier. The patient is alert and oriented. What is the care teams most appropriate treatment? A) Administering syrup of ipecac B) Performing a gastric lavage C) Giving milk to drink D) Referring to psychiatry

C A patient who has swallowed an acidic substance, such as toilet bowl cleaner, may be given milk or water to drink for dilution. Gastric lavage must be performed within 1 hour of ingestion. A psychiatric consult may be considered once the patient is physically stable and it is deemed appropriate by the physician. Syrup of ipecac is no longer used in clinical settings.

The nurse on a bone marrow transplant unit is caring for a patient with cancer who is preparing for HSCT. What is a priority nursing diagnosis for this patient? A) Fatigue related to altered metabolic processes B) Altered nutrition: less than body requirements related to anorexia C) Risk for infection related to altered immunologic response D) Body image disturbance related to weight loss and anorexia

C A priority nursing diagnosis for this patient is risk for infection related to altered immunologic response. Because the patients immunity is suppressed, he or she will be at a high risk for infection. The other listed nursing diagnoses are valid, but they are not as high a priority as is risk for infection.

A patient who underwent surgery for esophageal cancer is admitted to the critical care unit following postanesthetic recovery. Which of the following should be included in the patients immediate postoperative plan of care? A) Teaching the patient to self-suction B) Performing chest physiotherapy to promote oxygenation C) Positioning the patient to prevent gastric reflux D) Providing a regular diet as tolerated

C After recovering from the effects of anesthesia, the patient is placed in a low Fowlers position, and later in a Fowlers position, to help prevent reflux of gastric secretions. The patient is observed carefully for regurgitation and dyspnea because a common postoperative complication is aspiration pneumonia. In this period of recovery, self-suctioning is also not likely realistic or safe. Chest physiotherapy is contraindicated because of the risk of aspiration. Nutrition is prioritized, but a regular diet is contraindicated in the immediate recovery from esophageal surgery.

A patient has undergone surgery for oral cancer and has just been extubated in postanesthetic recovery. What nursing action best promotes comfort and facilitates spontaneous breathing for this patient? A) Placing the patient in a left lateral position B) Administering opioids as ordered C) Placing the patient in Fowlers position D) Teaching the patient to use the patient-controlled analgesia (PCA) system

C After the endotracheal tube or airway has been removed and the effects of the anesthesia have worn off, the patient may be placed in Fowlers position to facilitate breathing and promote comfort. Lateral positioning does not facilitate oxygenation or comfort. Medications do not facilitate spontaneous breathing.

A client just returned to the surgical unit after a gastric bypass. What action by the nurse is the priority? A) Assess the clients pain. B) Check the surgical incision. C) Ensure an adequate airway. D) Program the morphine pump.

C All actions are appropriate care measures for this client; however, airway is always the priority. Bariatric clients tend to have short, thick necks that complicate airway management.

A patient is brought to the ED by friends. The friends tell the nurse that the patient was using cocaine at a party. On arrival to the ED the patient is in visible distress with an axillary temperature of 40.1C (104.2F). What would be the priority nursing action for this patient? A) Monitor cardiovascular effects. B) Administer antipyretics. C) Ensure airway and ventilation. D) Prevent seizure activity.

C Although all of the listed actions may be necessary for this patients care, the priority is to establish a patent airway and adequate ventilation.

A patients screening colonoscopy revealed the presence of numerous polyps in the large bowel. What principle should guide the subsequent treatment of this patients health problem? A) Adherence to a high-fiber diet will help the polyps resolve. B) The patient should be assured that these are a normal, age-related physiologic change. C) The patients polyps constitute a risk factor for cancer. D) The presence of polyps is associated with an increased risk of bowel obstruction.

C Although most polyps do not develop into invasive neoplasms, they must be identified and followed closely. They are very common, but are not classified as a normal, age-related physiologic change. Diet will not help them resolve and they do not typically lead to obstructions.

A patient has undergone a laparoscopic cholecystectomy and is being prepared for discharge home. When providing health education, the nurse should prioritize which of the following topics? A) Management of fluid balance in the home setting B) The need for blood glucose monitoring for the next week C) Signs and symptoms of intra-abdominal complications D) Appropriate use of prescribed pancreatic enzymes

C Because of the early discharge following laparoscopic cholecystectomy, the patient needs thorough education in the signs and symptoms of complications. Fluid balance is not typically a problem in the recovery period after laparoscopic cholecystectomy. There is no need for blood glucose monitoring or pancreatic enzymes.

An older adult has a diagnosis of Alzheimers disease and has recently been experiencing fecal incontinence. However, the nurse has observed no recent change in the character of the patients stools. What is the nurses most appropriate intervention? A) Keep a food diary to determine the foods that exacerbate the patients symptoms. B) Provide the patient with a bland, low-residue diet. C) Toilet the patient on a frequent, scheduled basis. D) Liaise with the primary care provider to obtain an order for loperamide.

C Because the patients fecal incontinence is most likely attributable to cognitive decline, frequent toileting is an appropriate intervention. Loperamide is unnecessary in the absence of diarrhea. Specific foods are not likely to be a cause of, or solution to, this patients health problem.

The nurse is caring for a patient after kidney surgery. The nurse is aware that bleeding is a major complication of kidney surgery and that if it goes undetected and untreated can result in hypovolemia and hemorrhagic shock in the patient. When assessing for bleeding, what assessment parameter should the nurse evaluate? A) Oral intake B) Pain intensity C) Level of consciousness D) Radiation of pain

C Bleeding is a major complication of kidney surgery. If undetected and untreated, this can result in hypovolemia and hemorrhagic shock. The nurses role is to observe for these complications, to report their signs and symptoms, and to administer prescribed parenteral fluids and blood and blood components. Monitoring of vital signs, skin condition, the urinary drainage system, the surgical incision, and the level of consciousness is necessary to detect evidence of bleeding, decreased circulating blood, and fluid volume and cardiac output. Bleeding is not normally evidenced by changes in pain or oral intake.

A patients health history is suggestive of inflammatory bowel disease. Which of the following would suggest Crohns disease, rather that ulcerative colitis, as the cause of the patients signs and symptoms? A) A pattern of distinct exacerbations and remissions B) Severe diarrhea C) An absence of blood in stool D) Involvement of the rectal mucosa

C Bloody stool is far more common in cases of UC than in Crohns. Rectal involvement is nearly 100% in cases of UC (versus 20% in Crohns) and patients with UC typically experience severe diarrhea. UC is also characterized by a pattern of remissions and exacerbations, while Crohns often has a more prolonged and variable course.

A patient has just been diagnosed with chronic pancreatitis. The patient is underweight and in severe pain and diagnostic testing indicates that over 80% of the patients pancreas has been destroyed. The patient asks the nurse why the diagnosis was not made earlier in the disease process. What would be the nurses best response? A) The symptoms of pancreatitis mimic those of much less serious illnesses. B) Your body doesnt require pancreatic function until it is under great stress, so it is easy to go unnoticed. C) Chronic pancreatitis often goes undetected until a large majority of pancreatic function is lost. D) Its likely that your other organs were compensating for your decreased pancreatic function.

C By the time symptoms occur in chronic pancreatitis, approximately 90% of normal acinar cell function (exocrine function) has been lost. Late detection is not usually attributable to the vagueness of symptoms. The pancreas contributes continually to homeostasis and other organs are unable to perform its physiologic functions.

A patient is admitted to the ICU after a motor vehicle accident. On the second day of the hospital admission, the patient develops acute kidney injury. The patient is hemodynamically unstable, but renal replacement therapy is needed to manage the patients hypervolemia and hyperkalemia. Which of the following therapies will the patients hemodynamic status best tolerate? A) Hemodialysis B) Peritoneal dialysis C) Continuous venovenous hemodialysis (CVVHD) D) Plasmapheresis

C CVVHD facilitates the removal of uremic toxins and fluid. The hemodynamic effects of CVVHD are usually mild in comparison to hemodialysis, so CVVHD is best tolerated by an unstable patient. Peritoneal dialysis is not the best choice, as the patient may have sustained abdominal injuries during the accident and catheter placement would be risky. Plasmapheresis does not achieve fluid removal and electrolyte balance.

A community health nurse is caring for a patient whose multiple health problems include chronic pancreatitis. During the most recent visit, the nurse notes that the patient is experiencing severe abdominal pain and has vomited 3 times in the past several hours. What is the nurses most appropriate action? A) Administer a PRN dose of pancreatic enzymes as ordered. B) Teach the patient about the importance of abstaining from alcohol. C) Arrange for the patient to be transported to the hospital. D) Insert an NG tube, if available, and stay with the patient.

C Chronic pancreatitis is characterized by recurring attacks of severe upper abdominal and back pain, accompanied by vomiting. The onset of these acute symptoms warrants hospital treatment. Pancreatic enzymes are not indicated and an NG tube would not be inserted in the home setting. Patient education is a later priority that may or may not be relevant.

A patient is admitted to the ICU with acute pancreatitis. The patients family asks what causes acute pancreatitis. The critical care nurse knows that a majority of patients with acute pancreatitis have what? A) Type 1 diabetes B) An impaired immune system C) Undiagnosed chronic pancreatitis D) An amylase deficiency

C Eighty percent of patients with acute pancreatitis have biliary tract disease or a history of long-term alcohol abuse. These patients usually have had undiagnosed chronic pancreatitis before their first episode of acute pancreatitis. Diabetes, an impaired immune function, and amylase deficiency are not specific precursors to acute pancreatitis.

A patient with cancer of the tongue has had a radical neck dissection. What nursing assessment would be a priority for this patient? A) Presence of acute pain and anxiety B) Tissue integrity and color of the operative site C) Respiratory status and airway clearance D) Self-esteem and body image

C Postoperatively, the patient is assessed for complications such as altered respiratory status, wound infection, and hemorrhage. The other assessments are part of the plan of care for a patient who has had a radical neck dissection, but are not the nurses chief priority.

A community health nurse serves a diverse population. What individual would likely face the highest risk for parotitis? A) A patient who is receiving intravenous antibiotic therapy in the home setting B) A patient who has a chronic venous ulcer C) An older adult whose medication regimen includes an anticholinergic D) A patient with poorly controlled diabetes who receives weekly wound care

C Elderly, acutely ill, or debilitated people with decreased salivary flow from general dehydration or medications are at high risk for parotitis. Anticholinergic medications inhibit saliva production. Antibiotics, diabetes, and wounds are not risk factors for parotitis.

A nurse is planning discharge teaching for a 21-year-old patient with a new diagnosis of ulcerative colitis. When planning family assessment, the nurse should recognize that which of the following factors will likely have the greatest impact on the patients coping after discharge? A) The familys ability to take care of the patients special diet needs B) The familys ability to monitor the patients changing health status C) The familys ability to provide emotional support D) The familys ability to manage the patients medication regimen

C Emotional support from the family is key to the patients coping after discharge. A 21-year-old would be expected to self-manage the prescribed medication regimen and the family would not be primarily responsible for monitoring the patients health status. It is highly beneficial if the family is willing and able to accommodate the patients dietary needs, but emotional support is paramount and cannot be solely provided by the patient alone.

A patient is experiencing respiratory insufficiency and cannot maintain spontaneous respirations. The nurse suspects that the physician will perform which of the following actions? A) Insert an oropharyngeal airway. B) Perform the jaw thrust maneuver. C) Perform endotracheal intubation. D) Perform a cricothyroidotomy.

C Endotracheal tubes are used in cases when the patient cannot be ventilated with an oropharyngeal airway, which is used in patients who are breathing spontaneously. The jaw thrust maneuver does not establish an airway and cricothyroidotomy would be performed as a last resort.

A nurse who provides care in a walk-in clinic assesses a wide range of individuals. The nurse should identify which of the following patients as having the highest risk for chronic pancreatitis? A) A 45-year-old obese woman with a high-fat diet B) An 18-year-old man who is a weekend binge drinker C) A 39-year-old man with chronic alcoholism D) A 51-year-old woman who smokes one-and-a-half packs of cigarettes per day

C Excessive and prolonged consumption of alcohol accounts for approximately 70% to 80% of all cases of chronic pancreatitis.

A nurse is preparing to discharge a patient after recovery from gastric surgery. What is an appropriate discharge outcome for this patient? A) The patients bowel movements maintain a loose consistency. B) The patient is able to tolerate three large meals a day. C) The patient maintains or gains weight. D) The patient consumes a diet high in calcium.

C Expected outcomes for the patient following gastric surgery include ensuring that the patient is maintaining or gaining weight (patient should be weighed daily), experiencing no excessive diarrhea, and tolerating six small meals a day. Patients may require vitamin B12 supplementation by the intramuscular route and do not require a diet excessively rich in calcium.

A patient who has recently undergone ESWL for the treatment of renal calculi has phoned the urology unit where he was treated, telling the nurse that he has a temperature of 101.1F (38.4C). How should the nurse best respond to the patient? A) Remind the patient that renal calculi have a noninfectious etiology and that a fever is unrelated to their recurrence. B) Remind the patient that occasional febrile episodes are expected following ESWL. C) Tell the patient to report to the ED for further assessment. D) Tell the patient to monitor his temperature for the next 24 hours and then contact his urologists office.

C Following ESWL, the development of a fever is abnormal and is suggestive of a UTI; prompt medical assessment and treatment are warranted. It would be inappropriate to delay further treatment.

A nurse is providing health promotion education to a patient diagnosed with an esophageal reflux disorder. What practice should the nurse encourage the patient to implement? A) Keep the head of the bed lowered. B) Drink a cup of hot tea before bedtime. C) Avoid carbonated drinks. D) Eat a low-protein diet.

C For a patient diagnosed with esophageal reflux disorder, the nurse should instruct the patient to keep the head of the bed elevated. Carbonated drinks, caffeine, and tobacco should be avoided. Protein limitation is not necessary.

The school nurse is teaching a nutrition class in the local high school. One student states that he has heard that certain foods can increase the incidence of cancer. The nurse responds, Research has shown that certain foods indeed appear to increase the risk of cancer. Which of the following menu selections would be the best choice for potentially reducing the risks of cancer? A) Smoked salmon and green beans B) Pork chops and fried green tomatoes C) Baked apricot chicken and steamed broccoli D) Liver, onions, and steamed peas

C Fruits and vegetables appear to reduce cancer risk. Salt-cured foods, such as ham and processed meats, as well as red meats, should be limited.

A nurse is working with a patient who has chronic constipation. What should be included in patient teaching to promote normal bowel function? A) Use glycerin suppositories on a regular basis. B) Limit physical activity in order to promote bowel peristalsis. C) Consume high-residue, high-fiber foods. D) Resist the urge to defecate until the urge becomes intense.

C Goals for the patient include restoring or maintaining a regular pattern of elimination by responding to the urge to defecate, ensuring adequate intake of fluids and high-fiber foods, learning about methods to avoid constipation, relieving anxiety about bowel elimination patterns, and avoiding complications. Ongoing use of pharmacologic aids should not be promoted, due to the risk of dependence. Increased mobility helps to maintain a regular pattern of elimination. The urge to defecate should be heeded.

The nurse is caring for a patient in acute kidney injury. Which of the following complications would most clearly warrant the administration of polystyrene sulfonate (Kayexalate)? A) Hypernatremia B) Hypomagnesemia C) Hyperkalemia D) Hypercalcemia

C Hyperkalemia, a common complication of acute kidney injury, is life-threatening if immediate action is not taken to reverse it. The administration of polystyrene sulfonate reduces serum potassium levels.

A patients colorectal cancer has necessitated a hemicolectomy with the creation of a colostomy. In the 4 days since the surgery, the patient has been unwilling to look at the ostomy or participate in any aspects of ostomy care. What is the nurses most appropriate response to this observation? A) Ensure that the patient knows that he or she will be responsible for care after discharge. B) Reassure the patient that many people are fearful after the creation of an ostomy. C) Acknowledge the patients reluctance and initiate discussion of the factors underlying it. D) Arrange for the patient to be seen by a social worker or spiritual advisor.

C If the patient is reluctant to participate in ostomy care, the nurse should attempt to dialogue about this with the patient and explore the factors that underlie it. It is presumptive to assume that the patients behavior is motivated by fear. Assessment must precede referrals and emphasizing the patients responsibilities may or may not motivate the patient.

A patient has had her indwelling urinary catheter removed after having it in place for 10 days during recovery from an acute illness. Two hours after removal of the catheter, the patient informs the nurse that she is experiencing urinary urgency resulting in several small-volume voids. What is the nurses best response? A) Inform the patient that urgency and occasional incontinence are expected for the first few weeks post-removal. B) Obtain an order for a loop diuretic in order to enhance urine output and bladder function. C) Inform the patient that this is not unexpected in the short term and scan the patients bladder following each void. D) Obtain an order to reinsert the patients urinary catheter and attempt removal in 24 to 48 hours.

C Immediately after the indwelling catheter is removed, the patient is placed on a timed voiding schedule, usually every 2 to 3 hours. At the given time interval, the patient is instructed to void. The bladder is then scanned using a portable ultrasonic bladder scanner; if the bladder has not emptied completely, straight catheterization may be performed. An indwelling catheter would not be reinserted to resolve the problem and diuretics would not be beneficial. Ongoing incontinence is not an expected finding after catheter removal.

A student nurse is caring for a patient who has a diagnosis of acute pancreatitis and who is receiving parenteral nutrition. The student should prioritize which of the following assessments? A) Fluid output B) Oral intake C) Blood glucose levels D) BUN and creatinine levels

C In addition to administering enteral or parenteral nutrition, the nurse monitors serum glucose levels every 4 to 6 hours. Output should be monitored but in most cases it is not more important than serum glucose levels. A patient on parenteral nutrition would have no oral intake to monitor. Blood sugar levels are more likely to be unstable than indicators of renal function.

The family of a patient in the ICU diagnosed with acute pancreatitis asks the nurse why the patient has been moved to an air bed. What would be the nurses best response? A) Air beds allow the care team to reposition her more easily while shes on bed rest. B) Air beds are far more comfortable than regular beds and shell likely have to be on bed rest a long time. C) The bed automatically moves, so shes less likely to develop pressure sores while shes in bed. D) The bed automatically moves, so she is likely to have less pain.

C It is important to turn the patient every 2 hours; use of specialty beds may be indicated to prevent skin breakdown. The rationale for a specialty bed is not related to repositioning, comfort, or ease of movement.

A previously healthy adults sudden and precipitous decline in health has been attributed to fulminant hepatic failure, and the patient has been admitted to the intensive care unit. The nurse should be aware that the treatment of choice for this patient is what? A) IV administration of immune globulins B) Transfusion of packed red blood cells and fresh-frozen plasma (FFP) C) Liver transplantation D) Lobectomy

C Liver transplantation carries the highest potential for the resolution of fulminant hepatic failure. This is preferred over other interventions, such as pharmacologic treatments, transfusions, and surgery.

Which statement by the nurse is most likely to help a morbidly obese 22-year-old man in losing weight on a 1000-calorie diet? A) It will be necessary to change lifestyle habits permanently to maintain weight loss. B) You will decrease your risk for future health problems such as diabetes by losing weight now. C) You are likely to notice changes in how you feel with just a few weeks of diet and exercise. D) Most of the weight that you lose during the first weeks of dieting is water weight rather than fat.

C Motivation is a key factor in successful weight loss and a short-term outcome provides a higher motivation. A 22-year-old patient is unlikely to be motivated by future health problems. Telling a patient that the initial weight loss is water will be discouraging, although this may be correct. Changing lifestyle habits is necessary, but this process occurs over time and discussing this is not likely to motivate the patient.

A patient with portal hypertension has been admitted to the medical floor. The nurse should prioritize which of the following assessments related to the manifestations of this health problem? A) Assessment of blood pressure and assessment for headaches and visual changes B) Assessments for signs and symptoms of venous thromboembolism C) Daily weights and abdominal girth measurement D) Blood glucose monitoring q4h

C Obstruction to blood flow through the damaged liver results in increased blood pressure (portal hypertension) throughout the portal venous system. This can result in varices and ascites in the abdominal cavity. Assessments related to ascites are daily weights and abdominal girths. Portal hypertension is not synonymous with cardiovascular hypertension and does not create a risk for unstable blood glucose or VTE.

The nurse is caring for acutely ill patient. What assessment finding should prompt the nurse to inform the physician that the patient may be exhibiting signs of acute kidney injury (AKI)? A) The patient is complains of an inability to initiate voiding. B) The patients urine is cloudy with a foul odor. C) The patients average urine output has been 10 mL/hr for several hours. D) The patient complains of acute flank pain.

C Oliguria (<500 mL/d of urine) is the most common clinical situation seen in AKI. Flank pain and inability to initiate voiding are not characteristic of AKI. Cloudy, foul-smelling urine is suggestive of a urinary tract infection.

A patient is receiving care in the intensive care unit for acute pancreatitis. The nurse is aware that pancreatic necrosis is a major cause of morbidity and mortality in patients with acute pancreatitis. Consequently, the nurse should assess for what signs or symptoms of this complication? A) Sudden increase in random blood glucose readings B) Increased abdominal girth accompanied by decreased level of consciousness C) Fever, increased heart rate and decreased blood pressure D) Abdominal pain unresponsive to analgesics

C Pancreatic necrosis is a major cause of morbidity and mortality in patients with acute pancreatitis because of resulting hemorrhage, septic shock, and multiple organ dysfunction syndrome (MODS). Signs of shock would include hypotension, tachycardia and fever. Each of the other listed changes in status warrants intervention, but none is clearly suggestive of an onset of pancreatic necrosis.

A nurse is caring for a patient with hepatic encephalopathy. The nurses assessment reveals that the patient exhibits episodes of confusion, is difficult to arouse from sleep and has rigid extremities. Based on these clinical findings, the nurse should document what stage of hepatic encephalopathy? A) Stage 1 B) Stage 2 C) Stage 3 D) Stage 4

C Patients in the third stage of hepatic encephalopathy exhibit the following symptoms: stuporous, difficult to arouse, sleeps most of the time, exhibits marked confusion, incoherent in speech, asterixis, increased deep tendon reflexes, rigidity of extremities, marked EEG abnormalities. Patients in stages 1 and 2 exhibit clinical symptoms that are not as advanced as found in stage 3, and patients in stage 4 are comatose. In stage 4, there is an absence of asterixis, absence of deep tendon reflexes, flaccidity of extremities, and EEG abnormalities.

A patient with liver disease has developed jaundice; the nurse is collaborating with the patient to develop a nutritional plan. The nurse should prioritize which of the following in the patients plan? A) Increased potassium intake B) Fluid restriction to 2 L per day C) Reduction in sodium intake D) High-protein, low-fat diet

C Patients with ascites require a sharp reduction in sodium intake. Potassium intake should not be correspondingly increased. There is no need for fluid restriction or increased protein intake.

A patient on the oncology unit is receiving carmustine, a chemotherapy agent, and the nurse is aware that a significant side effect of this medication is thrombocytopenia. Which symptom should the nurse assess for in patients at risk for thrombocytopenia? A) Interrupted sleep pattern B) Hot flashes C) Epistaxis (nose bleed) D) Increased weight

C Patients with thrombocytopenia are at risk for bleeding due to decreased platelet counts. Patients with thrombocytopenia do not exhibit interrupted sleep pattern, hot flashes, or increased weight.

A nurse is assessing a patient who has just been admitted to the postsurgical unit following surgical resection for the treatment of oropharyngeal cancer. What assessment should the nurse prioritize? A) Assess ability to clear oral secretions. B) Assess for signs of infection. C) Assess for a patent airway. D) Assess for ability to communicate.

C Postoperatively, the nurse assesses for a patent airway. The patients ability to manage secretions has a direct bearing on airway patency. However, airway patency is the overarching goal. This immediate physiologic need is prioritized over communication, though this is an important consideration. Infection is not normally a threat in the immediate postoperative period.

An oncology patient has just returned from the postanesthesia care unit after an open hemicolectomy. This patients plan of nursing care should prioritize which of the following? A) Assess the patient hourly for signs of compartment syndrome. B) Assess the patients fine motor skills once per shift. C) Assess the patients wound for dehiscence every 4 hours. D) Maintain the patients head of bed at 45 degrees or more at all times.

C Postoperatively, the nurse assesses the patients responses to the surgery and monitors the patient for possible complications, such as infection, bleeding, thrombophlebitis, wound dehiscence, fluid and electrolyte imbalance, and organ dysfunction. Fine motor skills are unlikely to be affected by surgery and compartment syndrome is a complication of fracture casting, not abdominal surgery. There is no need to maintain a high head of bed.

Traditionally, nurses have been involved with tertiary cancer prevention. However, an increasing emphasis is being placed on both primary and secondary prevention. What would be an example of primary prevention? A) Yearly Pap tests B) Testicular self-examination C) Teaching patients to wear sunscreen D) Screening mammograms

C Primary prevention is concerned with reducing the risks of cancer in healthy people through practices such as use of sunscreen. Secondary prevention involves detection and screening to achieve early diagnosis, as demonstrated by Pap tests, mammograms, and testicular exams.

A nurse is caring for a patient with severe hemolytic jaundice. Laboratory tests show free bilirubin to be 24 mg/dL. For what complication is this patient at risk? A) Chronic jaundice B) Pigment stones in portal circulation C) Central nervous system damage D) Hepatomegaly

C Prolonged jaundice, even if mild, predisposes to the formation of pigment stones in the gallbladder, and extremely severe jaundice (levels of free bilirubin exceeding 20 to 25 mg/dL) poses a risk for CNS damage. There are not specific risks of hepatomegaly or chronic jaundice resulting from high bilirubin.

The nurse is caring for a 39-year-old woman with a family history of breast cancer. She requested a breast tumor marking test and the results have come back positive. As a result, the patient is requesting a bilateral mastectomy. This surgery is an example of what type of oncologic surgery? A) Salvage surgery B) Palliative surgery C) Prophylactic surgery D) Reconstructive surgery

C Prophylactic surgery is used when there is an extensive family history and nonvital tissues are removed. Salvage surgery is an additional treatment option that uses an extensive surgical approach to treat the local recurrence of a cancer after the use of a less extensive primary approach. Palliative surgery is performed in an attempt to relieve complications of cancer, such as ulceration, obstruction, hemorrhage, pain, and malignant effusion. Reconstructive surgery may follow curative or radical surgery in an attempt to improve function or obtain a more desirable cosmetic effect.

A few months after bariatric surgery, a 56-year-old man tells the nurse, My skin is hanging in folds. I think I need cosmetic surgery. Which response by the nurse is most appropriate? A) The important thing is that you are improving your health. B) The skinfolds will disappear once most of the weight is lost. C) Cosmetic surgery is a possibility once your weight has stabilized. D) Perhaps you would like to talk to a counselor about your body image.

C Reconstructive surgery may be used to eliminate excess skinfolds after at least a year has passed since the surgery. Skinfolds may not disappear over time, especially in older patients. The response, The important thing is that your weight loss is improving your health, ignores the patients concerns about appearance and implies that the nurse knows what is important. Whereas it may be helpful for the patient to talk to a counselor, it is more likely to be helpful to know that cosmetic surgery is available.

A nurse is working with a female patient who has developed stress urinary incontinence. Pelvic floor muscle exercises have been prescribed by the primary care provider. How can the nurse best promote successful treatment? A) Clearly explain the potential benefits of pelvic floor muscle exercises. B) Ensure the patient knows that surgery will be required if the exercises are unsuccessful. C) Arrange for biofeedback when the patient is learning to perform the exercises. D) Contact the patient weekly to ensure that she is performing the exercises consistently.

C Research shows that written or verbal instruction alone is usually inadequate to teach an individual how to identify and strengthen the pelvic floor for sufficient bladder and bowel control. Biofeedback-assisted pelvic muscle exercise (PME) uses either electromyography or manometry to help the individual identify the pelvic muscles as he or she attempts to learn which muscle group is involved when performing PME. This objective assessment is likely superior to weekly contact with the patient. Surgery is not necessarily indicated if behavioral techniques are unsuccessful.

A public health nurse has formed an interdisciplinary team that is developing an educational program entitled Cancer: The Risks and What You Can Do About Them. Participants will receive information, but the major focus will be screening for relevant cancers. This program is an example of what type of health promotion activity? A) Disease prophylaxis B) Risk reduction C) Secondary prevention D) Tertiary prevention

C Secondary prevention involves screening and early detection activities that seek to identify early stage cancer in individuals who lack signs and symptoms suggestive of cancer. Primary prevention is concerned with reducing the risks of disease through health promotion strategies. Tertiary prevention is the care and rehabilitation of the patient after having been diagnosed with cancer.

A patient who has had a radical neck dissection is being prepared for discharge. The discharge plan includes referral to an outpatient rehabilitation center for physical therapy. What would the goals of physical therapy for this patient include? A) Muscle training to relieve dysphagia B) Relieving nerve paralysis in the cervical plexus C) Promoting maximum shoulder function D) Alleviating achalasia by decreasing esophageal peristalsis

C Shoulder drop occurs as a result of radical neck dissection. Shoulder function can be improved by rehabilitation exercises. Rehabilitation would not be initiated until the patients neck incision and graft, if present, were sufficiently healed. Nerve paralysis in the cervical plexus and other variables affecting swallowing would be managed by a speech therapist rather than a physical therapist.

A patient has a glomerular filtration rate (GFR) of 43 mL/min/1.73 m2. Based on this GFR, the nurse interprets that the patients chronic kidney disease is at what stage? A) stage 1 B) stage 2 C) stage 3 D) stage 4

C Stages of chronic renal failure are based on the GFR. Stage 3 is defined by a GFR in the range of 30 to 259 mL/min/1.73 m . This is considered a moderate decrease in GFR

A 58-year-old male patient has been hospitalized for a wedge resection of the left lower lung lobe after a routine chest x-ray shows carcinoma. The patient is anxious and asks if he can smoke. Which statement by the nurse would be most therapeutic? A) Smoking is the reason you are here. B) The doctor left orders for you not to smoke. C) You are anxious about the surgery. Do you see smoking as helping? D) Smoking is OK right now, but after your surgery it is contraindicated.

C Stating You are anxious about the surgery. Do you see smoking as helping? acknowledges the patients feelings and encourages him to assess his previous behavior. Saying Smoking is the reason you are here belittles the patient. Citing the doctors orders does not address the patients anxiety. Sanctioning smoking would be highly detrimental to this patient.

Diagnostic testing has revealed that a patients hepatocellular carcinoma (HCC) is limited to one lobe. The nurse should anticipate that this patients plan of care will focus on what intervention? A) Cryosurgery B) Liver transplantation C) Lobectomy D) Laser hyperthermia

C Surgical resection is the treatment of choice when HCC is confined to one lobe of the liver and the function of the remaining liver is considered adequate for postoperative recovery. Removal of a lobe of the liver (lobectomy) is the most common surgical procedure for excising a liver tumor. While cryosurgery and liver transplantation are other surgical options for management of liver cancer, these procedures are not performed at the same frequency as a lobectomy. Laser hyperthermia is a nonsurgical treatment for liver cancer.

A nurse is creating a care plan for a patient with acute pancreatitis. The care plan includes reduced activity. What rationale for this intervention should be cited in the care plan? A) Bed rest reduces the patients metabolism and reduces the risk of metabolic acidosis. B) Reduced activity protects the physical integrity of pancreatic cells. C) Bed rest lowers the metabolic rate and reduces enzyme production. D) Inactivity reduces caloric need and gastrointestinal motility.

C The acutely ill patient is maintained on bed rest to decrease the metabolic rate and reduce the secretion of pancreatic and gastric enzymes. Staying in bed does not release energy from the body to fight the disease.

A patient has been admitted to the postsurgical unit following the creation of an ileal conduit. What should the nurse measure to determine the size of the appliance needed? A) The circumference of the stoma B) The narrowest part of the stoma C) The widest part of the stoma D) Half the width of the stoma

C The correct appliance size is determined by measuring the widest part of the stoma with a ruler. The permanent appliance should be no more than 1.6 mm (1/8 inch) larger than the diameter of the stoma and the same shape as the stoma to prevent contact of the skin with drainage.

A nurse is caring for a patient who is in the diuresis phase of AKI. The nurse should closely monitor the patient for what complication during this phase? A) Hypokalemia B) Hypocalcemia C) Dehydration D) Acute flank pain

C The diuresis period is marked by a gradual increase in urine output, which signals that glomerular filtration has started to recover. The patient must be observed closely for dehydration during this phase; if dehydration occurs, the uremic symptoms are likely to increase. Excessive losses of potassium and calcium are not typical during this phase, and diuresis does not normally result in pain.

A patient who had surgery for gallbladder disease has just returned to the postsurgical unit from postanesthetic recovery. The nurse caring for this patient knows to immediately report what assessment finding to the physician? A) Decreased breath sounds B) Drainage of bile-colored fluid onto the abdominal dressing C) Rigidity of the abdomen D) Acute pain with movement

C The location of the subcostal incision will likely cause the patient to take shallow breaths to prevent pain, which may result in decreased breath sounds. The nurse should remind patients to take deep breaths and cough to expand the lungs fully and prevent atelectasis. Acute pain is an expected assessment finding following surgery; analgesics should be administered for pain relief. Abdominal splinting or application of an abdominal binder may assist in reducing the pain. Bile may continue to drain from the drainage tract after surgery, which will require frequent changes of the abdominal dressing. Increased abdominal tenderness and rigidity should be reported immediately to the physician, as it may indicate bleeding from an inadvertent puncture or nicking of a major blood vessel during the surgical procedure.

A nurse is providing discharge education to a patient who has undergone a laparoscopic cholecystectomy. During the immediate recovery period, the nurse should recommend what foods? A) High-fiber foods B) Low-purine, nutrient-dense foods C) Low-fat foods high in proteins and carbohydrates D) Foods that are low-residue and low in fat

C The nurse encourages the patient to eat a diet that is low in fats and high in carbohydrates and proteins immediately after surgery. There is no specific need to increase fiber or avoid purines. A low-residue diet is not indicated.

The hospice nurse is caring for a patient with cancer in her home. The nurse has explained to the patient and the family that the patient is at risk for hypercalcemia and has educated them on that signs and symptoms of this health problem. What else should the nurse teach this patient and family to do to reduce the patients risk of hypercalcemia? A) Stool softeners are contraindicated. B) Laxatives should be taken daily. C) Consume 2 to 4 L of fluid daily. D) Restrict calcium intake.

C The nurse should identify patients at risk for hypercalcemia, assess for signs and symptoms of hypercalcemia, and educate the patient and family. The nurse should teach at-risk patients to recognize and report signs and symptoms of hypercalcemia and encourage patients to consume 2 to 4 L of fluid daily unless contraindicated by existing renal or cardiac disease. Also, the nurse should explain the use of dietary and pharmacologic interventions, such as stool softeners and laxatives for constipation, and advise patients to maintain nutritional intake without restricting normal calcium intake.

A patient is brought to the ED by family members who tell the nurse that the patient has been exhibiting paranoid, agitated behavior. What should the nurse do when interacting with this patient? A) Keep the patient in a confined space. B) Use therapeutic touch appropriately. C) Give the patient honest answers about likely treatment. D) Attempt to convince the patient that his or her fears are unfounded.

C The nurse should offer appropriate and honest explanations in order to foster rapport and trust. Confinement is likely to cause escalation, as is touching the patient. The nurse should not normally engage in trying to convince the patient that his or her fears are unjustified, as this can also cause escalation.

The nurse is creating an education plan for a patient who underwent a nephrectomy for the treatment of a renal tumor. What should the nurse include in the teaching plan? A) The importance of increased fluid intake B) Signs and symptoms of rejection C) Inspection and care of the incision D) Techniques for preventing metastasis

C The nurse teaches the patient to inspect and care for the incision and perform other general postoperative care, including activity and lifting restrictions, driving, and pain management. There would be no need to teach the signs or symptoms of rejection as there has been no transplant. Increased fluid intake is not normally recommended and the patient has minimal control on the future risk for metastasis.

A female patient has been experiencing recurrent urinary tract infections. What health education should the nurse provide to this patient? A) Bathe daily and keep the perineal region clean. B) Avoid voiding immediately after sexual intercourse. C) Drink liberal amounts of fluids. D) Void at least every 6 to 8 hours.

C The patient is encouraged to drink liberal amounts of fluids (water is the best choice) to increase urine production and flow, which flushes the bacteria from the urinary tract. Frequent voiding (every 2 to 3 hours) is encouraged to empty the bladder completely because this can significantly lower urine bacterial counts, reduce urinary stasis, and prevent reinfection. The patient should be encouraged to shower rather than bathe.

A patient who has undergone liver transplantation is ready to be discharged home. Which outcome of health education should the nurse prioritize? A) The patient will obtain measurement of drainage from the T-tube. B) The patient will exercise three times a week. C) The patient will take immunosuppressive agents as required. D) The patient will monitor for signs of liver dysfunction.

C The patient is given written and verbal instructions about immunosuppressive agent doses and dosing schedules. The patient is also instructed on steps to follow to ensure that an adequate supply of medication is available so that there is no chance of running out of the medication or skipping a dose. Failure to take medications as instructed may precipitate rejection. The nurse would not teach the patient to measure drainage from a T-tube as the patient wouldnt go home with a T-tube. The nurse may teach the patient about the need to exercise or what the signs of liver dysfunction are, but the nurse would not stress these topics over the immunosuppressive drug regimen.

You are a floor nurse caring for a patient with alcohol withdrawal syndrome. What would be an appropriate nursing action to minimize the potential for hallucinations? A) Engage the patient in a process of health education. B) Administer opioid analgesics as ordered. C) Place the patient in a private, well-lit room. D) Provide television or a radio as therapeutic distraction

C The patient should be placed in a quiet single room with lights on and in a calm nonstressful environment. TV and radio stimulation should be avoided. Analgesics are not normally necessary, and would potentially contribute to hallucinations. Health education would be inappropriate while the patient is experiencing acute withdrawal.

The nurse is caring for a patient who underwent percutaneous lithotripsy earlier in the day. What instruction should the nurse give the patient? A) Limit oral fluid intake for 1 to 2 days. B) Report the presence of fine, sand like particles through the nephrostomy tube. C) Notify the physician about cloudy or foul-smelling urine. D) Report any pink-tinged urine within 24 hours after the procedure.

C The patient should report the presence of foul-smelling or cloudy urine since this is suggestive of a UTI. Unless contraindicated, the patient should be instructed to drink large quantities of fluid each day to flush the kidneys. Sand like debris is normal due to residual stone products. Hematuria is common after lithotripsy.

A patient has undergone rigid fixation for the correction of a mandibular fracture suffered in a fight. What area of care should the nurse prioritize when planning this patients discharge education? A) Resumption of activities of daily living B) Pain control C) Promotion of adequate nutrition D) Strategies for promoting communication

C The patient who has had rigid fixation should be instructed not to chew food in the first 1 to 4 weeks after surgery. A liquid diet is recommended, and dietary counseling should be obtained to ensure optimal caloric and protein intake. The nature of this surgery threatens the patients nutritional status; this physiologic need would likely supersede the resumption of ADLs. Pain should be under control prior to discharge and communication is not precluded by this surgery.

A patient with a recent history of nephrolithiasis has presented to the ED. After determining that the patients cardiopulmonary status is stable, what aspect of care should the nurse prioritize? A) IV fluid administration B) Insertion of an indwelling urinary catheter C) Pain management D) Assisting with aspiration of the stone

C The patient with kidney stones is often in excruciating pain, and this is a high priority for nursing interventions. In the short term, this would supersede the patients need for IV fluids or for catheterization. Kidney stones cannot be aspirated.

A patient has been admitted to the hospital for the treatment of chronic pancreatitis. The patient has been stabilized and the nurse is now planning health promotion and educational interventions. Which of the following should the nurse prioritize? A) Educating the patient about expectations and care following surgery B) Educating the patient about the management of blood glucose after discharge C) Educating the patient about postdischarge lifestyle modifications D) Educating the patient about the potential benefits of pancreatic transplantation

C The patients lifestyle (especially regarding alcohol use) is a major determinant of the course of chronic pancreatitis. The disease is not often managed by surgery and blood sugar monitoring is not necessarily indicated for every patient after hospital treatment. Transplantation is not an option.

What information will the nurse include for an overweight 35-year-old woman who is starting a weight- loss plan? A) Weigh yourself at the same time every morning and evening. B) Stick to a 600- to 800-calorie diet for the most rapid weight loss. C) Low carbohydrate diets lead to rapid weight loss but are difficult to maintain. D) Weighing all foods on a scale is necessary to choose appropriate portion sizes.

C The restrictive nature of fad diets makes the weight loss achieved by the patient more difficult to maintain. Portion size can be estimated in other ways besides weighing. Severely calorie-restricted diets are not necessary for patients in the overweight category of obesity and need to be closely supervised. Patients should weigh weekly rather than daily.

A patient with a fractured femur presenting to the ED exhibits cool, moist skin, increased heart rate, and falling BP. The care team should consider the possibility of what complication of the patients injuries? A) Myocardial infarction B) Hypoglycemia C) Hemorrhage D) Peritonitis

C The signs and symptoms the patient is experiencing suggest a volume deficit from an internal bleed. That the symptoms follow an acute injury suggests hemorrhage rather than myocardial infarction or hypoglycemia. Peritonitis would be an unlikely result of a femoral fracture.

A patient with a diagnosis of cirrhosis has developed variceal bleeding and will imminently undergo variceal banding. What psychosocial nursing diagnosis should the nurse most likely prioritize during this phase of the patients treatment? A) Decisional Conflict B) Deficient Knowledge C) Death Anxiety D) Disturbed Thought Processes

C The sudden hemorrhage that accompanies variceal bleeding is intensely anxiety-provoking. The nurse must address the patients likely fear of death, which is a realistic possibility. For most patients, anxiety is likely to be a more acute concern than lack of knowledge or decisional conflict. The patient may or may not experience disturbances in thought processes.

A nurse is addressing the prevention of esophageal cancer in response to a question posed by a participant in a health promotion workshop. What action has the greatest potential to prevent esophageal cancer? A) Promotion of a nutrient-dense, low-fat diet B) Annual screening endoscopy for patients over 50 with a family history of esophageal cancer C) Early diagnosis and treatment of gastroesophageal reflux disease D) Adequate fluid intake and avoidance of spicy foods

C There are numerous risk factors for esophageal cancer but chronic esophageal irritation or GERD is among the most significant. This is a more significant risk factor than dietary habits. Screening endoscopies are not recommended solely on the basis of family history.

A nurse is working with a patient who is learning to care for a continent ileostomy (Kock pouch). Following the initial period of healing, the nurse is teaching the patient how to independently empty the ileostomy. The nurse should teach the patient to do which of the following actions? A) Aim to eventually empty the pouch every 90 minutes. B) Avoid emptying the pouch until it is visibly full. C) Insert the catheter approximately 5 cm into the pouch. D) Aspirate the contents of the pouch using a 60 mL piston syringe.

C To empty a Kock pouch, the catheter is gently inserted approximately 5 cm to the point of the valve or nipple. The length of time between drainage periods is gradually increased until the reservoir needs to be drained only every 4 to 6 hours and irrigated once each day. It is not appropriate to wait until the pouch is full, and this would not be visible. The contents of the pouch are not aspirated.

A nurse is performing an admission assessment of a patient with a diagnosis of cirrhosis. What technique should the nurse use to palpate the patients liver? A) Place hand under the right lower abdominal quadrant and press down lightly with the other hand. B) Place the left hand over the abdomen and behind the left side at the 11th rib. C) Place hand under right lower rib cage and press down lightly with the other hand. D) Hold hand 90 degrees to right side of the abdomen and push down firmly

C To palpate the liver, the examiner places one hand under the right lower rib cage and presses downward with light pressure with the other hand. The liver is not on the left side or in the right lower abdominal quadrant.

The nurse is caring for a patient with an indwelling urinary catheter. The nurse is aware that what nursing action helps prevent infection in a patient with an indwelling catheter? A) Vigorously clean the meatus area daily. B) Apply powder to the perineal area twice daily. C) Empty the drainage bag at least every 8 hours. D) Irrigate the catheter every 8 hours with normal saline.

C To reduce the risk of bacterial proliferation, the nurse should empty the collection bag at least every 8 hours through the drainage spout, and more frequently if there is a large volume of urine. Vigorous cleaning of the meatus while the catheter is in place is discouraged, because the cleaning action can move the catheter, increasing the risk of infection. The spout (or drainage port) of any urinary drainage bag can become contaminated when opened to drain the bag. Irrigation of the catheter opens the closed system, increasing the likelihood of infection.

A 61-year-old man is being admitted for bariatric surgery. Which nursing action can the nurse delegate to unlicensed assistive personnel (UAP)? A) Demonstrate use of the incentive spirometer. B) Plan methods for bathing and turning the patient. C) Assist with IV insertion by holding adipose tissue out of the way. D) Develop strategies to provide privacy and decrease embarrassment.

C UAP can assist with IV placement by assisting with patient positioning or holding skinfolds aside. Planning for care and patient teaching require registered nurse (RN)level education and scope of practice.

The nurse is caring for a 54-year-old female patient on the first postoperative day after a Roux-en-Y gastric bypass procedure. Which assessment finding should be reported immediately to the surgeon? A) Bilateral crackles audible at both lung bases B) Redness, irritation, and skin breakdown in skinfolds C) Emesis of bile-colored fluid past the nasogastric (NG) tube D) Use of patient-controlled analgesia (PCA) several times an hour for pain

C Vomiting with an NG tube in place indicates that the NG tube needs to be repositioned by the surgeon to avoid putting stress on the gastric sutures. The nurse should implement actions to decrease skin irritation and have the patient cough and deep breathe, but these do not indicate a need for rapid notification of the surgeon. Frequent PCA use after bariatric surgery is expected.

A patient is brought by friends to the ED after being involved in a motor vehicle accident. The patient sustained blunt trauma to the abdomen. What nursing action would be most appropriate for this patient? A) Ambulate the patient to expel flatus. B) Place the patient in a high Fowlers position. C) Immobilize the patient on a backboard. D) Place the patient in a left lateral position.

C When admitted for blunt trauma, patients must be immobilized until spinal injury is ruled out. Ambulation, side-lying, and upright positioning would be contraindicated until spinal injury is ruled out.

The nurse is caring for a patient receiving hemodialysis three times weekly. The patient has had surgery to form an arteriovenous fistula. What is most important for the nurse to be aware of when providing care for this patient? A) Using a stethoscope for auscultating the fistula is contraindicated. B) The patient feels best immediately after the dialysis treatment. C) Taking a BP reading on the affected arm can damage the fistula. D) The patient should not feel pain during initiation of dialysis.

C When blood flow is reduced through the access for any reason (hypotension, application of BP cuff/tourniquet), the access site can clot. Auscultation of a bruit in the fistula is one way to determine patency. Typically, patients feel fatigued immediately after hemodialysis because of the rapid change in fluid and electrolyte status. Although the area over the fistula may have some decreased sensation, a needle stick is still painful.

An 84-year-old woman diagnosed with cancer is admitted to the oncology unit for surgical treatment. The patient has been on chemotherapeutic agents to decrease the tumor size prior to the planned surgery. The nurse caring for the patient is aware that what precipitating factors in this patient may contribute to AKI? Select all that apply. A) Anxiety B) Low BMI C) Age-related physiologic changes D) Chronic systemic disease E) NPO status

C, D Changes in kidney function with normal aging increase the susceptibility of elderly patients to kidney dysfunction and renal failure. In addition, the presence of chronic, systemic diseases increases the risk of AKI. Low BMI and anxiety are not risk factors for acute renal disease. NPO status is not a risk, provided adequate parenteral hydration is administered.

The nurse is performing an initial assessment of an older adult resident who has just relocated to the long-term care facility. During the nurses interview with the patient, she admits that she drinks around 20 ounces of vodka every evening. What types of cancer does this put her at risk for? Select all that apply. A) Malignant melanoma B) Brain cancer C) Breast cancer D) Esophageal cancer E) Liver cancer

C, D, E Dietary substances that appear to increase the risk of cancer include fats, alcohol, salt-cured or smoked meats, nitrate- and nitrite-containing foods, and red and processed meats. Alcohol increases the risk of cancers of the mouth, pharynx, larynx, esophagus, liver, colorectum, and breast.

A patient has been admitted to the medical unit with a diagnosis of ureteral colic secondary to urolithiasis. When planning the patients admission assessment, the nurse should be aware of the signs and symptoms that are characteristic of this diagnosis? Select all that apply. A) Diarrhea B) High fever C) Hematuria D) Urinary frequency E) Acute pain

C, D, E Stones lodged in the ureter (ureteral obstruction) cause acute, excruciating, colicky, wavelike pain, radiating down the thigh and to the genitalia. Often, the patient has a desire to void, but little urine is passed, and it usually contains blood because of the abrasive action of the stone. This group of symptoms is called ureteral colic. Diarrhea is not associated with this presentation and a fever is usually absent due to the noninfectious nature of the health problem.

A patient with esophageal varices is being cared for in the ICU. The varices have begun to bleed and the patient is at risk for hypovolemia. The patient has Ringers lactate at 150 cc/hr infusing. What else might the nurse expect to have ordered to maintain volume for this patient? A) Arterial line B) Diuretics C) Foley catheter D) Volume expanders

D Because patients with bleeding esophageal varices have intravascular volume depletion and are subject to electrolyte imbalance, IV fluids with electrolytes and volume expanders are provided to restore fluid volume and replace electrolytes. Diuretics would reduce vascular volume. An arterial line and Foley catheter are likely to be ordered, but neither actively maintains the patients volume.

A nurse is talking with a patient who is scheduled to have a hemicolectomy with the creation of a colostomy. The patient admits to being anxious, and has many questions concerning the surgery, the care of a stoma, and necessary lifestyle changes. Which of the following nursing actions is most appropriate? A) Reassure the patient that the procedure is relatively low risk and that patients are usually successful in adjusting to an ostomy. B) Provide the patient with educational materials that match the patients learning style. C) Encourage the patient to write down these concerns and questions to bring forward to the surgeon. D) Maintain an open dialogue with the patient and facilitate a referral to the wound-ostomy- continence (WOC) nurse.

D A wound-ostomy-continence (WOC) nurse is a registered nurse who has received advanced education in an accredited program to care for patients with stomas. The enterostomal nurse therapist can assist with the selection of an appropriate stoma site, teach about stoma care, and provide emotional support. The surgeon is less likely to address the patients psychosocial and learning needs. Reassurance does not address the patients questions, and education may or may not alleviate anxiety.

A patient has been treated in the hospital for an episode of acute pancreatitis. The patient has acknowledged the role that his alcohol use played in the development of his health problem, but has not expressed specific plans for lifestyle changes after discharge. What is the nurses most appropriate response? A) Educate the patient about the link between alcohol use and pancreatitis. B) Ensure that the patient knows the importance of attending follow-up appointments. C) Refer the patient to social work or spiritual care. D) Encourage the patient to connect with a community-based support group.

D After the acute attack has subsided, some patients may be inclined to return to their previous drinking habits. The nurse provides specific information about resources and support groups that may be of assistance in avoiding alcohol in the future. Referral to Alcoholics Anonymous as appropriate or other support groups is essential. The patient already has an understanding of the effects of alcohol, and follow-up appointments will not necessarily result in lifestyle changes. Social work and spiritual care may or may not be beneficial.

A patient has been diagnosed with a small bowel obstruction and has been admitted to the medical unit. The nurses care should prioritize which of the following outcomes? A) Preventing infection B) Maintaining skin and tissue integrity C) Preventing nausea and vomiting D) Maintaining fluid and electrolyte balance

D All of the listed focuses of care are important for the patient with a small bowel obstruction. However, the patients risk of fluid and electrolyte imbalances is an immediate threat to safety, and is a priority in nursing assessment and interventions.

A 55-year-old man has been newly diagnosed with acute pancreatitis and admitted to the acute medical unit. How should the nurse most likely explain the pathophysiology of this patients health problem? A) Toxins have accumulated and inflamed your pancreas. B) Bacteria likely migrated from your intestines and became lodged in your pancreas. C) A virus that was likely already present in your body has begun to attack your pancreatic cells. D) The enzymes that your pancreas produces have damaged the pancreas itself.

D Although the mechanisms causing pancreatitis are unknown, pancreatitis is commonly described as the autodigestion of the pancreas. Less commonly, toxic substances and microorganisms are implicated as the cause of pancreatitis.

An oncology nurse is contributing to the care of a patient who has failed to respond appreciably to conventional cancer treatments. As a result, the care team is considering the possible use of biologic response modifiers (BRFs). The nurse should know that these achieve a therapeutic effect by what means? A) Promoting the synthesis and release of leukocytes B) Focusing the patients immune system exclusively on the tumor C) Potentiating the effects of chemotherapeutic agents and radiation therapy D) Altering the immunologic relationship between the tumor and the patient

D BRFs alter the immunologic relationship between the tumor and the cancer patient (host) to provide a therapeutic benefit. They do not necessarily increase white cell production or focus the immune system solely on the tumor. BRFs do not potentiate radiotherapy and chemotherapy.

A nurse is performing health education with a patient who has a history of frequent, serious dental caries. When planning educational interventions, the nurse should identify a risk for what nursing diagnosis? A) Ineffective Tissue Perfusion B) Impaired Skin Integrity C) Aspiration D) Imbalanced Nutrition: Less Than Body Requirements

D Because digestion normally begins in the mouth, adequate nutrition is related to good dental health and the general condition of the mouth. Any discomfort or adverse condition in the oral cavity can affect a persons nutritional status. Dental caries do not typically affect the patients tissue perfusion or skin integrity. Aspiration is not a likely consequence of dental caries.

A patient is postoperative day 3 following the creation of an ileal conduit for the treatment of invasive bladder cancer. The patient is quickly learning to self-manage the urinary diversion, but expresses concern about the presence of mucus in the urine. What is the nurses most appropriate response? A) Report this finding promptly to the primary care provider. B) Obtain a sterile urine sample and send it for culture. C) Obtain a urine sample and check it for pH. D) Reassure the patient that this is an expected phenomenon.

D Because mucous membrane is used in forming the conduit, the patient may excrete a large amount of mucus mixed with urine. This causes anxiety in many patients. To help relieve this anxiety, the nurse reassures the patient that this is a normal occurrence after an ileal conduit procedure. Urine testing for culture or pH is not required.

A female patients most recent urinalysis results are suggestive of bacteriuria. When assessing this patient, the nurses data analysis should be informed by what principle? A) Most UTIs in female patients are caused by viruses and do not cause obvious symptoms. B) A diagnosis of bacteriuria requires three consecutive positive results. C) Urine contains varying levels of healthy bacterial flora. D) Urine samples are frequently contaminated by bacteria normally present in the urethral area.

D Because urine samples (especially in women) are commonly contaminated by the bacteria normally present in the urethral area, a bacterial count exceeding 105 colonies/mL of clean-catch, midstream urine is the measure that distinguishes true bacteriuria from contamination. A diagnosis does not require three consecutive positive results and urine does not contain a normal flora in the absence of a UTI. Most UTIs have a bacterial etiology.

A nurse provides care on a bone marrow transplant unit and is preparing a female patient for a hematopoietic stem cell transplantation (HSCT) the following day. What information should the nurse emphasize to the patients family and friends? A) Your family should likely gather at the bedside in case theres a negative outcome. B) Make sure she doesnt eat any food in the 24 hours before the procedure. C) Wear a hospital gown when you go into the patients room. D) Do not visit if youve had a recent infection.

D Before HSCT, patients are at a high risk for infection, sepsis, and bleeding. Visitors should not visit if they have had a recent illness or vaccination. Gowns should indeed be worn, but this is secondary in importance to avoiding the patients contact with ill visitors. Prolonged fasting is unnecessary. Negative outcomes are possible, but the procedure would not normally be so risky as to require the family to gather at the bedside.

A patient who has been diagnosed with cholecystitis is being discharged home from the ED to be scheduled for surgery later. The patient received morphine during the present ED admission and is visibly drowsy. When providing health education to the patient, what would be the most appropriate nursing action? A) Give written instructions to patient. B) Give verbal instructions to one of the patients family members. C) Telephone the patient the next day with verbal instructions. D) Give verbal and written instructions to patient and a family member.

D Before discharge, verbal and written instructions for continuing care are given to the patient and the family or significant others. Discharge teaching is completed prior to the patient leaving the ED, so phoning the patient the next day in not acceptable.

An oncology nurse is caring for a patient who has developed erythema following radiation therapy. What should the nurse instruct the patient to do? A) Periodically apply ice to the area. B) Keep the area cleanly shaven. C) Apply petroleum jelly to the affected area. D) Avoid using soap on the treatment area.

D Care to the affected area must focus on preventing further skin irritation, drying, and damage. Soaps, petroleum ointment, and shaving the area could worsen the erythema. Ice is also contraindicated.

A patient has been diagnosed with a malignancy of the oral cavity and is undergoing oncologic treatment. The oncologic nurse is aware that the prognosis for recovery from head and neck cancers is often poor because of what characteristic of these malignancies? A) Radiation therapy often results in secondary brain tumors. B) Surgical complications are exceedingly common. C) Diagnosis rarely occurs until the cancer is endstage. D) Metastases are common and respond poorly to treatment.

D Deaths from malignancies of the head and neck are primarily attributable to local-regional metastasis to the cervical lymph nodes in the neck. This often occurs by way of the lymphatics before the primary lesion has been treated. This local-regional metastasis is not amenable to surgical resection and responds poorly to chemotherapy and radiation therapy. This high mortality rate is not related to surgical complications, late diagnosis, or the development of brain tumors.

A nurse is caring for a patient with liver failure and is performing an assessment in the knowledge of the patients increased risk of bleeding. The nurse recognizes that this risk is related to the patients inability to synthesize prothrombin in the liver. What factor most likely contributes to this loss of function? A) Alterations in glucose metabolism B) Retention of bile salts C) Inadequate production of albumin by hepatocytes D) Inability of the liver to use vitamin K

D Decreased production of several clotting factors may be partially due to deficient absorption of vitamin K from the GI tract. This probably is caused by the inability of liver cells to use vitamin K to make prothrombin. This bleeding risk is unrelated to the roles of glucose, bile salts, or albumin.

A patient with liver cancer is being discharged home with a biliary drainage system in place. The nurse should teach the patients family how to safely perform which of the following actions? A) Aspirating bile from the catheter using a syringe B) Removing the catheter when output is 15 mL in 24 hours C) Instilling antibiotics into the catheter D) Assessing the patency of the drainage catheter

D Families should be taught to provide basic catheter care, including assessment of patency. Antibiotics are not instilled into the catheter and aspiration using a syringe is contraindicated. The family would not independently remove the catheter; this would be done by a member of the care team when deemed necessary.

A local public health nurse is informed that a cook in a local restaurant has been diagnosed with hepatitis A. What should the nurse advise individuals to obtain who ate at this restaurant and have never received the hepatitis A vaccine? A) The hepatitis A vaccine B) Albumin infusion C) The hepatitis A and B vaccines D) An immune globulin injection

D For people who have not been previously vaccinated, hepatitis A can be prevented by the intramuscular administration of immune globulin during the incubation period, if given within 2 weeks of exposure. Administration of the hepatitis A vaccine will not protect the patient exposed to hepatitis A, as protection will take a few weeks to develop after the first dose of the vaccine. The hepatitis B vaccine provides protection again the hepatitis B virus, but plays no role in protection for the patient exposed to hepatitis A. Albumin confers no therapeutic benefit.

An emergency department nurse is admitting a 3-year-old brought in after swallowing a piece from a wooden puzzle. The nurse should anticipate the administration of what medication in order to relax the esophagus to facilitate removal of the foreign body? A) Haloperidol B) Prostigmine C) Epinephrine D) Glucagon

D Glucagon is administered prior to removal of a foreign body because it relaxes the smooth muscle of the esophagus, facilitating insertion of the endoscope. Haloperidol is an antipsychotic drug and is not indicated. Prostigmine is prescribed for patients with myastheniagravis. It increases muscular contraction, an effect opposite that which is desired to facilitate removal of the foreign body. Epinephrine is indicated in asthma attack and bronchospasm.

A nurse is providing care for a patient whose neck dissection surgery involved the use of a graft. When assessing the graft, the nurse should prioritize data related to what nursing diagnosis? A) Risk for Disuse Syndrome B) Unilateral Neglect C) Risk for Trauma D) Ineffective Tissue Perfusion

D Grafted skin is highly vulnerable to inadequate perfusion and subsequent ischemia and necrosis. This is a priority over chronic pain, which is unlikely to be a long-term challenge. Neglect and disuse are not risks related to the graft site.

The nurse is caring for a patient has just been given a 6-month prognosis following a diagnosis of extensive stage small-cell lung cancer. The patient states that he would like to die at home, but the team believes that the patients care needs are unable to be met in a home environment. What might you suggest as an alternative? A) Discuss a referral for rehabilitation hospital. B) Panel the patient for a personal care home. C) Discuss a referral for acute care. D) Discuss a referral for hospice care.

D Hospice care can be provided in several settings. Because of the high cost associated with free-standing hospices, care is often delivered by coordinating services provided by both hospitals and the community. The primary goal of hospice care is to provide support to the patient and family. Patients who are referred to hospice care generally have fewer than 6 months to live. Each of the other listed options would be less appropriate for the patients physical and psychosocial needs.

A radial graft is planned in the treatment of a patients oropharyngeal cancer. In order to ensure that the surgery will be successful, the care team must perform what assessment prior to surgery? A) Assessing function of cranial nerves V, VI, and IX B) Assessing for a history of GERD C) Assessing for signs or symptoms of atherosclerosis D) Assessing the patency of the ulnar artery

D If a radial graft is to be performed, an Allen test on the donor arm must be performed to ensure that the ulnar artery is patent and can provide blood flow to the hand after removal of the radial artery. The success of this surgery is not primarily dependent on CN function or the absence of GERD and atherosclerosis.

A triage nurse is talking to a patient when the patient begins choking on his lunch. The patient is coughing forcefully. What should the nurse do? A) Stand him up and perform the abdominal thrust maneuver from behind. B) Lay him down, straddle him, and perform the abdominal thrust maneuver. C) Leave him to get assistance. D) Stay with him and encourage him, but not intervene at this time.

D If the patient is coughing, he should be able to dislodge the object or cause a complete obstruction. If complete obstruction occurs, the nurse should perform the abdominal thrust maneuver with the patient standing. If the patient is unconscious, the nurse should lay the patient down. A nurse should never leave a choking patient alone.

A patient presents to the emergency department (ED) complaining of severe right upper quadrant pain. The patient states that his family doctor told him he had gallstones. The ED nurse should recognize what possible complication of gallstones? A) Acute pancreatitis B) Atrophy of the gallbladder C) Gallbladder cancer D) Gangrene of the gallbladder

D In calculous cholecystitis, a gallbladder stone obstructs bile outflow. Bile remaining in the gallbladder initiates a chemical reaction; autolysis and edema occur; and the blood vessels in the gallbladder are compressed, compromising its vascular supply. Gangrene of the gallbladder with perforation may result. Pancreatitis, atrophy, and cancer of the gallbladder are not plausible complications.

A nurse is caring for an older adult who has been experiencing severeClostridium difficile-related diarrhea. When reviewing the patients most recent laboratory tests, the nurse should prioritize which of the following? A) White blood cell level B) Creatinine level C) Hemoglobin level D) Potassium level

D In elderly patients, it is important to monitor the patients serum electrolyte levels closely. Diarrhea is less likely to cause an alteration in white blood cell, creatinine, and hemoglobin levels.

A patient with liver cancer is being discharged home with a hepatic artery catheter in place. The nurse should be aware that this catheter will facilitate which of the following? A) Continuous monitoring for portal hypertension B) Administration of immunosuppressive drugs during the first weeks after transplantation C) Real-time monitoring of vascular changes in the hepatic system D) Delivery of a continuous chemotherapeutic dose

D In most cases, the hepatic artery catheter has been inserted surgically and has a prefilled infusion pump that delivers a continuous chemotherapeutic dose until completed. The hepatic artery catheter does not monitor portal hypertension, deliver immunosuppressive drugs, or monitor vascular changes in the hepatic system.

A nurse is caring for a patient who is postoperative day 1 following neck dissection surgery. The nurse is performing an assessment of the patient and notes the presence of high-pitched adventitious sounds over the patients trachea on auscultation. The patients oxygen saturation is 90% by pulse oximetry with a respiratory rate of 31 breaths per minute. What is the nurses most appropriate action? A) Encourage the patient to perform deep breathing and coughing exercises hourly. B) Reposition the patient into a prone or semi-Fowlers position and apply supplementary oxygen by nasal cannula. C) Activate the emergency response system. D) Report this finding promptly to the physician and remain with the patient.

D In the immediate postoperative period, the nurse assesses for stridor (coarse, high-pitched sound on inspiration) by listening frequently over the trachea with a stethoscope. This finding must be reported immediately because it indicates obstruction of the airway. The patients current status does not warrant activation of the emergency response system, and encouraging deep breathing and repositioning the patient are inadequate responses.

An oncology patient will begin a course of chemotherapy and radiation therapy for the treatment of bone metastases. What is one means by which malignant disease processes transfer cells from one place to another? A) Adhering to primary tumor cells B) Inducing mutation of cells of another organ C) Phagocytizing healthy cells D) Invading healthy host tissues

D Invasion, which refers to the growth of the primary tumor into the surrounding host tissues, occurs in several ways. Malignant cells are less likely to adhere than are normal cells. Malignant cells do not cause healthy cells to mutate. Malignant cells do not eat other cells.

A patient who attempted suicide being treated in the ED is accompanied by his mother, father, and brother. When planning the nursing care of this family, the nurse should perform which of the following action? A) Refer the family to psychiatry in order to provide them with support. B) Explore the causes of the patients suicide attempt with the family. C) Encourage the family to participate in the bedside care of the patient. D) Ensure that the family receives appropriate crisis intervention services.

D It is essential that family crisis intervention services are available for families of ED patients. It would be inappropriate and insensitive to explore causes of the patients suicide attempt with the family. Family participation in bedside care is often impractical in the ED setting. Psychiatry is not the normal source of psychosocial support and crisis intervention.

A patient seeking care because of recurrent heartburn and regurgitation is subsequently diagnosed with a hiatal hernia. Which of the following should the nurse include in health education? A) Drinking beverages after your meal, rather than with your meal, may bring some relief. B) Its best to avoid dry foods, such as rice and chicken, because theyre harder to swallow. C) Many patients obtain relief by taking over-the-counter antacids 30 minutes before eating. D) Instead of eating three meals a day, try eating smaller amounts more often.

D Management for a hiatal hernia includes frequent, small feedings that can pass easily through the esophagus. Avoiding beverages and particular foods or taking OTC antacids are not noted to be beneficial.

A nurse is caring for a patient who is acutely ill and has included vigilant oral care in the patients plan of care. Why are patients who are ill at increased risk for developing dental caries? A) Hormonal changes brought on by the stress response cause an acidic oral environment B) Systemic infections frequently migrate to the teeth C) Hydration that is received intravenously lacks fluoride D) Inadequate nutrition and decreased saliva production can cause cavities

D Many ill patients do not eat adequate amounts of food and therefore produce less saliva, which in turn reduces the natural cleaning of the teeth. Stress response is not a factor, infections generally do not attack the enamel of the teeth, and the fluoride level of the patient is not significant in the development of dental caries in the ill patient.

An older adult has experienced a new onset of urinary incontinence and family members identify this problem as being unprecedented. When assessing the patient for factors that may have contributed to incontinence, the nurse should prioritize what assessment? A) Reviewing the patients 24-hour food recall for changes in diet B) Assessing for recent contact with individuals who have UTIs C) Assessing for changes in the patients level of psychosocial stress D) Reviewing the patients medication administration record for recent changes

D Many medications affect urinary continence in addition to causing other unwanted or unexpected effects. Stress and dietary changes could potentially affect the patients continence, but medications are more frequently causative of incontinence. UTIs can cause incontinence, but these infections do not result from contact with infected individuals.

A nurses colleague has applied an incontinence pad to an older adult patient who has experienced occasional episodes of functional incontinence. What principle should guide the nurses management of urinary incontinence in older adults? A) Diuretics should be promptly discontinued when an older adult experiences incontinence. B) Restricting fluid intake is recommended for older adults experiencing incontinence. C) Urinary catheterization is a first-line treatment for incontinence in older adults with incontinence. D) Urinary incontinence is not considered a normal consequence of aging.

D Nursing management is based on the premise that incontinence is not inevitable with illness or aging and that it is often reversible and treatable. Diuretics cannot always be safely discontinued. Fluid restriction and catheterization are not considered to be safe, first-line interventions for the treatment of incontinence.

A patient being treated in the hospital has been experiencing occasional urinary retention. What nursing action should the nurse take to encourage a patient who is having difficulty voiding? A) Use a slipper bedpan. B) Apply a cold compress to the perineum. C) Have the patient lie in a supine position. D) Provide privacy for the patient.

D Nursing measures to encourage normal voiding patterns include providing privacy, ensuring an environment and body position conducive to voiding, and assisting the patient with the use of the bathroom or bedside commode, rather than a bedpan, to provide a more natural setting for voiding. Most people find supine positioning not conducive to voiding.

A nurse is participating in the emergency care of a patient who has just developed variceal bleeding. What intervention should the nurse anticipate? A) Infusion of intravenous heparin B) IV administration of albumin C) STAT administration of vitamin K by the intramuscular route D) IV administration of octreotide (Sandostatin)

D Octreotide (Sandostatin)a synthetic analog of the hormone somatostatinis effective in decreasing bleeding from esophageal varices, and lacks the vasoconstrictive effects of vasopressin. Because of this safety and efficacy profile, octreotide is considered the preferred treatment regimen for immediate control of variceal bleeding. Vitamin K and albumin are not administered and heparin would exacerbate, not alleviate, bleeding.

The nurse performing the health interview of a patient with a new onset of periorbital edema has completed a genogram, noting the health history of the patients siblings, parents, and grandparents. This assessment addresses the patients risk of what kidney disorder? A) Nephritic syndrome B) Acute glomerulonephritis C) Nephrotic syndrome D) Polycystic kidney disease (PKD)

D PKD is a genetic disorder characterized by the growth of numerous cysts in the kidneys. Nephritic syndrome, acute glomerulonephritis, and nephrotic syndrome are not genetic disorders.

A patient has been diagnosed with pancreatic cancer and has been admitted for care. Following initial treatment, the nurse should be aware that the patient is most likely to require which of the following? A) Inpatient rehabilitation B) Rehabilitation in the home setting C) Intensive physical therapy D) Hospice care

D Pancreatic carcinoma has only a 5% survival rate at 5 years regardless of the stage of disease at diagnosis or treatment. As a result, there is a higher likelihood that the patient will require hospice care than physical therapy and rehabilitation.

A patient has been experiencing disconcerting GI symptoms that have been worsening in severity. Following medical assessment, the patient has been diagnosed with lactose intolerance. The nurse should recognize an increased need for what form of health promotion? A) Annual screening colonoscopies B) Adherence to recommended immunization schedules C) Regular blood pressure monitoring D) Frequent screening for osteoporosis

D Persons with lactose intolerance often experience hypocalcemia and a consequent risk of osteoporosis related to malabsorption of calcium. Lactose intolerance does not create an increased need for screening for colorectal cancer, immunizations, or blood pressure monitoring.

A 15-year-old is admitted to the renal unit with a diagnosis of postinfectious glomerular disease. The nurse should recognize that this form of kidney disease may have been precipitated by what event? A) Psychosocial stress B) Hypersensitivity to an immunization C) Menarche D) Streptococcal infection

D Postinfectious causes of postinfectious glomerular disease are group A beta-hemolytic streptococcal infection of the throat that precedes the onset of glomerulonephritis by 2 to 3 weeks. Menarche, stress, and hypersensitivity are not typical causes.

A patient has been experiencing occasional episodes of constipation and has been unable to achieve consistent relief by increasing physical activity and improving his diet. What pharmacologic intervention should the nurse recommend to the patient for ongoing use? A) Mineral oil enemas B) Bisacodyl (Dulcolax) C) Senna (Senokot) D) Psyllium hydrophilic mucilloid (Metamucil)

D Psyllium hydrophilic mucilloid (Metamucil) is a bulk-forming laxative that is safe for ongoing use. None of the other listed laxatives should be used on an ongoing basis because of the risk of dependence.

An oncology patient has begun to experience skin reactions to radiation therapy, prompting the nurse to make the diagnosis Impaired Skin Integrity: erythematous reaction to radiation therapy. What intervention best addresses this nursing diagnosis? A) Apply an ice pack or heating pad PRN to relieve pain and pruritis B) Avoid skin contact with water whenever possible C) Apply phototherapy PRN D) Avoid rubbing or scratching the affected area

D Rubbing and or scratching will lead to additional skin irritation, damage, and increased risk of infection. Extremes of hot, cold, and light should be avoided. No need to avoid contact with water.

A 62-year-old woman diagnosed with breast cancer is scheduled for a partial mastectomy. The oncology nurse explained that the surgeon will want to take tissue samples to ensure the disease has not spread to adjacent axillary lymph nodes. The patient has asked if she will have her lymph nodes dissected, like her mother did several years ago. What alternative to lymph node dissection will this patient most likely undergo? A) Lymphadenectomy B) Needle biopsy C) Open biopsy D) Sentinel node biopsy

D Sentinel lymph node biopsy (SLNB), also known as sentinel lymph node mapping, is a minimally invasive surgical approach that, in some instances, has replaced more invasive lymph node dissections (lymphadenectomy) and their associated complications such as lymphedema and delayed healing. SLNB has been widely adopted for regional lymph node staging in selected cases of melanoma and breast cancer.

A community health nurse is preparing for an initial home visit to a patient discharged following a total gastrectomy for treatment of gastric cancer. What would the nurse anticipate that the plan of care is most likely to include? A) Enteral feeding via gastrostomy tube (G tube) B) Gastrointestinal decompression by nasogastric tube C) Periodic assessment for esophageal distension D) Monthly administration of injections of vitamin B12

D Since vitamin B12 is absorbed in the stomach, the patient requires vitamin B12 replacement to prevent pernicious anemia. A gastrectomy precludes the use of a G tube. Since the stomach is absent, a nasogastric tube would not be indicated. As well, this is not possible in the home setting. Since there is no stomach to act as a reservoir and fluids and nutrients are passing directly into the jejunum, distension is unlikely.

A nurse is admitting a patient diagnosed with late-stage gastric cancer. The patients family is distraught and angry that she was not diagnosed earlier in the course of her disease. What factor contributes to the fact that gastric cancer is often detected at a later stage? A) Gastric cancer does not cause signs or symptoms until metastasis has occurred. B) Adherence to screening recommendations for gastric cancer is exceptionally low. C) Early symptoms of gastric cancer are usually attributed to constipation. D) The early symptoms of gastric cancer are usually not alarming or highly unusual.

D Symptoms of early gastric cancer, such as pain relieved by antacids, resemble those of benign ulcers and are seldom definitive. Symptoms are rarely a cause for alarm or for detailed diagnostic testing. Symptoms precede metastasis, however, and do not include constipation.

A nurse is caring for a patient who has been the victim of sexual assault. The nurse documents that the patient appears to be in a state of shock, verbalizing fear, guilt, and humiliation. What phase of rape trauma syndrome is this patient most likely experiencing? A) Reorganization phase B) Denial phase C) Heightened anxiety phase D) Acute disorganization phase

D The acute disorganization phase may manifest as an expressed state in which shock, disbelief, fear, guilt, humiliation, anger, and other such emotions are encountered. These varied responses to the assault are not associated with a denial, heightened anxiety, or reorganization phase.

A patient with chronic kidney disease has been hospitalized and is receiving hemodialysis on a scheduled basis. The nurse should include which of the following actions in the plan of care? A) Ensure that the patient moves the extremity with the vascular access site as little as possible. B) Change the dressing over the vascular access site at least every 12 hours. C) Utilize the vascular access site for infusion of IV fluids. D) Assess for a thrill or bruit over the vascular access site each shift.

D The bruit, or thrill, over the venous access site must be evaluated at least every shift. Frequent dressing changes are unnecessary and the patient does not normally need to immobilize the site. The site must not be used for purposes other than dialysis.

A nurse is assessing an elderly patient with gallstones. The nurse is aware that the patient may not exhibit typical symptoms, and that particular symptoms that may be exhibited in the elderly patient may include what? A) Fever and pain B) Chills and jaundice C) Nausea and vomiting D) Signs and symptoms of septic shock

D The elderly patient may not exhibit the typical symptoms of fever, pain, chills jaundice, and nausea and vomiting. Symptoms of biliary tract disease in the elderly may be accompanied or preceded by those of septic shock, which include oliguria, hypotension, change in mental status, tachycardia, and tachypnea.

After vertical banded gastroplasty, a 42-year-old male patient returns to the surgical nursing unit with a nasogastric tube to low, intermittent suction and a patient-controlled analgesia (PCA) machine for pain control. Which nursing action should be included in the postoperative plan of care? A) Offer sips of fruit juices at frequent intervals. B) Irrigate the nasogastric (NG) tube frequently. C) Remind the patient that PCA use may slow the return of bowel function. D) Support the surgical incision during patient coughing and turning in bed.

D The incision should be protected from strain to decrease the risk for wound dehiscence. The patient should be encouraged to use the PCA because pain control will improve the cough effort and patient mobility. NG irrigation may damage the suture line or overfill the stomach pouch. Sugar-free clear liquids are offered during the immediate postoperative time to decrease the risk for dumping syndrome.

A patient has presented with signs and symptoms that are characteristic of acute kidney injury, but preliminary assessment reveals no obvious risk factors for this health problem. The nurse should recognize the need to interview the patient about what topic? A) Typical diet B) Allergy status C) Psychosocial stressors D) Current medication use

D The kidneys are susceptible to the adverse effects of medications because they are repeatedly exposed to substances in the blood. Nephrotoxic medications are a more likely cause of AKI than diet, allergies, or stress.

A patient admitted with nephrotic syndrome is being cared for on the medical unit. When writing this patients care plan, based on the major clinical manifestation of nephrotic syndrome, what nursing diagnosis should the nurse include? A) Constipation related to immobility B) Risk for injury related to altered thought processes C) Hyperthermia related to the inflammatory process D) Excess fluid volume related to generalized edema

D The major clinical manifestation of nephrotic syndrome is edema, so the appropriate nursing diagnosis is Excess fluid volume related to generalized edema. Edema is usually soft, pitting, and commonly occurs around the eyes, in dependent areas, and in the abdomen.

A nurse is performing an admission assessment for an 81-year-old patient who generally enjoys good health. When considering normal, age-related changes to hepatic function, the nurse should anticipate what finding? A) Similar liver size and texture as in younger adults B) A nonpalpable liver C) A slightly enlarged liver with palpably hard edges D) A slightly decreased size of the liver

D The most common age-related change in the liver is a decrease in size and weight. The liver is usually still palpable, however, and is not expected to have hardened edges.

A nurse is teaching a group of adults about screening and prevention of colorectal cancer. The nurse should describe which of the following as the most common sign of possible colon cancer? A) Development of new hemorrhoids B) Abdominal bloating and flank pain C) Unexplained weight gain D) Change in bowel habits

D The most common presenting symptom associated with colorectal cancer is a change in bowel habits. The passage of blood is the second most common symptom. Symptoms may also include unexplained anemia, anorexia, weight loss, and fatigue. Hemorrhoids and bloating are atypical.

A gerontologic nurse is assessing a patient who has numerous comorbid health problems. What assessment findings should prompt the nurse to suspect a UTI? Select all that apply. A) Food cravings B) Upper abdominal pain C) Insatiable thirst D) Uncharacteristic fatigue E) New onset of confusion

D The most common subjective presenting symptom of UTI in older adults is generalized fatigue. The most common objective finding is a change in cognitive functioning. Food cravings, increased thirst, and upper abdominal pain necessitate further assessment and intervention, but none is directly suggestive of a UTI.

A patient returns to the floor after a laparoscopic cholecystectomy. The nurse should assess the patient for signs and symptoms of what serious potential complication of this surgery? A) Diabetic coma B) Decubitus ulcer C) Wound evisceration D) Bile duct injury

D The most serious complication after laparoscopic cholecystectomy is a bile duct injury. Patients do not face a risk of diabetic coma. A decubitus ulcer is unlikely because immobility is not expected. Evisceration is highly unlikely, due to the laparoscopic approach.

A home health nurse is caring for a patient discharged home after pancreatic surgery. The nurse documents the nursing diagnosis Risk for Imbalanced Nutrition: Less than Body Requirements on the care plan based on the potential complications that may occur after surgery. What are the most likely complications for the patient who has had pancreatic surgery? A) Proteinuria and hyperkalemia B) Hemorrhage and hypercalcemia C) Weight loss and hypoglycemia D) Malabsorption and hyperglycemia

D The nurse arrives at this diagnosis based on the complications of malabsorption and hyperglycemia. These complications often lead to the need for dietary modifications. Pancreatic enzyme replacement, a low-fat diet, and vitamin supplementation often are also required to meet the patients nutritional needs and restrictions. Electrolyte imbalances often accompany pancreatic disorders and surgery, but the electrolyte levels are more often deficient than excessive. Hemorrhage is a complication related to surgery, but not specific to the nutritionally based nursing diagnosis. Weight loss is a common complication, but hypoglycemia is less likely.

A patient is being treated on the acute medical unit for acute pancreatitis. The nurse has identified a diagnosis of Ineffective Breathing Pattern Related to Pain. What intervention should the nurse perform in order to best address this diagnosis? A) Position the patient supine to facilitate diaphragm movement. B) Administer corticosteroids by nebulizer as ordered. C) Perform oral suctioning as needed to remove secretions. D) Maintain the patient in a semi-Fowlers position whenever possible.

D The nurse maintains the patient in a semi-Fowlers position to decrease pressure on the diaphragm by a distended abdomen and to increase respiratory expansion. A supine position will result in increased pressure on the diaphragm and potentially decreased respiratory expansion. Steroids and oral suctioning are not indicated.

The nurse is orienting a new nurse to the oncology unit. When reviewing the safe administration of antineoplastic agents, what action should the nurse emphasize? A) Adjust the dose to the patients present symptoms. B) Wash hands with an alcohol-based cleanser following administration. C) Use gloves and a lab coat when preparing the medication. D) Dispose of the antineoplastic wastes in the hazardous waste receptacle.

D The nurse should use surgical gloves and disposable long-sleeved gowns when administering antineoplastic agents. The antineoplastic wastes are disposed of as hazardous materials. Dosages are not adjusted on a short-term basis. Hand and arm hygiene must be performed before and after administering the medication.

The nurse is collaborating with the wound-ostomy-continence (WOC) nurse to teach a patient how to manage her new ileal conduit in the home setting. To prevent leakage or skin breakdown, the nurse should encourage which of the following practices? A) Empty the collection bag when it is between one-half and two-thirds full. B) Limit fluid intake to prevent production of large volumes of dilute urine. C) Reinforce the appliance with tape if small leaks are detected. D) Avoid using moisturizing soaps and body washes when cleaning the peristomal area.

D The patient is instructed to avoid moisturizing soaps and body washes when cleaning the area because they interfere with the adhesion of the pouch. To maintain skin integrity, a skin barrier or leaking pouch is never patched with tape to prevent accumulation of urine under the skin barrier or faceplate. Fluids should be encouraged, not limited, and the collection bag should not be allowed to become more than one-third full.

A nurse is assessing a patient who has been diagnosed with cholecystitis, and is experiencing localized abdominal pain. When assessing the characteristics of the patients pain, the nurse should anticipate that it may radiate to what region? A) Left upper chest B) Inguinal region C) Neck or jaw D) Right shoulder

D The patient may have biliary colic with excruciating upper right abdominal pain that radiates to the back or right shoulder. Pain from cholecystitis does not typically radiate to the left upper chest, inguinal area, neck, or jaw.

An obtunded patient is admitted to the ED after ingesting bleach. The nurse should prepare to assist with what intervention? A) Prompt administration of an antidote B) Gastric lavage C) Administration of activated charcoal D) Helping the patient drink large amounts of water

D The patient who has ingested a corrosive poison, such as bleach, is given water or milk to drink for dilution. Gastric lavage is not used to treat ingestion of corrosives and activated charcoal is ineffective. There is no antidote for a corrosive substance such as bleach.

Which of the following is the most plausible nursing diagnosis for a patient whose treatment for colon cancer has necessitated a colonostomy? A) Risk for Unstable Blood Glucose Due to Changes in Digestion and Absorption B) Unilateral Neglect Related to Decreased Physical Mobility C) Risk for Excess Fluid Volume Related to Dietary Changes and Changes In Absorption D) Ineffective Sexuality Patterns Related to Changes in Self-Concept

D The presence of an ostomy frequently has an effect on sexuality; this should be addressed thoughtfully in nursing care. None of the other listed diagnoses reflects the physiologic changes that result from colorectal surgery.

A patients large bowel obstruction has failed to resolve spontaneously and the patients worsening condition has warranted admission to the medical unit. Which of the following aspects of nursing care is most appropriate for this patient? A) Administering bowel stimulants as ordered B) Administering bulk-forming laxatives as ordered C) Performing deep palpation as ordered to promote peristalsis D) Preparing the patient for surgical bowel resection

D The usual treatment for a large bowel obstruction is surgical resection to remove the obstructing lesion. Administration of laxatives or bowel stimulants are contraindicated if the bowel is obstructed. Palpation would be painful and has no therapeutic benefit.

A patient has a flaccid bladder secondary to a spinal cord injury. The nurse recognizes this patients high risk for urinary retention and should implement what intervention in the patients plan of care? A) Relaxation techniques B) Sodium restriction C) Lower abdominal massage D) Double voiding

D To enhance emptying of a flaccid bladder, the patient may be taught to double void. After each voiding, the patient is instructed to remain on the toilet, relax for 1 to 2 minutes, and then attempt to void again in an effort to further empty the bladder. Relaxation does not affect the neurologic etiology of a flaccid bladder. Sodium restriction and massage are similarly ineffective.

An adult patient has been hospitalized with pyelonephritis. The nurses review of the patients intake and output records reveals that the patient has been consuming between 3 L and 3.5 L of oral fluid each day since admission. How should the nurse best respond to this finding? A) Supplement the patients fluid intake with a high-calorie diet. B) Emphasize the need to limit intake to 2 L of fluid daily. C) Obtain an order for a high-sodium diet to prevent dilutional hyponatremia. D) Encourage the patient to continue this pattern of fluid intake.

D Unless contraindicated, 3 to 4 L of fluids per day is encouraged to dilute the urine, decrease burning on urination, and prevent dehydration. No need to supplement this fluid intake with additional calories or sodium.

A nurse is amending a patients plan of care in light of the fact that the patient has recently developed ascites. What should the nurse include in this patients care plan? A) Mobilization with assistance at least 4 times daily B) Administration of beta-adrenergic blockers as ordered C) Vitamin B12 injections as ordered D) Administration of diuretics as ordered

D Use of diuretics along with sodium restriction is successful in 90% of patients with ascites. Beta- blockers are not used to treat ascites and bed rest is often more beneficial than increased mobility. Vitamin B12 injections are not necessary.

A patient is brought to the ER in an unconscious state. The physician notes that the patient is in need of emergency surgery. No family members are present, and the patient does not have identification. What action by the nurse is most important regarding consent for treatment? A) Ask the social worker to come and sign the consent. B) Contact the police to obtain the patients identity. C) Obtain a court order to treat the patient. D) Clearly document LOC and health status on the patients chart.

D When patients are unconscious and in critical condition, the condition and situation should be documented to administer treatment quickly and timely when no consent can be obtained by usual routes. A social worker is not asked to sign the consent. Finding the patients identity is not a priority. Obtaining a court order would take too long.

The nurse is caring for a patient with a history of systemic lupus erythematosus who has been recently diagnosed with end-stage kidney disease (ESKD). The patient has an elevated phosphorus level and has been prescribed calcium acetate to bind the phosphorus. The nurse should teach the patient to take the prescribed phosphorus-binding medication at what time? A) Only when needed B) Daily at bedtime C) First thing in the morning D) With each meal

D Both calcium carbonate and calcium acetate are medications that bind with the phosphate and assist in excreting the phosphate from the body, in turn lowering the phosphate levels. Phosphate-binding medications must be administered with food to be effective.


Set pelajaran terkait

Genetics (BIO 352) Final Exam URI

View Set

React Native top 80 interview questions

View Set

MGMT 309: Chapter 4 "Responding to the Ethical and Social Environment"

View Set

Chapter 23: Asepsis, & Infection Control

View Set